SlideShare una empresa de Scribd logo
1 de 43
Descargar para leer sin conexión
Capítulo 1
Lógica matemática
1.1. Formas proposicionales
La lógica matemática se ocupa del análisis de las proposiciones y demostraciones del razona-
miento lógico, proporciona ideas claras y precisas sobre la naturaleza de la conclusión deductiva,
desarrolla el pensamiento funcional y hace una contribución esencial al desarrollo del pensamiento
científico y creador. Esto se manifiesta, por ejemplo, en la correcta comprensión de las estructuras
lógicas y las tareas formales, en el reconocimiento de las semejanzas de los diferentes fenómenos
lógicos, en la aplicación de las leyes y reglas lógicas y en la pretensión de claridad, sencillez y
economía en la expresión lingüística.
Una de las propiedades de la forma de expresión matemática, es la de representar los objetos,
las imágenes mentales, los vínculos y las relaciones mediante símbolos (signos), y combinarlos entre
sí.
Definición 1.1 Constante
Una constante es un signo que tiene una determinada significación fija.
Es decir; una constante tiene, en todo el desarrollo de una investigación o en la solución de una
tarea, siempre la misma significación.
Definición 1.2 Variable
Una variable es un signo que representa cualquier elemento de un dominio básico previamente
establecido.
Esto quiere decir que una variable se puede sustituir por el signo de cualquier elemento del
dominio básico. Entonces se habla de la sustitución de la variable, o de la interpretación de la
variable.
Definición 1.3 Término
Por término entendemos las constantes, las variables y sus combinaciones mediante los signos de
operación y los signos técnicos.
Los términos son, por tanto, las denominaciones de los objetos matemáticos o las combinacio-
nes de signos donde se presentan variables, constantes y signos de operaciones, y que mediante la
interpretación de las variables se omiten en las designaciones de los objetos matemáticos. El ob-
jeto matemático, identificado como un término, y en cuya denominación se omite este calificativo
1
CAPÍTULO 1. LÓGICA MATEMÁTICA 2
después de la interpretación de las variables, se conoce como valor del término.
Las proposiciones son estructuras lingüísticas cuyo valor de verdad es, o verdadero o falso. La
lógica clásica, a través de sus axiomas y principios, ha hecho algunas consideraciones sobre el con-
tenido de verdad de una proposición. El principio de la bivalencia expresa: Toda proposición o es
falsa o es verdadera.
De este principio se pueden deducir dos teoremas.
1. El teorema de la tercera posibilidad excluida, expresa:
Toda proposición es falsa o verdadera.
2. El teorema de la contradicción excluida, expresa:
Ninguna proposición es falsa y verdadera al mismo tiempo.
En las observaciones posteriores veremos que los dos teoremas, considerados en conjunto, ex-
presan exactamente lo mismo que el principio de la bivalencia. Por consiguiente, se puede proceder
a la inversa; es decir deducir el teorema de la bivalencia a partir del principio de la tercera posibi-
lidad excluida y del principio de la contradicción excluida.
A cada proposición se le hace corresponder un valor de verdad, o falso F o verdadero V. Es por
esta razón que también se habla de una lógica bivalente. La asignación de los valores de verdad F o
V de una proposición, no es tan sencillo de determinar. Aunque en el principio de la bivalencia se
expresa claramente que una proposición es falsa o verdadera, no se puede decir inmediatamente si
cada proposición es falsa o verdadera. En matemáticas existen actualmente muchas proposiciones
que hasta el momento no han podido ser demostradas, concebida, la demostración, como una
aseveración de la verdad, a continuación se dan dos ejemplos de este tipo de proposiciones.
Ejemplo 1.1 La proposición: ¨Todo número par que sea mayor que 4, se puede representar
como la suma de dos números primos, excepto el 2¨, existe desde el año 1742. Hasta el momento
no se ha podido demostrar si es una proposición falsa o verdadera. (Suposición de Goldbach).
Definición 1.4 Forma proposicional
Una estructura lingüística que contiene por lo menos una variable libre, se convierte en una pro-
posición, cuando se sustituyen todas las variables por símbolos, que denotan objetos del dominio
básico, recibe el nombre de forma proposicional.
Ejemplo 1.2 8 + x <12 con x ∈ N no representa evidentemente ninguna proposición. Esta
sucesión de signos no es ni falsa ni verdadera. Mediante las sustituciones de la variable x podemos
formar proposiciones falsas y verdaderas. Así, con las sustituciones 0, 1, 2, 3 obtenemos siempre
proposiciones verdaderas, y cualquier otra sustitución dará lugar a proposiciones falsas. En es-
te caso, encontramos una expresión lingüística especial que no es una proposición, pero que, sin
embargo, se convierte en una proposición mediante la sustitución de la variable.
A las expresiones matemáticas de este tipo se las denomina formas proposicionales. Las variables
en tales expresiones se denominan variables libres. Las formas proposicionales surgen cuando entre
los términos que contienen variables se coloca un determinado signo de relación. De forma análoga
al convenio establecido para la notación de términos, denotamos una forma proposicional con
P(x1, x2, ..., xn). Todos los elementos cuyos símbolos convierten una forma proposicional en una
proposición, constituyen el conjunto solución de esta forma proposicional. El conjunto solución
comprende solamente aquellos elementos del dominio básico cuyos símbolos convierten una forma
proposicional en una proposición verdadera. Las formas proposicionales se pueden clasificar en la
forma siguiente: aquellas formas proposicionales que mediante una sustitución por lo menos, se
CAPÍTULO 1. LÓGICA MATEMÁTICA 3
pueden transformar en una proposición verdadera, se denominan interpretables. Todas las demás
se denominan no interpretables. Entre las interpretables se destacan las formas proposicionales de
validez general, que son aquellas que al hacer cualquier sustitución por los elementos del dominio
básico se convierten en una proposición verdadera.
Ejemplo 1.3 (x + y)2
= x2
+ 2xy + y2
∀ x, y ∈ R. En toda sustitución de x e y por elementos
del dominio básico se obtiene una proposición verdadera. Este ejemplo es, por tanto, una forma
proposicional de validez general en el conjunto de los números reales. El conjunto solución es el
conjunto de todos los pares (x, y) donde x e y son elementos de un dominio básico; luego, en este
caso coincide con el conjunto base de solución. Este ejemplo trata entonces de una identidad.
Ejemplo 1.4 Sea (x + y)2
= x2
+ y2
∀ x, y ∈ R.
(x + y)2
x2
+ y2
Valor de verdad
0 0 V
9 9 V
36 36 V
9 5 F
361 193 F
La presente tabla muestra que a partir de esta forma proposicional se pueden obtener propo-
siciones falsas y verdaderas. El conjunto solución es, un subconjunto propio del conjunto base de
solución. El conjunto solución consta, de los pares ordenados de elementos del dominio básico. Este
ejemplo trata entonces de una neutralidad.
Ejemplo 1.5 x2
− 5x + 10 = 0 x ∈ R. En el dominio básico no hay elementos que satisfagan
esta forma proposicional, es decir, toda sustitución la convierte en una proposición falsa. Este
ejemplo trata por consiguiente de una contradicción.
1.1.1. Operaciones entre proposiciones lógicas
En esta sección trataremos exclusivamente las proposiciones y las formas proposicionales. Pri-
meramente, introduciremos algunas combinaciones de proposiciones, mediante las cuales a su vez se
obtienen otras proposiciones. Después obtendremos mediante definiciones las funciones proposicio-
nales y más tarde las funciones veritativas. En todas las operaciones con proposiciones señalaremos
el proceso de abstracción circunstancia - proposiciones - valores de verdad.
Definición 1.5 Proposición
Denominaremos proposición a una frase narrativa que puede calificarse como verdadera o falsa,
pero no ambas al mismo tiempo.
Los valores verdadero y falso mencionados en la definición se denominan valores de certeza o
valores de verdad. Así cuando una proposición se considere verdadera o falsa diremos que dicha
proposición tiene valor de certeza verdadero o falso.
Ejemplo 1.6 Las siguientes frases son proposiciones:
- La tierra es plana.
- 547 es un número primo.
- Los números irracionales son complejos.
- Los números complejos son un subconjunto de los reales.
- La Escuela Politécnica del Ejercito es un instituto de educación superior.
- No es verdadero que 3 sea un entero par o 7 un primo.
CAPÍTULO 1. LÓGICA MATEMÁTICA 4
- 2n = n2
para alguna n ∈ N.
- 289301 + 1 es un número primo.
- Si un árbol tiene n vértices, entonces tiene exactamente n - 1 aristas.
- 2n + n es un número primo para una infinidad de n.
- Todo entero par mayor que 4 es la suma de dos números primos.
- Las matemáticas son divertidas.
- Los árboles son más interesantes que las matrices.
Ejemplo 1.7 Las siguientes frases no son proposiciones:
- ¿Porqué es importante la lógica proposicional?
- 323789 ext 205
- ¿Porqué es importante la inducción?
- x - y = y - x.
Es importante hacer notar que el valor de verdad de una proposición no es trabajo ni parte
de la lógica aquí tratada, por tal razón dichos valores los supondremos ya asignados. Todas las
proposiciones constituyen una clase que, a su vez, se descompone en dos subclases, en la clase de
las proposiciones verdaderas V y en la clase de las proposiciones falsas F.
La verdad o falsedad de las proposiciones no puede ser demostrada inmediatamente en todos
los casos, pero, para toda proposición, independientemente de que aún no haya sido comprobada ni
refutada, solo cabe una de las dos posibilidades, es verdadera V o es falsa F. El proceso de negación
lo denominamos operación lógica de un lugar. Los enlaces de dos proposiciones, como resultado de
los cuales se obtiene una proposición única se denominan operaciones lógicas de dos lugares.
Definición 1.6 Función proposicional de n-lugares
Cuando a cada n-uplo de proposiciones se le hace corresponder unívocamente una proposición, esta
correspondencia se denomina función proposicional de n-lugares.
Se entiende por n-uplo, un conjunto de n elementos dependientes del orden, en este caso pro-
posiciones. De todas las funciones proposicionales, las llamadas funciones proposicionales clásicas
tienen una gran importancia por las razones siguientes:
a) porque las restantes funciones se pueden representar en términos de estas.
b) porque en la lógica formal tradicional se han tratado especialmente las cinco siguientes fun-
ciones:
Nombre Argumento Funciones proposicionales Número de lugares
Negación P No P Uno
Conjunción P, Q P y Q Dos
Disyunción P, Q P o Q Dos
Implicación P, Q Si P, entonces Q Dos
Equivalencia P, Q P exactamente cuando Q Dos
En estas funciones proposicionales el valor de verdad de la proposición resultante depende so-
lamente de los valores de verdad de los argumentos correspondientes, y no de su contenido, y se
denominan funciones proposicionales extensionales. Además de las funciones proposicionales clási-
cas existen otras funciones proposicionales que son extensionales.
En el transcurso de las observaciones hemos hecho abstracción del contenido concreto de las
proposiciones o de los enlaces de proposiciones y, alcanzado las etapas de abstracción de las fun-
ciones proposicionales.
CAPÍTULO 1. LÓGICA MATEMÁTICA 5
Continuamos el proceso de abstracción sobre la base de las afirmaciones ya hechas. Si también
realizamos el paso de transición de las proposiciones a los valores de verdad, entonces obtenemos
las funciones veritativas correspondientes a las funciones proposicionales.
Definición 1.7 Función veritativa de n-lugares
Cuando a cada n-uplo de valores de verdad se le hace corresponder unívocamente un valor de
verdad, entonces esta correspondencia recibe el nombre de función veritativa de n lugares.
Las funciones proposicionales y las funciones veritativas pertenecen a diferentes niveles de abs-
tracción. Por este motivo es conveniente introducir otros símbolos para las funciones veritativas.
Con la negación de una proposición queremos expresar la idea de que esto no se refiere a la cir-
cunstancia que a ella corresponde. Cuando negamos una proposición P, entonces obtenemos otra
proposición ¬P, es decir, la negación de P. A través de esta operación obtenemos una proposición
cuyo valor de verdad es contrario al valor de verdad de P.
Frecuentemente en el lenguaje común, una negación se expresa mediante prefijos que indican
negación o mediante adverbios de negación. Por ejemplo, se dice en lugar de no regular, irregular,
y en lugar de no un, simplemente ningún, etc. Al formular ciertas negaciones pueden surgir con
mucha facilidad algunas confusiones, cuando simplemente se expresa la negación mediante antóni-
mos o contrarios.
Por ejemplo, negro y blanco, pequeño y grande, positivo y negativo, orden y caos son, en cierto
sentido, contrarios que no pueden ser expresados a través de una negación. Aquí definiremos la
negación como una función veritativa, aunque la denominación de negación se utiliza también para
la función proposicional ¬P y para la operación negación.
Definición 1.8 Negación
Se denomina negación a la función veritativa de un lugar, cuyos valores se fijan de la manera
siguiente:
P ¬ P
V F
F V
La negación corresponde a la función proposicional de un lugar ¬P. La afirmación ¬P es verda-
dera cuando la proposición P es falsa, y ¬P es falsa cuando la proposición P es verdadera. Mediante
la negación de una proposición P se obtiene una nueva proposición ¬P cuyo valor de verdad es
opuesto al valor de verdad de P.
Ejemplo 1.8 Sea P: ¨Los billetes de $ 5000 contienen la efigie de Rumiñahui¨.
La negación de P es la proposición:
¬P: ¨Los billetes de $ 5000 no contienen la efigie de Rumiñahui¨.
Dadas las proposiciones P, Q consideremos la construcción de proposiciones de la forma (P y
Q).
Ejemplo 1.9 En la búsqueda de un profesor la ESPE publica el siguiente aviso:
¨Se solicita profesor con especialidad en Pedagogía y Álgebra¨.
Se presentan cuatro candidatos A, B, C y D con las características siguientes:
A: Tiene ambas especialidades (V, V)
B: Sólo tiene la especialidad de Pedagogía (V, F)
C: Sólo tiene la especialidad de Álgebra (F, V)
CAPÍTULO 1. LÓGICA MATEMÁTICA 6
D: Sólo tiene la especialidad de Química (F, F)
Como podemos darnos cuenta, puesto que A cumplió con los requisitos, entonces A es el ganador
del empleo.
Definición 1.9 Conjunción
Se denomina conjunción o producto lógico de las proposiciones P y Q, dadas en este orden, a la
función veritativa de dos lugares que se obtiene enunciando Q luego de enunciar P, unidas ambas
por la palabra ¨y¨, cuyos valores de verdad se fijan de la siguiente manera:
P Q P ∧ Q
V V V
V F F
F V F
F F F
El enlace de proposiciones P ∧ Q es verdadera cuando ambas proposiciones P, Q son verda-
deras. Una conjunción cuyo valor de verdad es V, expresa que las circunstancias que se reflejan
a través de las proposiciones parciales, existen en conjunto. Pero, cuando una conjunción tiene el
valor de verdad F, esto significa que, por lo menos, una de sus proposiciones parciales no refleja co-
rrectamente una circunstancia. En las explicaciones posteriores, consideraremos el enlace de varias
proposiciones como una conjunción si aparece la expresión y/o sus sinónimos.
Ejemplo 1.10 Sean las proposiciones:
P: ¨2 es un divisor de 10¨
Q: ¨5 es un divisor de 10¨
La conjunción de P y Q es la siguiente proposición:
P ∧ Q: ¨2 es un divisor de 10, pero también 5 es un divisor de 10¨
Por lo tanto la proposición P ∧ Q es verdadera.
Estudiaremos ahora proposiciones de la forma (P o Q) y (o P o Q).
Ejemplo 1.11 Consideremos ahora el siguiente aviso:
¨Se solicita profesor con especialidad de Pedagogía o Álgebra¨
Se presentan cuatro candidatos A, B, C y D con las características siguientes:
A: Tiene ambas especialidades (V, V)
B: Sólo tiene la especialidad de Pedagogía (V, F)
C: Sólo tiene la especialidad de Algebra (F, V)
D: Sólo tiene la especialidad de Química (F, F)
En este caso sólo D no podrá ser seleccionado.
El punto central de esta parte lo constituye el uso de la palabra ¨o¨, la cual puede ser utilizada
en un sentido exclusivista ¨o ... o ...¨ o no exclusivista. Por este motivo procederemos en dos pasos
intermedios.
Definición 1.10 Disyunción
Se denomina disyunción a la función veritativa bivalente cuyos valores se fijan de la manera si-
guiente:
P Q P ∨ Q
V V V
V F V
F V V
F F F
CAPÍTULO 1. LÓGICA MATEMÁTICA 7
La disyunción corresponde a la función proposicional bivalente P o Q. Según la definición
anterior P o Q es verdadera cuando, como mínimo, una de las proposiciones enlazadas es verdadera.
En discusiones posteriores, el enlace de varias proposiciones con ¨o¨ recibe el nombre de disyunción.
Entonces, un enlace de proposiciones de este tipo representa una proposición verdadera cuando
todas las proposiciones enlazadas son verdaderas.
Ejemplo 1.12 - 2 · 3 = 6 ó 3 + 2 = 5
- 75 % de 45 m es 135/4 m, ó 33,75 m.
A causa de la extensionalidad de las funciones proposicionales las proposiciones enlazadas pue-
den o no, tener relaciones de contenido entre sí. La abstracción hecha de las relaciones de contenido
entre las proposiciones enlazadas es necesaria para poder fundamentar la relación lógica. Mediante
la definición anterior se ha determinado el sentido de la palabra ¨o¨. Para nosotros son de gran
interés aún los valores de verdad de las proposiciones parciales. En otros enlaces se procederá de
forma similar.
Definición 1.11 Alternativa
Se denomina alternativa a la función veritativa de dos lugares cuyos valores se fijan de la manera
siguiente:
P Q P ∨ Q
V V V
V F V
F V V
F F F
La alternativa corresponde a la función proposicional de dos lugares ¨o P o Q¨ es verdadero
cuando una de las dos proposiciones es verdadera. Es falso cuando ambas proposiciones son verda-
deras o falsas. La alternativa es igualmente extensional.
En las explicaciones que demos posteriormente, un enlace de más de dos proposiciones con ¨o
... o¨ recibe el nombre de alternativa. En el uso diario del lenguaje se dice frecuentemente ¨o¨ en
lugar de ¨o ... o¨, actuando esta palabra, en tales casos, de forma excluyente.
Cuando en el lenguaje familiar corriente se habla de una disyunción, se hace referente a la
alternativa que hemos definido. Estos hechos hay que tenerlos siempre en cuenta.
Ejemplo 1.13 - La suma de los siete primeros números naturales es o par o impar.
- 1969 es o un número primo o divisible por 9.
Un verdadero enlace mediante la alternativa de ambas proposiciones refleja que de dos circuns-
tancias posibles existe exactamente una. Para evitar las confusiones se debe utilizar, en tales casos,
siempre ¨o ... o¨. ¨o¨ puede usarse en el lenguaje común pero con otro sentido, cuando se quiere
decir que las dos circunstancias enlazadas entre sí no pueden existir en conjunto. Como máximo,
esto puede referirse a una de ellas.
El conocimiento de estas distintas interpretaciones de ¨o¨ en el lenguaje común es muy im-
portante para la conclusión lógica y, además, nos motiva a velar por la exactitud de nuestras
formulaciones.
A continuación formularemos, mediante la disyunción, algunos teoremas importantes de la
lógica de las proposiciones.
CAPÍTULO 1. LÓGICA MATEMÁTICA 8
Teorema 1.1 Toda proposición es verdadera o falsa.
Este teorema se denomina, teorema del tercero excluido. Esto podemos representarlo mediante
la función proposicional ¨P o ¬P¨. Esta función proposicional es una identidad, porque para
cualquier argumento P, siempre obtenemos una proposición verdadera.
Teorema 1.2 Toda proposición es o verdadera o falsa.
Este teorema se denomina, principio de la bivalencia. Este principio expresa que entre una
proposición y su negación no hay una tercera posibilidad, y que una proposición no puede ser
simultáneamente verdadera y falsa.
Proposiciones como P ∧ Q y P ∨ Q que resultan de combinar otras proposiciones reciben el
nombre de proposiciones compuestas.
Es posible una proposición compuesta G ∼= G(P1, P2, ..., Pn) sea verdadera sin importar qué
asignaciones de verdad se hayan hecho a las proposiciones P1, P2, ..., Pn.
Ejemplo 1.14 En el Instituto de Ciencias Básicas existe el siguiente reglamento: ¨Para que
un estudiante pueda tomar materias de avance de primer nivel, tiene que haber aprobado materias
concatenadas de prepolitécnico¨. ¿En cuáles de los siguientes casos se viola el reglamento?
A: Toma avances y aprobó materias concatenadas (V, V).
B: Toma avances y no aprobó materias concatenadas (V, F).
C: No toma avances pero aprobó materias concatenadas (F, V).
D: Ni toma avances ni aprobó materias concatenadas (F, F).
Un poco de reflexión nos conduce a aceptar que se viola el reglamento en el caso B.
Definición 1.12 Implicación
Se denomina implicación o condicional a la función veritativa de dos lugares cuyos valores de
verdad se fijan de la manera siguiente:
P Q P → Q
V V V
V F F
F V V
F F V
La proposición P se denomina hipótesis o antecedente y la proposición Q, conclusión o conse-
cuente. Considérese el problema de asignar un valor de verdad a la proposición implicación ¨si P,
entonces Q¨. En efecto, si la hipótesis P es verdadera y la conclusión Q es también verdadera (esto
es, la hipótesis y la conclusión son ambas verdaderas), entonces la proposición condicional ¨si P,
entonces Q¨ debe ser verdadera.
Por otra parte, si la hipótesis P es verdadera y la conclusión Q es falsa, entonces ¨si P, enton-
ces Q¨ debe ser falsa. (No se debe deducir una conclusión falsa de una hipótesis verdadera). La
definición normal dice que ¨si P, entonces Q¨ es verdadera en caso de que la hipótesis P sea falsa,
sin considerar el valor de verdad de la conclusión Q. En las ulteriores explicaciones, al enlace de
varias proposiciones con ¨si P, entonces Q¨ lo llamaremos implicación.
En el lenguaje ordinario, la hipótesis y la conclusión en una proposición implicación están
normalmente relacionadas, pero en lógica no se requiere que la hipótesis y la conclusión en una
CAPÍTULO 1. LÓGICA MATEMÁTICA 9
proposición implicación se refieran al mismo tema.
Tienen interés particular las proposiciones implicación verdaderas. Los teoremas de matemáti-
cas con frecuencia se expresan como proposiciones implicación. Una demostración de un teorema
de esta forma lo constituye la verificación de que la proposición implicación es verdadera.
Sean P ∼= P(P1, P2, ..., Pn) y Q ∼= Q(P1, P2, ..., Pn) proposiciones compuestas y supóngase
que P → Q es verdadera. Se sabe que si P es falsa, P → Q es verdadera, no importando si Q es
verdadera o falsa. Por otra parte, si P es verdadera, Q también debe serlo, pues en caso contrario
P → Q sería falsa.
Ejemplo 1.15 Sean las siguientes proposiciones:
P : Hoy es 30 de Febrero.
Q : Entre 5 y 15 hay números primos.
P → Q : Si hoy es 30 de febrero, entonces entre 5 y 15 hay números primos.
Esta proposición compuesta es verdadera, ya que podemos decir que toda implicación, cuyo primer
miembro sea falso tiene el valor de verdad V, sin tener en cuenta si el primero y el segundo
miembros tienen relación de contenido o no.
Ejemplo 1.16 La proposición compuesta ¨si entre 5 y 15 hay números primos, entonces entre
13 y 15 hay números primos¨, es falsa, ya que el primer miembro de esta implicación es verdadero
y su segundo miembro es falso.
Otra proposición compuesta de gran utilidad es
P si y sólo si Q
Este enunciado se interpreta como:
(Si P, entonces Q) y (si Q, entonces P)
Determínese el valor de verdad de la primera proposición. Supóngase que P y Q son ambas
verdaderas. Entonces las dos proposiciones implicación de la segunda son verdaderas. Y como la
conectiva ¨y¨ resulta verdadera para ambas verdaderas, se tiene que la segunda también lo es.
Dado que la primera se interpreta como la segunda, se considera que la primera es verdadera
cuando ambas P y Q lo son. Si P y Q son falsas, nuevamente las dos proposiciones implicación
de la segunda son verdaderas. En consecuencia, la segunda es verdadera. Por lo tanto, si ambas
proposiciones P y Q son falsas, se considera que la primera es verdadera.
Si P es falsa y Q es verdadera, entonces la segunda proposición implicación en la segunda es
falsa. Ahora bien, cuando en la conectiva ¨y¨ uno de los valores es falso, el resultado es falso. Por
consiguiente, se considera que la primera es falsa si P es falsa y Q es verdadera. Esto motiva la
siguiente definición.
Definición 1.13 Equivalencia
Se denomina implicación o condicional a la función veritativa de dos lugares cuyos valores de
verdad se fijan de la manera siguiente:
P Q P ↔ Q
V V V
V F F
F V F
F F V
CAPÍTULO 1. LÓGICA MATEMÁTICA 10
Otra forma de enunciar ¨P si y sólo si Q¨ es ¨P es una condición necesaria y suficiente para
Q¨.
Así mismo, ¨P si y sólo si Q¨ en ocasiones se escribe ¨P ssi Q¨.
La proposición compuesta (Si P, entonces Q) y (si Q, entonces P), también podemos expresarla en
símbología lógica de la siguiente manera
P ↔ Q ∼= (P → Q) ∧ (Q → P)
Ejemplo 1.17 Sean
P: El número 2013 es divisible por 3.
Q: La suma de las cifras básicas de 2013 es divisible por 3.
P ↔ Q: El número 2013 es divisible por 3 cuando la sumas de sus cifras básicas es divisible por 3.
Esta proposición es verdadera, ya que ambos enlaces son verdaderos.
Ejemplo 1.18 Si P → Q es una proposición implicación, entonces denominamos:
Q → P recíproca de P → Q.
¬P → ¬Q inversa de P → Q.
¬Q → ¬P contrapositiva de P → Q.
Ejemplo 1.19 Implicación: Si 2272 es divisible por 4, entonces 2272 es un número par.
Recíproca: Si 2272 es un número par, entonces 2272 es divisible por 4.
Contrapositiva: Si 2272 no es un número par, entonces 2272 no es divisible por 4.
Inversa: Si 2272 no es divisible por 4, entonces 2272 no es un número par.
Ejemplo 1.20 Implicación: Si un triángulo es equilátero, entonces es isósceles.
Recíproca: Si un triángulo es isósceles, entonces es equilátero.
Contrapositiva: Si un triángulo no es isósceles, entonces tampoco es equilátero.
Inversa: Si un triángulo no es equilátero, entonces tampoco es isósceles.
1.1.2. Tarea
1. Suponga que x, y, z ∈ R. Represente en forma simbólica los enunciados dados tomando:
P: x < y, Q: y < z, R: x < z
a) (x ≥ y e y < z) o x ≥ z;
b) No es cierto que (x < y e y < z);
c) x < y o no es verdad que (y < z y x < z);
d) (No es verdad que (x < y y (x < z o y < z))) o (x ≥ y y x < z).
Resp: a) ; b) ; c) ; d) .
2. Sean P, Q, R las proposiciones:
P: Está lloviendo, Q: El Sol está brillando, R: Hay nubes en el cielo.
Traduzca la siguiente notación lógica, utilizando P, Q, R y conectivos lógicos.
a) Está lloviendo y el Sol está brillando;
b) Si está lloviendo, entonces hay nubes en el cielo;
c) Si no está lloviendo, entonces el Sol no está brillando y hay nubes en el cielo;
d) El Sol está brillando si y sólo si no está lloviendo;
e) Si no hay nubes en el cielo, entonces el Sol está brillando.
Resp: a) ; b) ; c) ; d) ; e) .
CAPÍTULO 1. LÓGICA MATEMÁTICA 11
3. Sean P, Q, R como en el ejercicio anterior. Traduzca lo siguiente a oraciones en español:
a) (P ∧ Q) → R; b) (P → R) → Q; c) ¬P ↔ (Q ∨ R); d) ¬(P ↔ (Q ∨ R));
e) ¬(P ∨ Q) ∧ R.
Resp: a) ; b) ; c) ; d) ; e) .
4. Sean p : tengo un loro y q : tengo un gato, escribir en lenguaje corriente y luego simplificar
¬(¬p ∨ ¬(¬q)) ∧ ¬(¬p)
Resp: p ∧ (¬q): tengo un loro y no tengo un gato.
5. A un blanco se han efectuado tres tiros. Sea Pi la proposición ¨el blanco ha sido batido
por el i-ésimo tiro¨, i = 1, 2, 3. ¿Qué significan las siguientes proposiciones:
a) P1 ∨ P2 ∨ P3; b) P1 ∧ P2 ∧ P3; c) (¬P1 ∨ ¬P2) ∧ P3?
¿Cuáles de estas tres proposiciones son verdaderas si P3 es verdadera y P1 y P2, falsas?
Resp: a) ; b) ; c) .
6. ¿Cuáles de las siguientes expresiones son proposiciones? Proporcione los valores de verdad
de las proposiciones:
a) x2
= x para toda x ∈ R; b) x2
= x para alguna x ∈ R; c) x2
= x;
d) x2
= x para exactamente una x ∈ R; e) xy = xz implica y = z;
f) xy = xz implica y = z para toda x, y, z ∈ R.
Resp: a) ; b) ; c) ; d) ; e) ; f) .
7. Considere la frase ambigua x2
= y2
implica x = y para todo x, y:
a) Transforme esta frase en una proposición no ambigua cuyo valor de verdad sea verdadero;
b) Transforme esta frase en una proposición no ambigua cuyo valor de verdad sea falso.
Resp: a) ; b) .
8. Formule verbalmente las expresiones simbólicas contenidas en los siguientes literales, uti-
lizando las proposiciones:
P : Hoy es lunes, Q : Está lloviendo, R : Hace calor.
a) ¬P ∧ (Q ∨ R); b) ¬(P ∨ Q) ∧ R; c) (P ∧ (Q ∨ R)) ∧ (R ∨ (Q ∨ P));
d) (P ∨ (¬P ∧ ¬(Q ∨ R))) ∧ (P ∨ ¬(R ∨ Q)); e) ¬P → (Q ∨ R); f) ¬(P ∨ Q) ↔ R;
g) (P ∧ (Q ∨ R)) → (R ∨ (Q ∨ P)); h) (P ∨ (¬P ∧ ¬(Q ∨ R))) → (P ∨ ¬(R ∨ Q)).
Resp: a) ; b) ; c) ; d) ; e) ; f) ; g) ; h) .
9. En los siguientes literales, represente cada proposición en la forma de una proposición
condicional:
a) Para todo número x ∈ R, |x| < 2 siempre que 0 < x < 2;
b) Una condición suficiente para que una función f sea integrable es que f sea continua.
Resp: a) ; b) .
10. Enuncie la recíproca, la inversa y la contrapositiva de cada uno de los literales del ejercicio
anterior.
Resp: .
11. Proporcione las recíprocas, las inversas y las contrapositivas de las siguientes proposiciones:
a) Si soy listo entonces soy rico; b) Si x2
= x entonces x = 0 o x = 1;
c) Si 2 + 2 = 4 entonces 2 + 4 = 8.
Resp: a) ; b) ; c) .
CAPÍTULO 1. LÓGICA MATEMÁTICA 12
12. a) Muestre que n = 3 es un contraejemplo de la afirmación n3
< 3n para toda n ∈ N;
b) ¿Puede encontrar otros contraejemplos?
Resp: a) ; b) .
13. a) Muestre que x = −1 es un contraejemplo de (x + 1)2
≥ x2
para toda x ∈ R;
b) Encuentre otro contraejemplo;
c) ¿Puede servir de contraejemplo cualquier número no negativo? Explique su respuesta.
Resp: a) ; b) ; c) .
14. Encuentre contraejemplos de las siguientes afirmaciones:
a) 2n
− 1 es primo para toda n ≥ 2; b) 2n
+ 3n
es primo para toda n ∈ N;
c) 2n
+ n es primo para todo entero impar positivo n.
Resp: a) ; b) ; c) .
15. a) Proporcione un contraejemplo para: x > y implica x2
> y2
para toda x, y ∈ R. Su
respuesta debe ser un par ordenado;
b) ¿Cómo debe restringir x e y para que sea verdadera la proposición de la parte a)?
Resp: a) ; b) .
16. Exprese en forma simbólica cada uno de los enunciados, suponiendo que x, y, z ∈ R y que
P : x < y, Q : y < z, R : x < z :
a) Si x < y, entonces y ≥ z; b) Si (x < y e y < z), entonces x < z;
c) Si (x ≥ y e y < z), entonces x ≥ z;
d) Si no es verdad que (x < z e y < z), entonces x ≥ z;
e) x < y si y sólo si (y < z y x < z);
f) Si es falso que (x < y y (ya sea x < y o y < z)), entonces (x ≥ y, entonces x < z).
Resp: a) ; b) ; c) ; d) ; e) ; f) .
17. ¿Cuáles de las proposiciones P, Q, R deben ser verdaderas y cuáles falsas para que
(¬(¬P ∨ P) ∧ Q) → R
sea verdadera?
Resp: .
18. Represente simbólicamente cada una de las proposiciones condicionales dadas a continua-
ción. Escriba su recíproca, inversa y contrapositiva tanto con símbolos como con palabras.
Determine también el valor de verdad para la proposición condicional, para su recíproca,
inversa y para su contrapositiva:
a) Si 4 < 6, entonces 9 > 12; b) Si 4 > 6, entonces 9 > 12;
c) |1| < 3 si −3 < 1 < 3; d) |4| < 3 si −3 < 4 < 3.
Resp: a) ; b) ; c) ; d) .
19. Proporcione la recíproca, inversa y contrapositiva de cada una de las siguientes proposi-
ciones:
a) Si x + y = 1 entonces x2
+ y2
≥ 1; b) Si 2 + 2 = 4 entonces 3 + 3 = 8.
Resp: a) ; b) .
20. Considere la proposición: si x > 0 entonces x2
> 0 para x ∈ N:
a) Proporcione la recíproca, inversa y contrapositiva de la proposición;
b) ¿Cuál de las siguientes proposiciones es verdadera: la proposición original, la recíproca,
la inversa o la contrapositiva?
Resp: a) ; b) .
CAPÍTULO 1. LÓGICA MATEMÁTICA 13
21. Determine los valores de verdad de las siguientes proposiciones compuestas:
a) Si 2 + 2 = 4, entonces 2 + 4 = 8; b) Si 2 + 2 = 5, entonces 2 + 4 = 8;
c) Si 2 + 2 = 4, entonces 2 + 4 = 6; d) Si 2 + 2 = 5, entonces 2 + 4 = 6;
e) Si la tierra es plana, entonces Vicente Rocafuerte fue el primer presidente de Ecuador;
f) Si la tierra es plana, entonces Sixto Durán-Ballen es presidente de Ecuador en el periódo
92 - 96;
g) Si Sixto Durán-Ballen es presidente de Ecuador en el periódo 92 - 96, entonces la tierra
es plana;
h) Si Sixto Durán-Ballen es presidente de Ecuador en el periódo 92 - 96, entonces 2 + 2 =
4.
Resp: a) ; b) ; c) ; d) ; e) ; f) ; g) ; h) .
22. Supóngase que sabemos que P → Q es falso. Proporcione los valores de verdad para:
a) P ∧ Q; b) P ∨ Q; c) Q → P; d) P → Q; e) ¬P → ¬Q;
f) ¬Q → ¬P; g) Q ∧ ¬P; h) P ∧ ¬Q; i) P ∨ Q; j) ¬(P ↔ Q).
Resp: a) ; b) ; c) ; d) ; e) ; f) ; g) ; h) ; i) ; j) .
23. Un lógico le dijo a su hijo Si no terminas tu cena, te irás directo a dormir y no verás
televisión. Terminó su cena y fue enviado directamente a la cama. Discútalo.
Resp: .
24. A la pregunta de cuál de tres estudiantes estudiaba lógica fue obtenida una respuesta
correcta: si la estudiaba el primero, también lo hacía el tercero, pero no era cierto que si la
estudiaba el segundo lo hacía asímismo el tercero. ¿Quién estudiaba lógica?
Resp: .
25. Luis, Carlos, Joe, Fred ocuparon en la olimpiada de matemáticas los cuatro primeros pues-
tos. Cuando les preguntaron acerca de la distribución de los puestos, dieron las tres siguientes
respuestas:
a) Fred - primero, Carlos - segundo; b) Fred - segundo, Luis - tercero;
c) Joe - segundo, Luis - cuarto.
¿Cómo se distribuyeron los puestos si en cada una de las respuestas sólo una de las afirma-
ciones era verdadera?
Resp: a) ; b) ; c) .
26. Determine cuál de cuatro estudiantes dio el examen si sabemos que:
a) Si lo dio el primero, el segundo también;
b) Si lo dio el segundo, el tercero también o bien el primero no lo dio;
c) Si no lo dio el cuarto, lo dio el primero, pero el tercero no;
d) Si el cuarto lo dio, el primero también.
Resp: a) ; b) ; c) ; d) .
27. Para una expedición de ocho pretendientes A, B, C, D, E, F, G, H hay que elegir seis
especialistas: biólogo, hidrólogo, sinóptico, radista, mecánico y médico. Las funciones del
biólogo pueden ser realizadas por E y G, las del hidrólogo, B y F. Las del sinóptico, F y G,
las del radista, C y D, las del mecánico, C y H, las del médico, A y D. Aunque algunos de los
pretendientes tienen dos especialidades, en la expedición cada uno puede realizar sólo una
función. ¿Quién y en calidad de qué ha de incluirse en la expedición si F no puede ir sin B,
D sin H y sin C, C no puede ir simultáneamente con G, y A no puede ir junto con B?
Resp: .
CAPÍTULO 1. LÓGICA MATEMÁTICA 14
1.2. Construcción de tablas de verdad
El enunciado G ∼= P → [(Q ∧ R) → Q] incluye tres proposiciones: P, Q y R, cada una puede
ser verdadera o falsa de manera independiente. Existen en total 23
= 8 combinaciones posibles de
los valores de verdad para P, Q y R y la tabla de verdad para G deberá dar el valor de verdad de
G para cada uno de los casos.
Definición 1.14 Combinaciones
Si una proposición compuesta G consta de n enunciados, habrá 2n
combinaciones de valores de
verdad, es decir, n filas en la tabla de verdad de G.
Una tabla que despliega todos los valores de verdad de una fórmula, para todas las posibles
interpretaciones que pueda tener, se denomina tabla de verdad de la fórmula. Esta tabla puede
construirse sistemáticamente de la siguiente manera:
1. Las primeras n columnas se encabezan con las variables proposicionales; y se construyen más
columnas para las combinaciones parciales de enunciados y se culmina con el enunciado dado.
2. Bajo cada una de las primeras n columnas, se enlistan las 2n
n-adas posibles de los valores de
verdad de los componentes del enunciado G. Cada n-tupla se enlista en una fila separada.
3. Para cada fila se calculan sucesivamente los valores de verdad restantes.
Ejemplo 1.21 Sea G ∼= (P → Q) → (¬P ∨ Q), construir la correspondiente tabla de verdad:
P Q P → Q ¬ P ∨ Q G
V V V V V
V F F V F
F V V V V
F F V V F
Ejemplo 1.22 Sea G ∼= [(P → Q) ∧ ¬Q] → ¬P), construir la correspondiente tabla de verdad:
P Q P → Q (P → Q) ∧¬ Q G
V V V F V
V F F F V
F V V V V
F F F F V
Ejemplo 1.23 Sea G ∼= [(P ∨ Q) ∧ ¬P] → Q), construir la correspondiente tabla de verdad:
P Q P ∨ Q (P ∨ Q) ∧¬ P G
V V V F V
V F V F V
F V V V V
F F F F V
Ejemplo 1.24 Sea G ∼= (P → Q) ↔ (¬Q → ¬P), construir la correspondiente tabla de verdad:
P Q P → Q ¬ Q → ¬ P G
V V V V V
V F F F V
F V V V V
F F V V V
CAPÍTULO 1. LÓGICA MATEMÁTICA 15
Ejemplo 1.25 Sea G ∼= (P ↔ Q) ↔ [(P → Q) ∧ (Q → P)], construir la correspondiente tabla
de verdad:
P Q P ↔ Q P → Q Q → P (P → Q) ∧ (Q → P) G
V V V V V V V
V F F F V F V
F V F V F F V
F F V V V V V
1.2.1. Operaciones con fórmulas lógicas y sus propiedades
En el estudio de las funciones proposicionales hemos utilizado las variables P, Q, R, ... para
designar las proposiciones. Estas variables podemos interpretarlas con elementos de un dominio
básico, es decir, con proposiciones. Su dominio está formado solamente por dos elementos, los
valores de verdad V y F. Las constantes en este caso las constituyen los conectores lógicos. Mediante
el enlace lineal de las variables con valores de verdad P, Q, etc., y conectores, así como mediante
la aplicación de los signos técnicos (paréntesis), podemos formar series de signos.
Definición 1.15 Fórmula bien formada
Una fórmula bien formada, se define dentro de la lógica proposicional en los siguientes términos
recursivos:
1) Las variables P, Q, ... son fórmulas.
2) a) Si P es una fórmula, entonces ¬P también es una fórmula.
b) Si P y Q son fórmulas entonces P ∨ Q, P ∧ Q, P → Q, P ↔ Q también son fórmulas.
3) Una serie de signos P, Q, ... es una fórmula solo cuando se trata de los casos 1 y 2.
En la representación simbólica se interpretan los signos ¬, ∧, ∨, →, ↔, que reciben el nom-
bre de conectores, como signos de funciones proposicionales y también como signos de funciones
veritativas. A los literales tales como P, Q, R,... que son usados para denotar proposiciones se
denominan fórmulas atómicas o átomos. No es difícil reconocer que expresiones como P →, P ∨ no
son fórmulas. Cuando no exista confusión se suprimen los paréntesis asignando rangos decrecientes
a los conectores proposicionales de la siguiente manera; ↔, →, ∧, ∨, ¬ de manera que al conector
proposicional con mas alto rango se lo evalue al final.
Ejemplo 1.26 1) P → Q ∧ R = P → (Q ∧ R);
2) P → Q ∧ ¬R ∨ S = P → Q ∧ (¬R ∨ S) = P → [Q ∧ (¬R ∨ S)].
Ahora vamos a establecer una relación entre los valores de verdad y las funciones veritativas
por una parte y las expresiones, por otra. Las variables P, Q, ... las utilizamos ahora como variables
del valor de verdad, y de igual forma los conectores proposicionales ¬, ∧, ∨, →, ↔ como signos de
las funciones veritativas clásicas.
Sobre la base de las afirmaciones hechas podemos indicar el correspondiente valor de verdad
para cada interpretación de las variables P, Q, ... con los valores de verdad. En las expresiones
complicadas de la lógica proposicional también es posible calcular de esta forma, en finitos pasos,
los valores de verdad, al hacer las diferentes interpretaciones de las variables.
Comparando las tablas de verdad podemos decidir si dos fórmulas G y H tienen la misma tabla
de valores de verdad. Con esto también podemos mostrar si una fórmula formada a partir de G
y H, es una identidad de la lógica proposicional. La igualdad de las tablas de valores de verdad
y la identidad de la lógica proposicional, sin embargo, no son exactamente lo mismo. La igualdad
CAPÍTULO 1. LÓGICA MATEMÁTICA 16
de la tabla de valores de verdad es una relación entre dos fórmulas; y la propiedad de ser una
identidad es una peculiaridad de una fórmula. Cuando nos interesamos por la igualdad de la tabla
de valores de verdad, entonces comparamos los valores de verdad de dos fórmulas en todas las
sustituciones posibles. Cuando nos interesamos por la validez general de una fórmula, queremos
establecer si esta determinada fórmula toma, en cada interpretación, el valor de verdad V. En este
caso, se determina el valor de verdad de una nueva fórmula formada a partir de las fórmulas G y
H en todas las sustituciones posibles. De las fórmulas con las mismas tablas de verdad, G y H, se
pueden formar siempre identidades de la lógica proposicional, es decir, fórmulas de validez general.
Teorema 1.3 Una fórmula doblemente negada tiene la misma tabla de valores de verdad que la
correspondiente fórmula dada, es decir; ¬¬ P ∼= P es una identidad de la lógica proposicional.
Demostración
P ¬ P ¬¬ P
V F V
F V F
Teorema 1.4 Para la conjunción, la disjunción y la equivalencia se cumplen la ley conmutativa
y la ley asociativa con respecto a la igualdad de las tablas de valores de verdad. Para la implicación
no se cumple ni la ley asociativa, ni la ley conmutativa.
Demostración
P Q P ∨ Q Q ∨ P P ∧ Q Q ∧ P P ↔ Q Q ↔ P
V V V V V V V V
V F V V F F F F
F V V V F F F F
F F F F F F V V
Dado que G1 = (P ↔ Q) ↔ R y G2 = P ↔ (Q ↔ R), entonces
P Q R (P ∨ Q) ∨ R P ∨ (Q ∨ R) (P ∧ Q) ∧ R P ∧ (Q ∧ R) G1 G2
V V V V V V V V V
V V F V V F F F F
V F V V V F F F F
V F F V V F F V V
F V V V V F F F F
F V F V V F F V V
F F V V V F F V V
F F F F F F F F F
En lógica las proposiciones idénticamente verdaderas o bien idénticamente falsas desempeñan
importante papel. Las proposiciones idénticamente verdaderas son siempre verdaderas indepen-
diente de si las proposiciones que las forman son verdaderas o falsas.
Teorema 1.5 Para las proposiciones idénticamente verdaderas e idénticamente falsas, con todo
P son ciertas las siguientes fórmulas:
P ∨ ¬P ∼= V; P ∨ V ∼= V; P ∨ F ∼= P
P ∧ ¬P ∼= F; P ∧ V ∼= P; P ∧ F ∼= F
Demostración
CAPÍTULO 1. LÓGICA MATEMÁTICA 17
P P ∨ ¬ Q P ∨ V P ∨ F P ∧ ¬ P P ∧ V P ∧ F
V V V V F V F
F V V F F F F
Teorema 1.6 Las equivalencias siguientes
P → Q ∼= ¬Q → ¬P; ¬P → Q ∼= ¬Q → P
P → ¬Q ∼= Q → ¬P; ¬P → ¬Q ∼= Q → P
son identidades de la lógica proposicional.
Demostración
P Q P → Q ¬ Q → ¬ P ¬ P → Q ¬ Q → P
V V V V V V
V F F F V V
F V V V V V
F F V V F F
P → ¬ Q Q → ¬ P ¬ P → ¬ Q Q → P
F F V V
V V V V
V V F F
V V V V
Teorema 1.7 Las equivalencias siguientes
¬(P ∨ Q) ∼= ¬P ∧ ¬Q; ¬(P ∧ Q) ∼= ¬P ∨ ¬Q
(P ∨ Q) ∧ P ∼= P; (P ∧ Q) ∨ P ∼= P
(P ∨ Q) ∧ Q ∼= Q; (P ∧ Q) ∨ Q ∼= Q
P → Q ∼= ¬P ∨ Q; P ↔ Q ∼= (P → Q) ∧ (Q → P)
son identidades de la lógica proposicional.
Demostración
P Q ¬(P ∨ Q) ¬ P ∧ ¬ Q ¬(P ∧ Q) ¬ P ∨¬ Q (P ∨ Q) ∧ P (P ∧ Q) ∨ P
V V F F F F V V
V F F F V V V V
F V F F V V F F
F F V V V V F F
(P ∨ Q)∧ Q (P ∧ Q) ∨ Q P → Q ¬ P ∨ Q P ↔ Q (P → Q) ∧ (Q → P)
V V V V V V
F F F F F F
V V V V F F
F F V V V V
Teorema 1.8 La conjunción es, con respecto a la disjunción en ambos lados, distributiva y
viceversa, es decir, que las siguientes fórmulas son identidades de la lógica proposicional
P ∧ (Q ∨ R) ∼= (P ∧ Q) ∨ (P ∧ R); (Q ∨ R) ∧ P ∼= (Q ∧ P) ∨ (R ∧ P)
P ∨ (Q ∧ R) ∼= (P ∨ Q) ∧ (P ∨ R); (Q ∧ R) ∨ P ∼= (Q ∨ P) ∧ (R ∨ P)
Demostración
CAPÍTULO 1. LÓGICA MATEMÁTICA 18
P Q R P ∧ (Q ∨ R) (P ∧ Q) ∨ (P ∧ R) (Q ∨ R) ∧ P (Q ∧ P) ∨ (R ∧ P)
V V V V V V V
V V F V V V V
V F V V V V V
V F F F F F F
F V V F F F F
F V F F F F F
F F V F F F F
F F F F F F F
P ∨ (Q ∧ R) (P ∨ Q) ∧ (P ∨ R) (Q ∧ R) ∨ P (Q ∨ P) ∧ (R ∨ P)
V V V V
V V V V
V V V V
V V V V
V V V V
F F F F
F F F F
F F F F
Teorema 1.9 Conjuntamente con la distributividad se cumple que la implicación, con respecto
a las demás funciones veritativas, es distributiva a la derecha, pero no distributiva a la izquierda,
es decir, que las siguientes fórmulas son de validez general
P → (Q ∧ R) ∼= (P → Q) ∧ (P → R); P → (Q ∨ R) ∼= (P → Q) ∨ (P → R)
P → (Q → R) ∼= (P → Q) → (P → R); P → (Q ↔ R) ∼= (P → Q) ↔ (P → R)
Demostración
P Q R P → (Q ∧ R) (P → Q) ∧ (P → R) P → (Q ∨ R) (P → Q) ∨ (P → R)
V V V V V V V
V V F F F V V
V F V F F V V
V F F F F F F
F V V V V V V
F V F V V V V
F F V V V V V
F F F V V V V
P → (Q → R) (P → Q) → (P → R) P → (Q ↔ R) (P → Q) ↔ (P → R)
V V V V
F F F F
V V F F
V V V V
V V V V
V V V V
V V V V
V V V V
CAPÍTULO 1. LÓGICA MATEMÁTICA 19
Teorema 1.10 Si la conclusión, segundo miembro, de una implicación es igualmente una impli-
cación, entonces las dos premisas (primeros miembros), se pueden unir formando una sola premisa
P → (Q → R) ∼= (P ∧ Q) → R; (P ∧ Q) ↔ R ∼= (P → R) ∨ (Q → R)
Demostración
P Q R P → (Q → R) (P ∧ Q) → R (P ∧ Q) ↔ R (P → R) ∨ (Q → R)
V V V V V V V
V V F F F F F
V F V V V V V
V F F V V V V
F V V V V V V
F V F V V V V
F F V V V V V
F F F V V V V
Ejemplo 1.27 Utilizando las leyes de la lógica proposicional, demostrar que:
(P ∧ Q) ↔ (P ∨ Q) ∼= (P ∨ Q) → (P ∧ Q).
Solución
(P ∧ Q) ↔ (P ∨ Q) ∼= [(P ∧ Q) → (P ∨ Q)] ∧ [(P ∨ Q) → (P ∧ Q)]
∼= [¬(P ∧ Q) ∨ (P ∨ Q)] ∧ [(P ∨ Q)) → (P ∧ Q)]
∼= [(¬P ∨ ¬Q) ∨ (P ∨ Q)] ∧ [(P ∨ Q) → (P ∧ Q)]
∼= (¬P ∨ ¬Q ∨ P ∨ Q) ∧ [(P ∨ Q) → (P ∧ Q)]
∼= [(¬P ∨ P) ∨ (¬Q ∨ Q)] ∧ [(P ∨ Q) → (P ∧ Q)]
∼= (V ∨ V) ∧ [(P ∨ Q) → (P ∧ Q)]
∼= V ∧ [(P ∨ Q) → (P ∧ Q)]
∼= (P ∨ Q) → (P ∧ Q).
Ejemplo 1.28 Utilizando las leyes de la lógica proposicional, demostrar que:
[(P → Q) ∧ ¬P] → ¬Q ∼= Q → P.
Solución
[(P → Q) ∧ ¬P] → ¬Q ∼= ¬[(P → Q) ∧ ¬P] ∨ ¬Q
∼= ¬[(¬P ∨ Q) ∧ ¬P] ∨ ¬Q
∼= ¬(¬P) ∨ ¬Q
∼= P ∨ ¬Q
∼= Q → P.
Ejemplo 1.29 Utilizando las leyes de la lógica proposicional, demostrar que:
[(P → Q) ∧ (P → R)] → (Q → R) ∼= Q → (P ∨ R).
CAPÍTULO 1. LÓGICA MATEMÁTICA 20
Solución
[(P → Q) ∧ (P → R)] → (Q → R) ∼= ¬[(¬P ∨ Q) ∧ (¬P ∨ R)] ∨ (¬Q ∨ R)
∼= ¬(¬P ∨ Q) ∨ ¬(¬P ∨ R) ∨ (¬Q ∨ R)
∼= (P ∧ ¬Q) ∨ (P ∧ ¬R) ∨ ¬Q ∨ R
∼= ¬Q ∨ R ∨ (P ∧ ¬R)
∼= ¬Q ∨ [(R ∨ P) ∧ (R ∨ ¬R)]
∼= ¬Q ∨ [(R ∨ P) ∧ V]
∼= ¬Q ∨ (R ∨ P)
∼= Q → (P ∨ R).
Ejemplo 1.30 Utilizando las leyes de la lógica proposicional, demostrar que:
[(P → Q) → R] → [(Q → P) → R] ∼= (P ∧ ¬Q) → R.
Solución
[(P → Q) → R] → [(Q → P) → R] ∼= ¬[¬(¬P ∨ Q) ∨ R] ∨ [¬(¬Q ∨ P) ∨ R]
∼= ¬[(P ∧ ¬Q) ∨ R] ∨ [(Q ∧ ¬P) ∨ R]
∼= [¬(P ∧ ¬Q) ∧ ¬R] ∨ [(Q ∧ ¬P) ∨ R]
∼= [(¬P ∨ Q) ∧ ¬R] ∨ [(Q ∧ ¬P) ∨ R]
∼= [(¬P ∨ Q) ∧ (Q ∧ ¬P) ∨ R] ∧ [¬R ∨ (Q ∧ ¬P) ∨ R]
∼= [(¬P ∨ Q ∨ Q) ∧ (¬P ∨ Q ∨ ¬P)] ∨ R ∧ V
∼= [(¬P ∨ Q) ∧ (¬P ∨ Q)] ∨ R
∼= (¬P ∨ Q) ∨ R
∼= ¬(P ∧ ¬Q) ∨ R
∼= (P ∧ ¬Q) → R.
Ejemplo 1.31 Utilizando las leyes de la lógica proposicional, demostrar que:
[(P → Q) → P] → (P → Q) ∼= P → Q.
Solución
[(P → Q) → P] → (P → Q) ∼= ¬[¬(¬P ∨ Q) ∨ P] ∨ (¬P ∨ Q)
∼= ¬[(P ∧ ¬Q) ∨ P] ∨ (¬P ∨ Q)
∼= ¬P ∨ ¬P ∨ Q
∼= ¬P ∨ Q
∼= P → Q.
1.2.2. Tautologías y falacias
Definición 1.16 Tautología
Si una proposición compuesta es siempre verdadera bajo todas sus interpretaciones, independien-
temente de los valores de verificación de sus componentes, decimos que la proposición compuesta
es una tautología.
CAPÍTULO 1. LÓGICA MATEMÁTICA 21
Es decir, a un enunciado que es verdadero para todos los valores posibles de sus variables
proposicionales se le denomina tautología. Cuando se comprueba que una equivalencia es una
tautología, significa que sus dos partes componentes son siempre o ambas verdaderas o ambas
falsas, para cualesquier valores de las variables proposicionales. Por tanto los dos lados son sólo
diferentes maneras de proponer el mismo enunciado y se dice que son logicamente equivalentes.
Definición 1.17 Falacia
Una fórmula G es una falacia, si ¬G es una tautología.
Ejemplo 1.32 Utilizando una tabla de verdad, determinar si la fórmula
G = (P → Q) → (¬P ∨ Q)
es tautología.
Solución
P Q P → Q ¬ P ∨ Q (P → Q) → (¬ P ∨ Q)
V V V V V
V F F F V
F V V V V
F F V V V
Por lo tanto G si es tautología.
Ejemplo 1.33 Utilizando una tabla de verdad, determinar si la fórmula
G = (Q → P) → (P → Q)
es tautología.
Solución
P Q Q → P P → Q (Q → P) → (P → Q)
V V V V V
V F V F F
F V F V V
F F V V V
Por lo tanto G no es tautología.
Ejemplo 1.34 Utilizando una tabla de verdad, determinar si la fórmula
G = (P → Q) ↔ (¬Q → ¬P)
es tautología.
Solución
P Q P → Q ¬ Q → ¬ Q (P → Q) ↔ (¬ Q → ¬ P)
V V V V V
V F F F V
F V V V V
F F V V V
Por lo tanto G si es tautología.
CAPÍTULO 1. LÓGICA MATEMÁTICA 22
Ejemplo 1.35 Utilizando una tabla de verdad, determinar si la fórmula
G = (P ↔ Q) ↔ [(P → Q) ∧ (Q → P)]
es tautología.
Solución
P Q P ↔ Q (P → Q) ∧ (Q → P (P ↔ Q) ↔ [(P → Q) ∧ (Q → P)]
V V V V V
V F F F V
F V F F V
F F V V V
Por lo tanto G si es tautología.
Ejemplo 1.36 Utilizando una tabla de verdad, determinar si la fórmula
G = [(P → Q) ∧ (Q → R)] → (P → R)
es tautología.
Solución
P Q R (P → Q) ∧ (Q → P P → Q [(P → Q) ∧ (Q → R)] → (P → R)
V V V V V V
V V F F F V
V F V F V V
V F F V F V
F V V V V V
F V F F V V
F F V V V V
F F F V V V
Por lo tanto G si es tautología.
Ejemplo 1.37 Utilizando una tabla de verdad, determinar si la fórmula
G = [P → (Q → R)] ↔ [(P → Q) → R]
es tautología.
Solución
P Q R (P → (Q → R) (P → Q) → R [P → (Q → R)] ↔ [(P → Q) → R]
V V V V V V
V V F F F V
V F V V V V
V F F V V V
F V V V V V
F V F V F F
F F V V V V
F F F V F F
Por lo tanto G no es tautología.
CAPÍTULO 1. LÓGICA MATEMÁTICA 23
1.2.3. Tarea
1. Construya la tabla de verdad para cada una de las siguientes proposiciones:
a) (P → Q) → [(P ∨ ¬ Q) → (P ∧ Q)]; b) [(P ∨ Q) ∧ R] → (P ∧ ¬Q);
c) [(P ↔ Q) ∨ (P → R)] → (¬Q ∧ P)]; d) P ∨ P; e) (P ∨ Q) ∨ R;
f) (P ∨ P) ∨ P; g) P ∨ Q; h) ¬(P ↔ Q).
Resp: a) ; b) ; c) ; d) ; e) ; f) ; g) ; h) .
2. Utilizando las leyes de la lógica proposicional, demostrar que
P ∨ Q ∼= (P ∨ Q) ∧ ¬(P ∧ Q)
Resp: .
3. Utilizando las leyes de la lógica proposicional, demuestre o refute:
a) P ∨ Q ∼= (P ∨ Q) ∧ ¬(P ∧ Q); b) P ∨ (Q → R) ∼= (P ∨ Q) → (P ∨ R);
c) (P ∨ Q) ∨ R ∼= P ∨ (Q ∨ R).
Resp: a) ; b) ; c) .
4. Simplifique las siguientes fórmulas y diga cuales son tautologías y cuales falacias:
a) P ∨ (¬P ∧ ¬Q)] ∨ (P ∧ ¬Q); b) (P ∧ Q) ∨ [(R ∨ P) ∧ ¬Q].
Resp: a) ; b) .
5. Simplifique las siguientes fórmulas y diga cuales son tautologías y cuales falacias:
a) (R ∧ Q) ∨ (P ∧ ¬Q ∧ R) ∨ (¬P ∧ ¬Q ∧ R); b) (P → Q) ∧ ¬(R → Q);
c) (¬P ∨ Q) ∧ ¬Q ∧ [¬(R ∧ Q) → P].
Resp: a) ; b) ; c) .
6. Simplifique las siguientes fórmulas y diga cuales son tautologías y cuales falacias:
a) (P ∧ Q) ∧ (R ∨ ¬S) ∧ (P → S); b) (¬P ∨ Q) ∧ (¬P → R) ∧ ¬R;
c) (P ∧ Q) ∧ (P → R) ∧ (Q → S).
Resp: a) ; b) ; c) .
7. Simplifique las siguientes fórmulas y diga cuales son tautologías y cuales falacias:
a) (P ∨ Q) ∧ ¬Q ∧ (P → R); b) (P ∧ Q) ∧ (P → ¬R) ∧ (Q → ¬R);
c) (P → ¬Q) ∧ Q ∧ (¬P → (R ∨ S)].
Resp: a) ; b) ; c) .
8. Simplifique las siguientes fórmulas y diga cuales son tautologías y cuales falacias:
a) (P → S) ∧ (P ∧ Q) ∧ [(S ∧ R) → ¬T] ∧ (Q → R);
b) ¬P ∧ (Q → P) ∧ [(¬Q ∨ R) → S];
c) (P ∧ ¬Q) ∧ (R → Q) ∧ (R ∨ S) ∧ [(S ∨ P) → T].
Resp: a) ; b) ; c) .
9. Simplifique las siguientes fórmulas y diga cuales son tautologías y cuales falacias:
a) (P ↔ Q) ↔ [(P → Q) ∧ (Q → P)]; b) [(P → Q) ∧ (Q → R) ∧ P] → R;
c) [P → (Q ∨ R)] ∧ (Q → ¬R) ∧ [(S → ¬R) ∧ P] → ¬S.
Resp: a) ; b) ; c) .
1.3. Transformación de fórmulas
La igualdad de los valores de verdad de dos proposiciones la hemos demostrado hasta ahora
utilizando las tablas completas de valores de verdad. Con su ayuda pudimos decidir si una fórmula
CAPÍTULO 1. LÓGICA MATEMÁTICA 24
dada es o no una identidad de la lógica proposicional.
Por esta vía hemos conocido mumerosas fórmulas con las tablas de valores. Otras identidades,
es decir; las leyes de la lógica proposicional, las obtenemos a partir de las fórmulas dadas y me-
diante sustituciones o transformaciones en fórmulas equivalentes.
En esta sección veremos cómo obtener equivalencias e implicaciones lógicas sin utilizar tablas
de verdad. También explicaremos el significado de teorema y de demostración. Empezaremos con
dos reglas útiles, que sin embargo deben manejarse con cuidado.
Teorema 1.11 Si en una fórmula de validez general, es decir, en una identidad de la lógica
proposicional, se sustituye una variable proposicional por una fórmula cualquiera en todos los lu-
gares donde se presenta la fórmula correspondiente, entonces se obtiene nuevamente una fórmula
de validez general.
Teorema 1.12 Cuando en una fórmula G se sustituye una cierta subfórmula G1 por una fór-
mula G2, que toma los mismos valores de verdad que G1, entonces la fórmula obtenida F tiene los
mismos valores de verdad que la fórmula G. La fórmula G, una vez sustituida G1 debe sustituirse
por G2 en todos los lugares donde esta se presenta.
Ejemplo 1.38 Consideremos la proposición
G ∼= [P ∧ (P → Q)] → Q
que es una tautología. Si reemplazamos, cada vez que aparece P, por la proposición
G1
∼= Q → R
obtenemos la tautología
H ∼= [(Q → R) ∧ ((Q → R) → Q)] → Q.
Si en cambio reemplazamos Q, cada vez que aparece, por G1, obtenemos la tautología
H ∼= [P ∧ (P → (Q → R))] → (Q → R).
Ejemplo 1.39 Consideremos la proposición
G ∼= ¬[(P → Q) ∧ (P → R)] → [Q → (P → R)]
que no es una tautología. Obtenemos proposiciones lógicamente equivalentes si reemplazamos P → Q
por su equivalencia lógica ¬P ∨ Q o si reemplazamos una o las dos veces que aparece P → R por
¬P ∨ R. Podemos también reemplazar (P → Q) ∧ (P → R) por P → (Q ∧ R). De esta manera G
es lógicamente equivalente a las siguientes proposiciones entre otras:
¬[(¬P ∨ Q) ∧ (P → R)] → [Q → (P → R)]
¬[(P → Q) ∧ (¬P ∨ R)] → [Q → (P → R)]
¬[(P → (Q ∧ R)] → [Q → (¬P ∨ R)].
Definición 1.18 Fórmula válida
Una fórmula G es válida o constituye una tautología, si y sólo si es verdadera bajo todas las
interpretaciones. En caso contrario la fórmula G es inválida.
CAPÍTULO 1. LÓGICA MATEMÁTICA 25
Definición 1.19 Fórmula inconsistente
Una fórmula G se denomina inconsistente o insatisfactible, si y sólo si es falsa bajo todas las
interpretaciones. En caso contrario la fórmula G es consistente o satisfactible.
De las definiciones anteriores, las observaciones siguientes son obvias:
1. Una fórmula es válida, si y sólo si su negación es inconsistente.
2. Una fórmula es inconsistente, si y sólo si su negación es válida.
3. Una fórmula es inválida, si y sólo si hay por lo menos una interpretación bajo la cual la
fórmula es falsa.
4. Una fórmula es inconsistente, si y sólo si hay por lo menos una interpretación bajo la cual
la fórmula es verdadera.
5. Si una fórmula es válida, entonces es consistente pero no viceversa.
6. Si una fórmula es inconsistente, entonces es inválida pero no viceversa.
Ejemplo 1.40 Verificar la validez o inconsistencia de la fórmula:
[(P → Q) ∧ (Q → R)] → (P → R)
Solución
P Q R (P → Q) ∧ (Q → R) P → R [(P → Q) ∧ (Q → R)] → (P → R)
V V V V V V
V V F F F V
V F V F V V
V F F V F V
F V V V V V
F V F F V V
F F V V V V
F F F V V V
Por lo tanto G es una fórmula válida.
Ejemplo 1.41 Verificar la validez o inconsistencia de la fórmula:
[(P → (Q → R)] ↔ [(P → Q) → R]
Solución
P Q R (P → (Q → R) (P → Q) → R [(P → (Q → R)] ↔ [(P → Q) → R]
V V V V V V
V V F F F V
V F V V V V
V F F V V V
F V V V V V
F V F V F F
F F V V V V
F F F V F F
Por lo tanto G no es una fórmula válida.
CAPÍTULO 1. LÓGICA MATEMÁTICA 26
1.3.1. Formas normales
En lógica matemática es muy importante el poder transformar fórmulas de una forma a otra,
especialmente a las denominadas formas normales. Para lograr estas transformaciones de fórmulas,
se utiliza el concepto de equivalencias de fórmulas.
Definición 1.20 Fórmulas equivalentes
Las fórmulas G y H son equivalentes si los valores de verdad de G y H son los mismos bajo todas
las interpretaciones de estas fórmulas.
Por supuesto que nuestro interés no se limita a estudiar una simple clasificación de los enun-
ciados del lenguaje; pero tampoco intentamos internarnos en el fascinante mundo de la deducción
lógica sin antes estar seguros de conocer y comprender algunos conceptos elementales. Las dos
formas normales que nos interesa obtener y que son utilizadas en prueba mecánica de teoremas,
son la forma normal conjuntiva y la forma normal disjuntiva.
Definición 1.21 Forma normal conjuntiva
Una fórmula G se dice que está en forma normal conjuntiva si y sólo si G tiene la forma
G ∼= G1 ∧ G2 ∧ · · · ∧ Gn n ∈ N
donde cada una de las fórmulas G1, G2, ..., Gn, se expresan como una conjunción de literales.
Ejemplo 1.42 Expresar la fórmula
G ∼= (Q → P) → (P → Q)
en forma normal conjuntiva.
Solución
(Q → P) → (P → Q) ∼= ¬(Q → P) ∨ (P → Q)
∼= ¬(¬Q ∨ P) ∨ (P → Q)
∼= (Q ∧ ¬P) ∨ (P → Q)
∼= [Q ∨ (P → Q)] ∧ [¬P ∨ (P → Q)].
Ejemplo 1.43 Expresar la fórmula
G ∼= (P ↔ Q) ↔ [(P → Q) ∧ (Q → P)]
en forma normal conjuntiva.
Solución
(P ↔ Q) ↔ [(P → Q) ∧ (Q → P)] ∼= (P ↔ Q) ↔ (P ↔ Q)
∼= [(P ↔ Q) → (P ↔ Q)] ∧ [(P ↔ Q) → (P ↔ Q)].
Definición 1.22 Forma normal disjuntiva
Una fórmula G se dice que está en forma normal disjuntiva si y sólo G si tiene la forma
G ∼= G1 ∨ G2 ∨ · · · ∨ Gn n ∈ N
donde cada una de las fórmulas G1, G2, ..., Gn, se expresan como una disjunción de literales.
CAPÍTULO 1. LÓGICA MATEMÁTICA 27
Ejemplo 1.44 Expresar la fórmula
G ∼= (Q → P) → (P → Q)
en forma normal disjuntiva.
Solución
(Q → P) → (P → Q) ∼= ¬(Q → P) ∨ (P → Q)
Ejemplo 1.45 Expresar la fórmula
G ∼= (P ↔ Q) ↔ [(P → Q) ∧ (Q → P)]
en forma normal conjuntiva.
Solución
(P ↔ Q) ↔ [(P → Q) ∧ (Q → P)] ∼= (P ↔ Q) ↔ (P ↔ Q)
∼= [(P ↔ Q) → (P ↔ Q)] ∧ [(P ↔ Q) → (P ↔ Q)]
∼= (P ↔ Q) → (P ↔ Q)
∼= ¬(P ↔ Q) ∨ (P ↔ Q)
Un hecho que es muy importante anotar, es que cualquier fórmula de la lógica proposicional
puede ser transformada a una de las formas normales, utilizando las leyes de la lógica proposicional.
1.3.2. Consecuencias lógicas
Definición 1.23 Consecuencia lógica
Dadas las fórmulas G1, G2, ..., Gn y una fórmula G, G se denomina consecuencia lógica de G1,
G2, ..., Gn si y sólo si para cualquier interpretación en la cual G1 ∧ G2 ∧ · · · ∧ Gn es verdad, G
también lo es G1, G2, ..., Gn se denominan axiomas de G.
Teorema 1.13 Dadas las fórmulas G1, G2, ..., Gn y una fórmula G, G es una consecuencia
lógica de G1, G2, ..., Gn si y sólo si la fórmula (G1 ∧ G2 ∧ · · · ∧ Gn) → G es válida.
Demostración
⇒ Suponga que G es una consecuencia lógica de G1, G2, ..., Gn. Sea I una interpretación ar-
bitraria. Si G1, G2, ..., Gn son verdaderos en I, entonces por definición de consecuencia lógica
G es verdadero en I. Entonces (G1 ∧ G2 ∧ · · · ∧ Gn) → G es verdadero en I. Por otra parte, si
G1, G2, ..., Gn son falsos en I, entonces (G1 ∧ G2 ∧ · · · ∧ Gn) → G es verdadero en I. Así, de-
mostramos que (G1 ∧ G2 ∧ · · · ∧ Gn) → G es verdadero bajo cualquier interpretación. Esto es,
(G1 ∧ G2 ∧ · · · ∧ Gn) → G es una fórmula válida.
⇐ Supongamos que (G1 ∧ G2 ∧ · · · ∧ Gn) → G es una fórmula válida. Para cualquier interpreta-
ción I, si G1 ∧ G2 ∧ · · · ∧ Gn es verdadero en I, G debe ser verdadero en I. Por consiguiente G es
una consecuencia lógica de G1, G2, ..., Gn.
Ejemplo 1.46 Sean
G1 P ∨ (¬Q → R)
G2 ¬(P ∨ S) ∧ ¬R
G Q
Pruebe si G es consecuencia lógica de G1 y G2.
Solución
CAPÍTULO 1. LÓGICA MATEMÁTICA 28
Debemos probar que la fórmula {[P ∨ (¬Q → R)] ∧ [¬(P ∨ S) ∧ ¬R]} → Q, es verdadera o falsa,
decir:
{[P ∨ (¬Q → R)] ∧ [¬(P ∨ S) ∧ ¬R]} → Q ∼= ¬{[P ∨ (Q ∨ R)] ∧ [¬P ∨ ¬S ∧ ¬R]} ∨ Q
∼= ¬[(P ∨ Q ∨ R) ∧ (¬P ∨ ¬S ∧ ¬R)] ∨ Q
∼= (¬P ∧ ¬Q ∧ ¬R) ∨ (P ∨ S ∨ R) ∨ Q
∼= ¬(P ∨ Q ∨ R) ∨ (P ∨ Q ∨ R) ∨ S
∼= V.
Lo cual indica que G es consecuencia lógica de G1 y G2.
Ejemplo 1.47 Sean
G1 P → Q)
G2 Q → R
G3 ¬R
G ¬R
Pruebe si G es consecuencia lógica de G1, G2 y G3.
Solución
Debemos probar que la fórmula [(P → Q) ∧ (Q → R) ∧ ¬R] → ¬R, es verdadera o falsa, decir:
[(P → Q) ∧ (Q → R) ∧ ¬R] → ¬R ∼= ¬[(¬P ∨ Q) ∧ (¬Q ∨ R) ∧ ¬R] ∨ ¬R
∼= ¬(¬P ∨ Q) ∧ ¬(¬Q ∨ R) ∨ (R ∨ ¬R)
∼= V.
Lo cual indica que G es consecuencia lógica de G1, G2 y G3.
Teorema 1.14 Dadas las fórmulas G1, G2, ..., Gn y una fórmula G, G es una consecuencia
lógica de G1, G2, ..., Gn si y sólo si la fórmula G1 ∧ G2 ∧ · · · ∧ Gn ∧ G es inconsistente.
Demostración
Por el teorema anterior, G es una consecuencia lógica de G1, G2, ..., Gn si y sólo si la fórmula
(G1 ∧ G2 ∧ · · · ∧ Gn) → G es válida. Así, G es una consecuencia lógica de G1, G2, ..., Gn si y sólo
si la negación de (G1 ∧ G2 ∧ · · · ∧ Gn) → G es inconsistente
¬[(G1 ∧ G2 ∧ · · · ∧ Gn) → G] ∼= ¬[¬(G1 ∧ G2 ∧ · · · ∧ Gn) ∨ G]
∼= ¬¬(G1 ∧ G2 ∧ · · · ∧ Gn) ∧ ¬G
∼= (G1 ∧ G2 ∧ · · · ∧ Gn) ∧ ¬G
∼= G1 ∧ G2 ∧ · · · ∧ Gn ∧ ¬G
Por lo tanto, concluimos que el teorema es verdadero.
Ejemplo 1.48 Sean
G1 P ∨ (¬Q → R)
G2 ¬(P ∨ S) ∧ ¬R
G Q
Pruebe si G es consecuencia lógica de G1 y G2.
Solución
CAPÍTULO 1. LÓGICA MATEMÁTICA 29
Debemos probar que la fórmula {[P ∨ (¬Q → R)] ∧ [¬(P ∨ S) ∧ ¬R]} ∧ ¬Q, es verdadera o falsa,
decir:
{[P ∨ (¬Q → R)] ∧ [¬(P ∨ S) ∧ ¬R]} ∧ ¬Q ∼= [(P ∨ Q ∨ R) ∧ (¬P ∧ ¬S ∧ ¬R)] ∧ ¬Q
∼= (P ∨ Q ∨ R) ∧ (¬P ∧ ¬Q ∧ ¬R) ∧ ¬S
∼= (P ∨ Q ∨ R) ∧ ¬(P ∨ Q ∨ R) ∧ ¬S
∼= F.
Lo cual indica que G es consecuencia lógica de G1 y G2.
Ejemplo 1.49 Sean
G1 P → Q)
G2 Q → R
G2 ¬R
G ¬R
Pruebe si G es consecuencia lógica de G1, G2 y G3.
Solución
Debemos probar que la fórmula [P → Q) ∧ (Q → R) ∧ ¬R] ∧ R, es verdadera o falsa, decir:
[P → Q) ∧ (Q → R) ∧ ¬R] ∧ R ∼= (P → Q) ∧ (Q → R) ∧ (¬R ∧ R)
∼= F.
Lo cual indica que G es consecuencia lógica de G1, G2 y G3.
1.3.3. Tarea
1. Determine la validez o inconsistencia, luego transforme a una de sus formas normales las
siguientes fórmulas::
a) [P ∨ (¬P ∧ ¬Q)] ∨ (P ∧ ¬Q);
b) (P ∧ Q) ∨ [(R ∨ P) ∧ ¬Q];
c) (R ∧ Q) ∨ (P ∧ ¬Q ∧ R) ∨ (¬P ∧ ¬Q ∧ R);
d) (¬P ∨ Q) ∧ ¬Q ∧ [¬(R ∧ Q) → P].
Resp: a) ; b) ; c) ; d) .
2. Determine la validez o inconsistencia, luego transforme a una de sus formas normales las
siguientes fórmulas:
a) (P ∧ Q) ∧ (R ∨ ¬S) ∧ (P → S);
b) (¬P ∨ Q) ∧ (¬P → R) ∧ ¬R;
c) (P ∧ Q) ∧ (P → R) ∧ (Q → S);
d) (P ∨ Q) ∧ ¬Q ∧ (P → R);
e) (P ∧ Q) ∧ (P → ¬R) ∧ (Q → ¬R);
f) [(P → Q) ∧ (Q → R) ∧ P] → R.
Resp: a) ; b) ; c) ; d) ; e) ; f) .
3. Determine la validez o inconsistencia, luego transforme a una de sus formas normales las
siguientes fórmulas:
a) (P → ¬Q) ∧ Q ∧ [¬P → (R ∨ S)];
b) (P → S) ∧ (P ∧ Q) ∧ [(S ∧ R) → ¬T] ∧ (Q → R);
c) ¬P ∧ (Q → P) ∧ [(¬Q ∨ R) → S];
CAPÍTULO 1. LÓGICA MATEMÁTICA 30
d) (P ∧ ¬Q) ∧ (R → Q) ∧ (R ∨ S) ∧ [(S ∨ P) → T];
e) (P ↔ Q) ↔ [(P → Q) ∧ (Q → P)];
f) [P → (Q ∨ R)] ∧ (Q → ¬R) ∧ [(S → ¬R) ∧ P] → ¬S.
Resp: a) ; b) ; c) ; d) ; e) ; f) .
4. Decir cual de los siguientes enunciados son consecuencia lógica:
a)
G1 (P ∨ Q) → R)
G2 S ∧ T
G3 U → ¬L
G4 P ∨ U
G5 S → L
G R
b)
G1 (P → Q) → R
G2 ¬R ∨ S
G3 ¬(P ∧ ¬Q)
G4 (S ∨ T) → U
G U
c)
G1 ¬(P ∨ Q) → (R ∨ S)
G2 ¬(P ∨ Q)
G3 ¬(R ∨ S) ∨ (T ∧ U)
G T ∧ U
Resp: a) ; b) ; c) .
5. Los alumnos son estudiosos o los estudiosos reprueban. Si los estudiosos reprueban, enton-
ces los inteligentes son felices o los alumnos no son estudiosos. Los alumnos son estudiosos
y los inteligentes no son felices. No es verdad que los inteligentes son felices. Los estudiantes
no reprueban?
Resp: .
6. Juego fútbol o estudio. Si paso el examen no estudio. Sucede que no voy a jugar fútbol.
En consecuencia no pasé el examen.
Resp: .
7. La lógica es fácil. Si el álgebra es hermosa, entonces la Lógica no es fácil o la Matemática
es la reina de las ciencias. El Algebra es hermosa. En consecuencia, la Matemática es la reina
de las ciencias.
Resp: .
8. Ayer no fue miércoles o mañana no es martes. Hoy es jueves y ayer fue miércoles. Hoy es
lunes si y sólo si mañana es martes. En consecuencia, hoy es lunes.
Resp: .
CAPÍTULO 1. LÓGICA MATEMÁTICA 31
9. Luis hará un viaje a Europa si logra terminar su carrera. Luis termina su carrera, y si
hace un viaje a Europa, entonces no asiste a nuestra reunión anual. En consecuencia, Luis
no asistirá a nuestra reunión anual.
Resp: .
10. Si faltan ejercicios o encuentro premisas, entonces acabo la tarea. Si el libro está claro y no
me falta creatividad, entonces encuentro premisas. No acabo la tarea. En consecuencia me
falta creatividad o el libro no está claro.
Resp: .
11. Si ganamos el campeonato, recibimos el premio. Si jugamos y ganamos el campeonato,
recibiremos el premio. Jugaremos y ganaremos el campeonato. En consecuencia, recibiremos
el premio.
Resp: .
12. Repruebo el examen o sigo mis estudios. Si repruebo el examen, perderé la beca y me iré
de la ciudad. No perderé la beca o no me iré de la ciudad. Luego, seguire estudiando.
Resp: .
13. Los aviones son veloces o las diligencias respetan los semáforos. Si los hombres vuelan y las
bicicletas no contaminan, entonces no es verdad que las diligencias respetan los semáforos.
Los hombres vuelas y las bicicletas no contaminan. En conclusión, los aviones son veloces.
Resp: .
1.4. Expresiones de la lógica de predicados
El cálculo proposicional es una teoría de la lógica, completa y autónoma, pero totalmente inade-
cuada para la mayor parte de las matemáticas. El problema reside en que el cálculo proposicional
no permite el uso de un número infinito de proposiciones. Además, la notación es difícil para ma-
nejar un gran número finito de proposiciones.
Por ejemplo, con frecuencia encontramos una sucesión infinita de proposiciones P(x) con índi-
ces en N. La afirmación informal ¨P(x) es verdadera para toda x¨ significa ¨P(0) es verdadera,
P(1) es verdadera, P(2) es verdadera, etc.¨ El único simbolismo que podríamos utilizar, según el
cálculo proposicional sería P(0)∧P(1)∧P(2)∧..., pero no es aceptable en el cálculo proposicional.
En forma similar, la afirmación informal ¨P(x) es verdadera para alguna x¨ correspondería al
inaceptado P(0)∨P(1)∨P(2)∨.... Para darle la vuelta a este problema, necesitamos dos símbolos
nuevos: uno que signifique ¨para todo¨ y otro que signifique ¨para algún¨.
Entonces necesitamos saber las reglas para utilizar los nuevos símbolos y combinarlos con los
viejos. Este sistema de símbolos y reglas se llama cálculo de predicados. Los nuevos símbolos que
introduciremos se llaman cuantificadores.
Supongamos que {P(x)/x ∈ U} es una familia con índices en un conjunto U que puede se
infinito; el conjunto U se llama el dominio de individuos o universo de individuos.
Mediante la introducción de ¨existe ...¨ es confirmada la existencia de por lo menos un elemento
del conjunto base que satisface la forma proposicional dada. Esta proposición es una proposición
existencial. Proposiciones con la formulación una parte, casi todo, la mayoría, algunos, etc., son
CAPÍTULO 1. LÓGICA MATEMÁTICA 32
también proposiciones existenciales. Cuando hablamos de proposiciones existenciales, nos referi-
mos también a proposiciones particulares, ya que estas no se refieren a todos los elementos del
conjunto que nos interesa, sino solo a una parte. En este caso denominamos a la cuantificación,
particularidad.
De forma análoga, se denomina a las proposiciones en que aparece la formulación ¨para todos¨,
proposiciones universales o generales, ya que estas se refieren a todos los elementos del conjunto de
variables. Tal cuantificación se denomina también generalización. La cuantificación particularidad
y generalización son operaciones de la lógica de predicados.
Partiendo de las formas proposicionales relacionadas previamente con los operadores, tales como
¨existe ...¨, ¨para todo ...¨, ¨no existe ningún ...¨, hemos obtenido proposiciones falsas o verdade-
ras. Para estos operadores denominados también cuantificadores, se han introducido en la lógica
matemática signos especiales.
El cuantificador existencial (particularizador) ¨existe (por lo menos) un ...¨ es simbolizado con
?∃. Si el símbolo ∃? se encuentra ante una forma proposicional P(x), esto quiere decir que existe
por lo menos un elemento del conjunto fundamental que posee la propiedad reflejada en la forma
proposicional P(x). Utilizamos las escrituras ¨∃ x P(x)¨.
La tachadura vertical o la relación que se establece entre el símbolo ¨∃¨ y el símbolo ¨ ∃¨, debe
expresar que no existe ningún elemento del conjunto fundamental que posea la propiedad indicada
en la forma proposicional P(x).
El cuantificador universal (operador universal, generalizador) ¨para todo ...¨ se representa con
el símbolo ¨∀¨. Si el símbolo ¨∀¨ se encuentra ante una forma proposicional P(x), esto quiere
decir que la propiedad reflejada en la forma proposicional P(x) es aplicable para cada elemento
del dominio de individuos. El cuantificador universal forma pareja con una variable, ∀ x, significa,
¨para todo x ...¨.
La tachadura vertical o la relación que se establece entre el cuantificador universal y el símbolo
¨ ∀¨ debe expresar que la propiedad reflejada en P(x) no es aplicable para todos los elementos del
dominio de individuos.
La lógica de predicados o lógica de primer grado, nos enseña que para la cuantificación sólo
son admisibles las variables de individuos. Las variables de individuos cuantificados dejan de ser
variables libres para convertirse en variables ligadas. Para crear expresiones de la lógica de predi-
cados utilizamos además de los símbolos para las variables de individuos, constantes de individuos,
variables predicativas, cuantificadores y los conectores de la lógica proposicional.
En la lógica proposicional comprobamos el valor de verdad de una expresión mediante la sus-
titución de las variables de dicha expresión por sus valores de verdad, teniendo en cuenta las
disposiciones correspondientes. El valor de verdad de una expresión de la lógica de predicados
depende no solo del cuantificador sino también de las variables de individuos y del conjunto de in-
dividuos tomado como base, así como de la sustitución o interpretación de las variables predicativas.
A la proposición compuesta ∀ x P(x) se le asignan valores de verdad de la manera siguiente:
¨∀ x P(x) es verdadero si P(x) es verdadero para toda x en U; en cualquier otro caso ∀ x P(x) es
falsa¨
CAPÍTULO 1. LÓGICA MATEMÁTICA 33
La proposición compuesta ∃ x P(x) tiene los siguientes valores de verdad:
¨∃ x P(x) es verdadero si P(x) es verdadera para al menos una x en U; ∃ x P(x) es falso si P(x)
es falsa para toda x en U¨
Analicemos la proposición ∀ x P(x) de manera más detallada. La expresión P(x) se llama
predicado. Para formar una oración hay que tener un sujeto. Por ejemplo, el predicado ¨... es mas
poblada que Quito¨ se transforma en la oración ¨Guayaquil es mas poblada que Quito¨ al dar co-
mo sujeto Guayaquil. Si llamamos P al predicado la oración podría escribirse como P(Guayaquil).
Cada sujeto da una oración.
En nuestra lógica simbólica dar un predicado es establecer una función que produce una proposi-
ción siempre que le demos un elemento del dominio de individuos, esto es, una función proposicional
- valuada con dominio de individuos U. Seguimos nuestra práctica usual y denotamos tal función
por P(x). La variable x en la expresión P(x) se llama variable libre del predicado. En tanto x
varía en U los valores de verdad de P(x) pueden variar. En contraste, la proposición ∀ x P(x)
tiene un significado fijo y un valor de verdad que no varía con x. La variable x en ∀ x P(x) se
llama variable acotada; está acotada por el cuantificador ∀. Como ∀ x P(x) tiene un significado fijo
y un valor de verdad sería inútil y poco natural cuantificarla de nuevo. Esto es, sería vano intro-
ducir ∀ x[∀ x P(x)] y ∃ x[∃ x P(x)] ya que sus valores de verdad son los mismos que los de ∀ x P(x).
Podemos también considerar predicados que son funciones de más de una variable, posiblemente
de más de un dominio de individuos, y en tales casos el uso de varios cuantificadores resulta natural.
Ejemplo 1.50 Con estos ejemplos en mente vamos a dar una descripción más detallada y
formal. Sean U1, U2, ..., Un conjuntos no vacíos. Un predicado de n argumentos sobre U1 x U2
x ... x Un es una función P(x1, x2, ..., xn) con dominio de individuos U1 x U2 x ... x Un y los
valores de la función son proposiciones. Las variables x1, x2, ..., xn para P(x1, x2, ..., xn) son todas
variables libres para el predicado y cada xj varía en su correspondiente dominio de individuos Uj. El
término ¨libre¨ es la abreviación de ¨libre para sustitución¨, queriendo decir que la variable xj está
disponible en caso de que queramos sustituir un valor particular de Uj cada vez que aparezca xj.
Si sustituimos xj por un valor, digamos que por ejemplo sustituimos x1 por a, en P(x1, x2, ..., xn)
obtenemos el predicado P(a, x2, ..., xn) que es libre en las restantes n − 1 variables x2, ..., xn pero
ya no lo es en x1. Al aplicar un cuantificador ∀xj o ∃xj a un predicado P(x1, x2, ..., xn) obtenemos
un predicado ∀xj P(x1, x2, ..., xn) o ∃xj P(x1, x2, ..., xn) cuyos valores dependen únicamente de las
restantes n − 1 variables. Decimos que el cuantificador liga la variable xj, haciendo que xj sea una
variable acotada para el predicado. Al aplicar n cuantificadores, uno para cada variable, obtenemos
que todas las variables estén acotadas y obtenemos una proposición cuyo valor de verdad puede
determinarse aplicando las reglas para ∀x y ∃x, para los dominios de individuos U1, U2 , ..., Un.
Ejemplo 1.51 Anteriormente notamos que un predicado de n argumentos se transforma en
un predicado de (n − 1) argumentos cuando se liga una de las variables con un cuantificador. Su
valor de verdad depende de los valores de verdad de las restantes (n − 1) variables libres y en
particular no depende de qué nombre elijamos para llamar la variable acotada. De esta manera si
P(x) es predicado de un argumento con dominio de individuos U, entonces ∀x P(x), ∀y P(y) y
∀z P(z) tienen todas el mismo valor de verdad, es decir P(n), es verdadero para toda n en U y
falso en cualquier otro caso. De manera semejante, si Q(x, y) es un predicado de dos argumentos
con dominio de individuos U y V , entonces ∃y Q(x, y), ∃t Q(x, t) y ∃s Q(x, s) describen todas el
mismo predicado de un argumento, a saber, el predicado que es verdadero para una x dada en U
si y sólo si Q(x, V ) es verdadero para alguna V en V que es el dominio de la segunda variable. Por
otro lado, el predicado ∃x Q(x, x), no es el mismo que los tres últimos. La diferencia consiste en
CAPÍTULO 1. LÓGICA MATEMÁTICA 34
que el cuantificador en este caso liga las dos variables libres.
Otra práctica común es dar una descripción del dominio de individuos justo después de la variable
cuantificada. Por ejemplo, en lugar de ¨sea R el dominio de individuos ... ∀x P(x)¨ podríamos
escribi ∀x ∈ R P(x). De manera similar, ∃x ∈ R ∀n ∈ P(xn
> x) se lee como ¨hay un número real
x tal que para toda n en P, xn
> x¨ o como ¨hay un número real x tal que xn
> x para toda n en
P¨.
1.4.1. Leyes de la lógica de predicados
Las ideas de demostración y de teorema que se discutió para el cálculo proposicional, pueden
extenderse al ámbito del cálculo de predicados. No es sorprendente que con más expresiones po-
sibles tengamos también mayores complicaciones. Una relación moderadamente completa de este
tema puede formar una parte sustancial de otro libro. En esta sección nos limitaremos a discutir
algunas de las más básicas y útiles conexiones entre los cuantificadores y los operadores lógicos.
En el capítulo anterior utilizamos la expresión proposición compuesta de manera informal para
describir proposiciones construidas a partir de proposiciones más simples.
Las leyes de la lógica de predicados que no se pueden obtener por medio de la sustitución de
las leyes de la lógica proposicional, son por ejemplo:
1. ∀x P(x) → P(a)
∀x P(x) → P(a) prueba que, si cada individuo de un conjunto posee una determinada
propiedad P, entonces existe también un individuo determinado a que posee esta propiedad.
2. P(a) → ∃x P(x)
P(a) → ∃x P(x) prueba que, si un individuo determinado de un conjunto de individuos
posee una determinada propiedad P, existe entonces, por lo menos un individuo a con esta
propiedad.
Toda expresión de la lógica proposicional con validez general puede convertirse en una expresión
de la lógica de predicados con validez general, pero el recíproco es falso.
Podríamos intentar obtener, por medio de la ssustitución de una expresión de la lógica pro-
posicional satisfactible sin validez general, una expresión de la lógica de predicados igualmente
satisfactible, pero sin validez general. Pongamos por ejemplo en la neutralidad de la lógica propo-
sicional P ∧ Q para la variable proposicional
P ∼= ∀x[P(x) ∨ ¬P(x)]
y para
Q ∼= ∃x[P(x) ∧ ¬P(x)]
de esta forma obtenemos la expresión
∀x[P(x) ∨ ¬P(x)] ∧ Q ∼= ∃x[P(x) ∧ ¬P(x)].
Esta expresión es una contradicción.
Por el contrario resulta que: ¨Toda expresión de la lógica proposicional, no ejecutable, satisfac-
tible, es también una expresión de la lógica de predicados, no ejecutable, satisfactible¨.
Algunas equivalencias de la lógica de predicados, que expresan la relación que se establece entre
los cuantificadores ∀ y ∃ reciben especial atención. Una equivalencia de la lógica de predicados tiene
CAPÍTULO 1. LÓGICA MATEMÁTICA 35
tanta validez general como una equivalencia de la lógica proposicional, si coinciden en cada caso
los valores de verdad de ambos términos en iguales sustituciones de sus variables.
Se obtiene una proposición verdadera en cada sustitución de las variables del dominio, a partir
de un conjunto no vacío dado, y en cada sustitución de las variables del predicado P. Esta expre-
sión es una forma, en la lógica de predicados del conocido teorema del tercer excluido de la lógica
proposicional.
Las identidades de la lógica de predicados (leyes) se pueden obtener de las identidades lógicas
proposicionales si las variables son sustituidas por formas proposicionales de la lógica de predicados
en las expresiones de la lógica proposicional correspondiente.
En muchos casos nos encontramos que estas expresiones tienen que ver con formas proposicio-
nales, que se han obtenido mediante la combinación de dos o más proposiciones como dos formas
proposicionales. La traducción de expresiones de la lógica de predicados en el lenguaje común es
generalmente más fácil que la traducción en dirección contraria. Sobre todo existen dificultades
cuando se presentan, por ejemplo, dos o más operadores.
Teorema 1.15 Las siguientes equivalencias son válidas:
∀x ∀y P(x, y) ∼= ∀y ∀x P(x, y) y ∃x ∃y P(x, y) ∼= ∃y ∃x P(x, y)
Demostración
Para demostrar que ∃x ∃y P(x, y) ∼= ∃y ∃x P(x, y) es una tautología, debemos revisar que esta
proposición es verdadera para todos los dominios del discurso posibles. Por la definición de ↔,
necesitamos revisar solamente que ∃y ∃x P(x, y) es verdadera para un dominio dado si y sólo si
∃x ∃y P(x, y) es verdadera para ese dominio.
Supongamos que ∃x ∃y P(x, y) tiene valor verdadero. Entonces ∃y P(x0, y) es verdadera para
alguna x0 en el universo, por lo tanto P(x0, y0) es verdadera para alguna y0 en el dominio. De ahí
que ∃x P(x, y0) es verdadera y por lo tanto ∃y ∃x P(x, y) es verdadera. La implicación en la otra
dirección es similar.
Más aún, las dos proposiciones ∃x ∃y P(x, y) y ∃y ∃x P(x, y) son lógicamente equivalentes a la
proposición ∃(x, y) P(x, y) donde (x, y) varía sobre D1 x D2, con D1 y D2 los dominios del discurso
de las variables x e y respectivamente.
Teorema 1.16 Es válida la siguiente identidad:
∃x ∃y P(x, y) ∼= ∀y ∃x P(x, y)
Demostración
Para poder demostrar este teorema, asumimos que si la parte izquierda de esta proposición es
verdadero entonces existe x0 en el dominio de discurso tal que ∀y P(x0, y) es verdadero y así
P(x0, y) es verdadero para toda y. Por lo tanto, para cada y, ∃x P(x, y) es verdadero; de hecho
la misma x0 sirve para cada y. Como ∃x P(x, y) es verdadero para toda y, el lado derecho de la
proposición tiene valor de verdad verdadero. De esta manera la proposición es una tautología.
Por otra parte el recíproco de esta proposición, es decir ∀y ∃x P(x, y) ∼= ∃x ∀y P(x, y) no es en
general verdadero. Para enfatizar la diferencia, supongamos que x e y varían sobre un dominio D
de tres elementos, digamos D = {a, b, c}. El predicado de 2 argumentos P(x, y) tiene nueve posibles
valores;
P(a, a); P(a, b); P(a, c); P(b, a); P(b, b); P(b, c); P(c, a); P(c, b); P(c, c).
CAPÍTULO 1. LÓGICA MATEMÁTICA 36
Entonces ∃x ∀y P(x, y) es verdadero si ∀y P(x0, y) es verdadero para alguna x0. Como x0 tiene
que ser igual a a, b o c vemos que ∃x ∀y P(x, y) es verdadero si y sólo si todas las proposiciones de
una de las filas dadas arriba son verdaderas. En contraste, ∀y ∃x P(x, y) sería verdadera siempre
que al menos una proposición de cada columna sea verdadera.
Por ejemplo si consideramos un predicado P(x, y) con valores de verdad
P(a, a) P(a, b) P(a, c) P(b, a) P(b, b) P(b, c) P(c, a) P(c, b) P(c, c)
V F F F F V F V V
entonces ∀y ∃x P(x, y) será verdadera en tanto que ∃x ∀y P(x, y) será falsa. Para esta elección
de predicado P(x, y), ∃x P(x, y) es verdadera para toda y pero la x adecuada depende de la y,
ninguna x única sirve para toda y.
Teorema 1.17 Las identidades siguientes son válidas:
¬∀x P(x) ∼= ∃x [¬P(x)]; ¬∃x P(x) ∼= ∀x [¬P(x)];
∀x P(x) ∼= ¬∃x [¬P(x)]; ∃x P(x) ∼= ¬∀x [¬P(x)].
Ejemplo 1.52 Las leyes de DeMorgan pueden utilizarse repetidamente para negar cualquier
proposición cuantificada
¬∃w ∀x ∃y ∃z P(w, x, y, z)
es sucesivamente lógica equivalente a
∀w[¬∀x ∃y ∃z P(w, x, y, z)]; ∀w ∃x[¬∃y ∃z P(w, x, y, z)];
∀w ∃x ∀y[¬∃z P(w, x, y, z)]; ∀w ∃x ∀y ∀z[¬P(w, x, y, z)];
Esto ilustra la regla general: La negación de un predicado cuantificado es lógicamente equiva-
lente a la proposición que se obtiene al sustituir cada ∀ por ∃ y cada ∃ por ∀ y reemplazando el
mismo predicado por su negación.
Ejemplo 1.53 La negación de
∀x ∀y ∃z (x < z < y) es ∃x ∃y ∀z [¬(x < z < y)].
Aplicando las leyes de DeMorgan vemos que la negación es lógicamente equivalente a
∃x ∃y ∀z [(z ≤ x) ∨ (z ∧ y)]
Ejemplo 1.54 La negación de
∀x ∀y (x < y → x2
< y2
) es ∃x ∃y [¬(x < z → x2
< y2
)].
Aplicando las leyes de DeMorgan vemos que la negación es lógicamente equivalente a
∃x ∃y [(x < y) ∧ (x2
≥ y2
)]
CAPÍTULO 1. LÓGICA MATEMÁTICA 37
1.4.2. Interpretación de fórmulas en la lógica de predicados
En la lógica proposicional una interpretación es una asignación de valores de verdad a átomos.
En la lógica de predicados, puesto que hay variables involucradas, hay que hacer más que eso. Para
definir una interpretación para una fórmula en la lógica de predicados, tenemos que especificar dos
cosas, el dominio y una asignación a constantes, símbolos de función y símbolos de predicado que
ocurren en la fórmula. A continuación se da la definición formal de interpretación de una fórmula
en la lógica de predicados.
Definición 1.24 Interpretación de una fórmula
Una interpretación de una fórmula G en la lógica de predicados, consicte de un dominio D no
vacío, y una asignación de valores a cada constante, símbolos de función, y símbolos de predicado
que ocurre en G de la siguiente manera:
1. A cada constante asignamos un elemento en D;
2. A cada símbolo de función asignamos una aplicación de Dn
a D, Dn
= {x1, x2, ..., xn ∈ D}
3. A cada símbolo de predicado asignamos una aplicación de Dn
a {V, F}.
Algunas veces para enfatizar el dominio D, hablaremos de una interpretación de la fórmula sobre
D. Cuando evaluamos el valor de verdad de una fórmula en una interpretación sobre el dominio
D, ∀x será interpretada como ¨para todos los elementos x en D¨, y ∃x como ¨hay un elemento en
D¨. Para cada interpretación de una fórmula sobre un dominio de individuos D, la fórmula puede
ser evaluada a V o F de acuerdo a las siguientes reglas:
1. Si los valores de verdad de las fórmulas H y G son evaluadas, entonces los valores de verdad
de las fórmulas ¬H, H ∨ G, H ∧ G, H → G, H ↔ G son evaluadas de la siguiente manera:
H G ¬ H H ∨ G H ∧ G H → G H ↔ G
V V F V V V V
V F F V F F F
F V V V F V F
F F V F F V V
2. ∀x H es evaluada a V si el valor verdadero de H es valuado a V para cada d ∈ D, de otra
manera es evaluado a F.
3. ∃x H es evaluado a V si el valor de verdad de H es V para por lo menos un d ∈ D, de otra
manera es evaluada a F.
Se puede notar fácilmente que cualquier fórmula conteniendo variables libres no puede ser
evaluada. En adelante asumiremos, ya sea que las fórmulas no contienen variables libres o que las
variables son tratadas como constantes.
Ejemplo 1.55 Considere la fórmula ∀x ∃y P(x, y), D = {1, 2}
P(1, 1) ∼= V ; P(1, 2) ∼= F; P(2, 1) ∼= F; P(2, 2) ∼= V.
Si x = 1, podemos ver que hay un y tal que P(1, y) es verdadero. Si x = 2 hay también un y
denominado 2 tal que P(2, y) es verdadero, por consiguiente en las interpretaciones de arriba, para
cada x en D hay un y tal que P(x, y) es verdadero, esto es ∀x ∃y P(x, y) es verdadero en esta
interpretación.
CAPÍTULO 1. LÓGICA MATEMÁTICA 38
Ejemplo 1.56 Considere la fórmula ∀x [P(x) → Q(f(x), k)]. Hay una constante k, un símbolo
de función f de un lugar, un símbolo de predicado P de un lugar, y un símbolo de predicado Q de
dos lugares. La siguiente es una interpretación I. Dominio D = {1, 2}.
Asignación para k: a = 1.
Asignación para f: f(1) = 2; f(2) = 1.
Asignaciones para P y Q:
P(1) = F; P(2) = V ; Q(1, 1) = V ; Q(1, 2) = V ; Q(2, 1) = F; Q(2, 2) = V.
Si x = 1, entonces
P(x) → Q(f(x), k) = P(1) → Q(f(1), k) = P(1) → Q(2, 1) = F → F = V.
Si x = 2, entonces
P(x) → Q(f(x), k) = P(2) → Q(f(2), k) = P(2) → Q(1, 1) = V → V = V.
Puesto que P(x) → Q(f(x), k) es verdadero para todos los valores de x en D, la fórmula
∀x [P(x) → Q(f(x), k)]
es verdadera bajo las interpretaciones I.
Ejemplo 1.57 Evaluar los valores de verdad de las siguientes fórmulas bajo las interpretacio-
nes dadas en el ejemplo anterior.
1. ∃x [P(f(x)) ∧ Q(x, f(k))];
2. ∃x [P(x) ∧ Q(x, k)];
3. ∀x ∃y [P(x) ∧ Q(x, y)].
Para 1): Si x = 1, entonces
P(f(x)) ∧ Q(x, f(k)) = P(f(1)) ∧ Q(1, f(1)) = P(2) ∧ Q(1, f(1)) = P(2) ∧ Q(1, 2) = V ∧ V = V.
Si x = 2, entonces
P(f(x)) ∧ Q(x, f(k)) = P(f(2)) ∧ Q(2, f(1)) = P(1) ∧ Q(2, 1) = F ∧ F = F.
Puesto que hay un elemento en el dominio D, esto es x = 1 tal que P(f(x))∧Q(x, f(k)) es verdadero,
el valor de verdad de la fórmula ∃x [P(f(x)) ∧ Q(x, f(k))] es verdadera bajo la interpretación I.
Para b): Si x = 1, entonces
P(x) ∧ Q(x, k) = P(1) ∧ Q(1, 1) = F ∧ V = F.
Si x = 2, entonces
P(x) ∧ Q(x, k) = P(2) ∧ Q(2, 1) = V ∧ F = F.
Puesto que no hay elemento en el dominio D tal que P(x) ∧ Q(x, k) sea verdadero, la fórmula
∃x [P(x) ∧ Q(x, k)]
es evaluada a falsa bajo la interpretación I.
Para c): Si x = 1, entonces P(x) = P(1) = F.
Por consiguiente P(x) ∧ Q(x, y) = F para y = 1 e y = 2. Puesto que existe un x, que es x = 1, la
fórmula ∃y [P(x)∧Q(x, y)] es falsa, la fórmula ∀x ∃y [P(x)∧Q(x, y)] es falsa bajo la interpretación
I, esto es, la fórmula es falsificada por I.
CAPÍTULO 1. LÓGICA MATEMÁTICA 39
Definición 1.25 Fórmula consistente
Una fórmula G es consistente (satisfactible) si y sólo si existe una interpretación I tal que G es
evaluada verdadero en I. Si una fórmula G es verdadera en una interpretación I, decimos que I es
un modelo de G e I satisface a G.
Definición 1.26 Fórmula válida
Una fórmula G es válida si, y sólo si cada interpretación de G satisface a G.
Definición 1.27 Fórmula inconsistente
Una fórmula G es inconsistente (insatisfactible) si y sólo si, no existe una interpretación que
satisface a G.
Las relaciones entre validez (inconsistencia) y consecuencias lógicas, como se indica en la lógica
proposicional, son también verdaderas para la lógica de predicados. En efecto, la lógica de predica-
dos puede ser considerada como una extensión de la lógica proposicional. Cuando una fórmula en
la lógica de predicados no contiene variables y cuantificadores, puede ser tratada justo como una
fórmula en la lógica proposicional.
Ejemplo 1.58
1. ∀x P(x) ∧ ∃y ¬P(y) es inconsistente;
2. ∀x P(x) → ∃y P(y) es válido;
3. P(k) → ¬∃x P(x) es consistente;
4. ∀x P(x) ∨ ∃y ¬P(y) es válido.
En la lógica de predicados, puesto que hay un número infinito de elementos en el dominio D, en
general, hay un número infinito de interpretaciones de una fórmula. Por consiguiente al contrario de
la lógica proposicional, no es posible verificar la validez e inconsistencia de una fórmula, evaluando
la fórmula bajo todas las posibles interpretaciones.
1.4.3. Forma normal prenexa
En la lógica proposicional hemos introducido dos formas normales, la forma normal conjuntiva
y la forma normal disjuntiva. En la lógica de predicados hay una forma normal llamada forma
normal Prenexa. La razón para considerar una forma normal Prenexa de una fórmula es simplificar
procedimientos de prueba.
Definición 1.28 Forma normal prenexa
Una fórmula G en la lógica de predicados se dice que es una forma normal Prenexa si y sólo si, la
fórmula G está en la forma
(Q1x1)(Q2x2)...(Qnxn)(M)
donde cada (Qixi), i = 1, 2, ..., n ya sea ∀xi o ∃xi, y M es una fórmula que no contiene cuan-
tificadores, (Q1x1)(Q2x2)...(Qnxn) es llamada el prefijo y M es llamada la matriz de la fórmula
G.
Dada una fórmula G, consideraremos un método de transformarla en una forma normal Prene-
xa. Esto se logra primero considerando algunos pasos básicos de fórmulas equivalentes en la lógica
de predicados. Recordemos que dos fórmulas G y H son equivalentes si, y sólo si los valores de
verdad de G y H son los mismos bajo cada interpretación.
CAPÍTULO 1. LÓGICA MATEMÁTICA 40
Los pares básicos de fórmulas equivalentes dadas en la lógica proposicional son todavía verdad
para la lógica de predicados, adicionalmente hay otros pares de fórmulas equivalentes conteniendo
cuantificadores, y que se estudiaron en secciones anteriores. Consideraremos estos pares adicionales
de fórmulas equivalentes.
Sea G una fórmula que contiene una variable libre x, para enfatizar que la variable libre está en
G, representamos G por G[x]. Sea H una fórmula que no contiene variable x, tenemos los siguientes
pares de fórmulas equivalentes, donde Q es ya sea ∀ o ∃:
1. (Qx)G[x] ∨ H ∼= (Qx)(G[x] ∨ H);
2. (Qx)G[x] ∧ H ∼= (Qx)(G[x] ∧ H);
3. ¬(∀xG[x]) ∼= ∃x(¬G[x]);
4. ¬(∃xG[x]) ∼= ∀x(¬G[x]).
Las leyes 1 y 2 son obviamente verdaderas puesto que H no contiene x, por consiguiente
puede ser introducida en el alcance del cuantificador Q. Las leyes 3 y 4 no son difíciles de
probar. Sea I cualquier interpretación arbitraria sobre el dominio D.
Si ¬(∀x G[x]) es verdadera en I, entonces ∀x G[x] es falsa en I. Esto significa que hay un
elemento d en D tal que G[d] es falso. Esto es ¬G[d] es verdadero en I. Por consiguiente,
∃x (¬G[x]) es verdadera en I. Por otra parte si ¬(∀x G[x]) es falsa en I, entonces ∀x G[x] es
verdadera en I. Esto significa que G[x] es verdadera para cada elemento x en D, esto es ¬G[x]
es falso para cada elemento x en D, por consiguiente, ∀x (¬G[x]) es falsa en I. Puesto que
¬(∀x G[x]) y ∀x (¬G[x]) siempre asume el mismo valor de verdad para cada interpretación
arbitraria, por definición, ¬(∀x G[x]) ∼= ∃x (¬G[x]). Así la ley 3 es probada e igualmente
podemos probar la ley 4.
Supongamos que F[x] y G[x] son dos fórmulas que contienen x,
5. ∀x F[x] ∧ ∀x G[x] ∼= ∀x (F[x] ∧ G[x])
6. ∃x F[x] ∨ ∃x G[x] ∼= ∃x (F[x] ∨ G[x])
Esto es, el cuantificador universal ∀ y el existencial ∃, pueden distribuirse sobre ∧ y ∨,
respectivamente. Sin embargo el cuantificador universal y existencial no pueden distribuirse
sobre ∨ y ∧ respectivamente. Esto es
∀x F[x] ∨ ∀x G[x] = ∀x (F[x] ∨ G[x])
∃x F[x] ∧ ∃x G[x] = ∃x (F[x] ∧ G[x])
Para casos como estos tenemos que hacer algo especial. Puesto que cada variable ligada en
una fórmula puede ser considerada como una variable renombrable, cada variable x puede
ser renombrada z, y la fórmula ∀x G[x] se transforma en ∀z G[z].
Supongamos que escogemos la variable z que no aparece en F[x]. Entonces
∀x F[x] ∨ ∀x G[x] ∼= ∀x F[x] ∨ ∀z G[z] ∼= ∀x∀z (F[x] ∨ G[z])
Similarmente, renombrando todas las x que ocurren en ∃x G[x] como z, podemos tener
∃x F[x] ∧ ∃x G[x] ∼= ∃x F[x] ∧ ∃z G[z] ∼= ∃x∃z (F[x] ∧ G[z])
CAPÍTULO 1. LÓGICA MATEMÁTICA 41
Por consiguiente, para estos dos casos podemos todavía pasar todos los cuantificadores a la
izquierda de la fórmula. En general, tenemos
7. (Q1x)F[x] ∨ (Q2x)G[x] ∼= (Q1x)(Q2x)(F[x] ∨ G[z])
8. (Q3x)F[x] ∧ (Q4x)G[x] ∼= (Q3x)(Q4x)(F[x] ∧ G[z])
donde Q1, Q2, Q3 y Q4 son ya sea ∀ o ∃, y z no aparece en F[x].
Naturalmente si Q1 = Q2 = ∃ y Q3 = Q4 = ∀, entonces no tenemos que renombrar las x en
(Q2x)G[x] o (Q4x)G[x]. Podemos usar las leyes 5 y 6 directamente. Usando las leyes de la lógica
proposicional y las leyes 1 - 8, podemos siempre transformar una fórmula dada en forma normal
Prenexa.
La siguiente es una guía del procedimiento de transformación:
PASO 1: Use las leyes
1. F ↔ G ∼= (F → G) ∧ (G → F);
2. F → G ∼= ¬F ∨ G;
Para eliminar las conectividades lógicas ↔ y →.
PASO 2: Repetidamente use las leyes
3. ¬(¬F) ∼= F;
4. ¬(F ∨ G) ∼= ¬F ∧ ¬G;
5. ¬(G ∧ G) ∼= ¬F ∨ ¬G;
6. ¬(∀x F[x]) ∼= ∃x (¬F[x]);
7. ¬(∃x F[x]) ∼= ∀x (¬F[x]);
para traer los signos de negación inmediatamente antes de los átomos.
PASO 3: Renombrar las variables ligadas si es necesario.
PASO 4: Use las leyes
8. (Qx)F[x] ∨ G ∼= (Qx)(F[x] ∨ G);
9. (Qx)F[x] ∧ G ∼= (Qx)(F[x] ∧ G);
10. ∀x F[x] ∧ x G[x] ∼= ∀x (F[x] ∧ G[x]);
11. ∃x F[x] ∨ ∃x G[x] ∼= ∃x (F[x] ∨ G[x]);
12. (Q1x)F[x] ∨ (Q2x)G[x] ∼= (Q1x)(Q2x)(F[x] ∨ G[z]);
13. (Q3x)F[x] ∧ (Q4x)G[x] ∼= (Q3x)(Q4x)(F[x] ∧ G[z]).
para mover los cuantificadores a la izquierda de la fórmula y obtener una forma normal
Prenexa.
CAPÍTULO 1. LÓGICA MATEMÁTICA 42
Ejemplo 1.59 Transformar la fórmula ∀x P(x) → ∃x Q(x) en forma normal prenexa.
Solución
∀x P(x) → ∃x Q(x) ∼= ¬∀x P(x) ∨ ∃x Q(x)
∼= ∃x¬P(x) ∨ ∃x Q(x)
∼= ∃x [¬P(x) ∨ Q(x)].
Ejemplo 1.60 Transformar la fórmula ∀x ∀y {∃z [P(x, z) ∧ P(y, z)] → ∃u Q(x, y, u)} en
forma normal Prenexa.
Solución
∀x ∀y{∃z [P(x, z) ∧ P(y, z)] → ∃u Q(x, y, u)} ∼= ∀x ∀y{¬∃z [P(x, z) ∧ P(y, z)] ∨ ∃u Q(x, y, u)}
∼= ∀x ∀y{∀z ¬[P(x, z) ∧ P(y, z)] ∨ ∃u Q(x, y, u)}
∼= ∀x ∀y ∀z ∃u {¬P(x, z) ∨ ¬P(y, z) ∨ ∃u Q(x, y, u)}.
1.4.4. Tarea
1. Sea A = {1, 2, 3, 4} el conjunto universal. Determine el valor de verdad de cada enunciado:
a) ∀x : x + 3 < 6; b) ∀x : x2
− 10 ≤ 8; c) ∃x : x2
> 1 → x + 2 = 0;
d) ∃x : 2x2
+ x = 15.
Resp: a) Falso; b) Verdadero; c) Verdadero; d) Falso.
2. Determine el valor de verdad de las siguientes proposiciones, siendo N el universo:
a) ∀x ∃y (2y = x); b) ∃y ∀x (2x = y); c) ∀x ∃y (2x = y);
d) ∃y ∀x (2y = x); e) ∀x ∀y [¬(2y = x)].
Resp: a) ; b) ; c) ; d) ; e) .
3. Determine el valor de verdad de las siguientes proposiciones, siendo R el universo:
a) ∀x ∃y (xy = 1); b) ∀x ∀y [(x + y)2
= x2
+ y2
]; c) ∃x ∃y (x2
+ y2
+ 1 = 2xy);
d) ∃x ∃y [(x + 2y = 4) ∧ (2x − y = 2)].
Resp: a) ; b) ; c) ; d) .
4. Determine el valor de verdad de las siguientes proposiciones, siendo R el universo:
a) ∀x ∈ R x2
≥ x; b) ∃x ∈ R 2x = x; c) ∀x ∈ R 2x−1
4x−2 = 1
2 ;
d) ∃x ∈ R x2
+ 2x + 1 ≤ 0.
Resp: a) ; b) ; c) ; d) .
5. Negar los siguientes enunciados:
a) ∃y p(y) → ∀x(¬q(x)); b) ∃x(¬p(x)) ∨ ∀x q(x); c) ∃x ∃y (p(x, y) → q(x, y)).
Resp: a) ∃y p(y) ∧ ∃x q(x); b) ∀x p(x) ∧ ∃x(¬q(x)); c) ∀ ∃y(p(x, y) ∧ ¬q(x, y)).
6. Negar las siguientes afirmaciones:
a) ∀x ∀y [(x + y es impar) → (x es impar ∨ y es impar)];
b) ∀x ∃y (x + y = 5 → y = −x); c) ∃x ∀y (x < y ∧ x2
≥ y);
d) ∀x ∀y ∃z (x < y → x + z = y).
Resp: a) ; b) ; c) ; d) .
7. Averiguar el valor de verdad siendo U = R:
a) ∀x ∈ R (x < 0 → x < 3); b) ∃x ∈ R (x2
≥ 0 → x4
= x3
);
c) ∀x ∈ R, ∃y ∈ R (x2
+ y2
= 1); d) ∀x ∈ R, ∀y ∈ R (y < x → 2y < 10).
Resp: a) Verdadero; b) Verdadero; c) Falso; d) Falso.
Logica Matemática

Más contenido relacionado

La actualidad más candente

Lógica proposicional tablas de verdad
Lógica proposicional tablas de verdadLógica proposicional tablas de verdad
Lógica proposicional tablas de verdadVideoconferencias UTPL
 
Proposiciones y su clasificación
Proposiciones y su clasificaciónProposiciones y su clasificación
Proposiciones y su clasificaciónangiegutierrez11
 
La relación entre la f y el r2
La relación entre la f y el r2La relación entre la f y el r2
La relación entre la f y el r2PatriciaPasiche
 
Ecuaciones diferenciales lineales
Ecuaciones diferenciales linealesEcuaciones diferenciales lineales
Ecuaciones diferenciales linealesfernandamendozadt
 
Logica matematica
Logica matematicaLogica matematica
Logica matematicadrakul09
 
Fracciones Parciales/ Segundo Caso/ Denominador con factores de primer grado...
Fracciones Parciales/ Segundo Caso/ Denominador con factores de primer grado...Fracciones Parciales/ Segundo Caso/ Denominador con factores de primer grado...
Fracciones Parciales/ Segundo Caso/ Denominador con factores de primer grado...Mareli Rodríguez Ovalle
 
Ecuaciones diferenciales lineales
Ecuaciones diferenciales linealesEcuaciones diferenciales lineales
Ecuaciones diferenciales linealesAlexCoeto
 
Negación de proposiciones con cuantificadores
Negación de proposiciones con cuantificadoresNegación de proposiciones con cuantificadores
Negación de proposiciones con cuantificadoresAntoKizz Caztro
 
Cálculo de Predicados
Cálculo de PredicadosCálculo de Predicados
Cálculo de Predicadosrezzaca
 
INTEGRALES IMPROPIAS
INTEGRALES IMPROPIASINTEGRALES IMPROPIAS
INTEGRALES IMPROPIASDi Pater
 
Teorema del residuo
Teorema del residuoTeorema del residuo
Teorema del residuoimanolrd
 
Función proposicional [recuperado]
Función proposicional [recuperado]Función proposicional [recuperado]
Función proposicional [recuperado]Victor Alegre
 
Leyes del algebra proposicional
Leyes del algebra proposicionalLeyes del algebra proposicional
Leyes del algebra proposicionalRonald Wielman
 
TRANSFORMACIONES LINEALES
TRANSFORMACIONES LINEALESTRANSFORMACIONES LINEALES
TRANSFORMACIONES LINEALESbriyit campos
 
Sistemas numericos y de conversion
Sistemas numericos y de conversionSistemas numericos y de conversion
Sistemas numericos y de conversionNadiaCerrud
 
LMF-T1: Sintaxis y semántica de la lógica proposicional
LMF-T1: Sintaxis y semántica de la lógica proposicionalLMF-T1: Sintaxis y semántica de la lógica proposicional
LMF-T1: Sintaxis y semántica de la lógica proposicionalJosé A. Alonso
 

La actualidad más candente (20)

Lógica proposicional tablas de verdad
Lógica proposicional tablas de verdadLógica proposicional tablas de verdad
Lógica proposicional tablas de verdad
 
Proposiciones y su clasificación
Proposiciones y su clasificaciónProposiciones y su clasificación
Proposiciones y su clasificación
 
Revisando la jerarquía de chomsky
Revisando la jerarquía de chomskyRevisando la jerarquía de chomsky
Revisando la jerarquía de chomsky
 
La relación entre la f y el r2
La relación entre la f y el r2La relación entre la f y el r2
La relación entre la f y el r2
 
Ecuaciones diferenciales lineales
Ecuaciones diferenciales linealesEcuaciones diferenciales lineales
Ecuaciones diferenciales lineales
 
Logica matematica
Logica matematicaLogica matematica
Logica matematica
 
Fracciones Parciales/ Segundo Caso/ Denominador con factores de primer grado...
Fracciones Parciales/ Segundo Caso/ Denominador con factores de primer grado...Fracciones Parciales/ Segundo Caso/ Denominador con factores de primer grado...
Fracciones Parciales/ Segundo Caso/ Denominador con factores de primer grado...
 
Limites
LimitesLimites
Limites
 
Ecuaciones diferenciales lineales
Ecuaciones diferenciales linealesEcuaciones diferenciales lineales
Ecuaciones diferenciales lineales
 
Negación de proposiciones con cuantificadores
Negación de proposiciones con cuantificadoresNegación de proposiciones con cuantificadores
Negación de proposiciones con cuantificadores
 
Cuantificadores
CuantificadoresCuantificadores
Cuantificadores
 
Cálculo de Predicados
Cálculo de PredicadosCálculo de Predicados
Cálculo de Predicados
 
INTEGRALES IMPROPIAS
INTEGRALES IMPROPIASINTEGRALES IMPROPIAS
INTEGRALES IMPROPIAS
 
Teorema del residuo
Teorema del residuoTeorema del residuo
Teorema del residuo
 
Función proposicional [recuperado]
Función proposicional [recuperado]Función proposicional [recuperado]
Función proposicional [recuperado]
 
Leyes del algebra proposicional
Leyes del algebra proposicionalLeyes del algebra proposicional
Leyes del algebra proposicional
 
TRANSFORMACIONES LINEALES
TRANSFORMACIONES LINEALESTRANSFORMACIONES LINEALES
TRANSFORMACIONES LINEALES
 
Sistemas numericos y de conversion
Sistemas numericos y de conversionSistemas numericos y de conversion
Sistemas numericos y de conversion
 
LMF-T1: Sintaxis y semántica de la lógica proposicional
LMF-T1: Sintaxis y semántica de la lógica proposicionalLMF-T1: Sintaxis y semántica de la lógica proposicional
LMF-T1: Sintaxis y semántica de la lógica proposicional
 
Leyes de las oposiciones lógicas
Leyes de las oposiciones lógicasLeyes de las oposiciones lógicas
Leyes de las oposiciones lógicas
 

Destacado

Tema 2 logica proposicional
Tema 2 logica proposicionalTema 2 logica proposicional
Tema 2 logica proposicionalEva Vásquez
 
Cap 1 logica
Cap 1 logicaCap 1 logica
Cap 1 logicaDLCR2015
 
Logica hipotesis y conclusion
Logica hipotesis y conclusionLogica hipotesis y conclusion
Logica hipotesis y conclusionPerez Kyria
 
Unidad 1, logica y conjuntos
Unidad 1, logica y conjuntosUnidad 1, logica y conjuntos
Unidad 1, logica y conjuntosROYBARRE
 
MODULO DE LÓGICA MATEMÁTICA 90004- 2012
MODULO DE LÓGICA MATEMÁTICA 90004- 2012MODULO DE LÓGICA MATEMÁTICA 90004- 2012
MODULO DE LÓGICA MATEMÁTICA 90004- 2012Armando Lopez Sierra
 

Destacado (7)

Lógica matemática
Lógica matemáticaLógica matemática
Lógica matemática
 
Lógica
LógicaLógica
Lógica
 
Tema 2 logica proposicional
Tema 2 logica proposicionalTema 2 logica proposicional
Tema 2 logica proposicional
 
Cap 1 logica
Cap 1 logicaCap 1 logica
Cap 1 logica
 
Logica hipotesis y conclusion
Logica hipotesis y conclusionLogica hipotesis y conclusion
Logica hipotesis y conclusion
 
Unidad 1, logica y conjuntos
Unidad 1, logica y conjuntosUnidad 1, logica y conjuntos
Unidad 1, logica y conjuntos
 
MODULO DE LÓGICA MATEMÁTICA 90004- 2012
MODULO DE LÓGICA MATEMÁTICA 90004- 2012MODULO DE LÓGICA MATEMÁTICA 90004- 2012
MODULO DE LÓGICA MATEMÁTICA 90004- 2012
 

Similar a Logica Matemática

Trabajo_calculoMarianySanchez.docx
Trabajo_calculoMarianySanchez.docxTrabajo_calculoMarianySanchez.docx
Trabajo_calculoMarianySanchez.docxmelaniepulido1
 
Aspectos resaltantes de la proposicion
Aspectos resaltantes de la proposicionAspectos resaltantes de la proposicion
Aspectos resaltantes de la proposicioneglisp
 
Calculo proposicional.docx
Calculo proposicional.docxCalculo proposicional.docx
Calculo proposicional.docxmelaniepulido1
 
Capitulo_1_Logica_y_Conjuntos.pdf
Capitulo_1_Logica_y_Conjuntos.pdfCapitulo_1_Logica_y_Conjuntos.pdf
Capitulo_1_Logica_y_Conjuntos.pdfYamilaNicelLescano
 
Trabajo de lógica matemática,,
Trabajo de lógica matemática,,Trabajo de lógica matemática,,
Trabajo de lógica matemática,,alejandraquevedo24
 
Trabajo de lógica matemática,,
Trabajo de lógica matemática,,Trabajo de lógica matemática,,
Trabajo de lógica matemática,,alejandraquevedo24
 
Trabajo de lógica matemática,,
Trabajo de lógica matemática,,Trabajo de lógica matemática,,
Trabajo de lógica matemática,,alejandraquevedo24
 
Lógica matemática
Lógica matemática Lógica matemática
Lógica matemática Dainer147
 
Trabajo de logica matematica
Trabajo de logica matematicaTrabajo de logica matematica
Trabajo de logica matematicaMaria_Olave
 
Trabajo de logica matematica modalidad.
Trabajo de logica matematica modalidad.Trabajo de logica matematica modalidad.
Trabajo de logica matematica modalidad.olave_julian
 

Similar a Logica Matemática (20)

Logica matematica
Logica matematicaLogica matematica
Logica matematica
 
Calculo proposicional
Calculo proposicionalCalculo proposicional
Calculo proposicional
 
Calculo proposicional
Calculo proposicionalCalculo proposicional
Calculo proposicional
 
Lógica proposicional
Lógica proposicionalLógica proposicional
Lógica proposicional
 
Lógica Matemática
Lógica MatemáticaLógica Matemática
Lógica Matemática
 
Trabajo_calculoMarianySanchez.docx
Trabajo_calculoMarianySanchez.docxTrabajo_calculoMarianySanchez.docx
Trabajo_calculoMarianySanchez.docx
 
Aspectos resaltantes de la proposicion
Aspectos resaltantes de la proposicionAspectos resaltantes de la proposicion
Aspectos resaltantes de la proposicion
 
lógica
lógicalógica
lógica
 
Calculo proposicional.docx
Calculo proposicional.docxCalculo proposicional.docx
Calculo proposicional.docx
 
Calculo proposicional.docx
Calculo proposicional.docxCalculo proposicional.docx
Calculo proposicional.docx
 
Capitulo_1_Logica_y_Conjuntos.pdf
Capitulo_1_Logica_y_Conjuntos.pdfCapitulo_1_Logica_y_Conjuntos.pdf
Capitulo_1_Logica_y_Conjuntos.pdf
 
Logica formal
Logica formalLogica formal
Logica formal
 
Yghor flores
Yghor floresYghor flores
Yghor flores
 
Trabajo de lógica matemática,,
Trabajo de lógica matemática,,Trabajo de lógica matemática,,
Trabajo de lógica matemática,,
 
Trabajo de lógica matemática,,
Trabajo de lógica matemática,,Trabajo de lógica matemática,,
Trabajo de lógica matemática,,
 
Trabajo de lógica matemática,,
Trabajo de lógica matemática,,Trabajo de lógica matemática,,
Trabajo de lógica matemática,,
 
Lógica matemática
Lógica matemática Lógica matemática
Lógica matemática
 
Lógica matemáticas
Lógica matemáticasLógica matemáticas
Lógica matemáticas
 
Trabajo de logica matematica
Trabajo de logica matematicaTrabajo de logica matematica
Trabajo de logica matematica
 
Trabajo de logica matematica modalidad.
Trabajo de logica matematica modalidad.Trabajo de logica matematica modalidad.
Trabajo de logica matematica modalidad.
 

Último

TIPOLOGÍA TEXTUAL- EXPOSICIÓN Y ARGUMENTACIÓN.pptx
TIPOLOGÍA TEXTUAL- EXPOSICIÓN Y ARGUMENTACIÓN.pptxTIPOLOGÍA TEXTUAL- EXPOSICIÓN Y ARGUMENTACIÓN.pptx
TIPOLOGÍA TEXTUAL- EXPOSICIÓN Y ARGUMENTACIÓN.pptxlclcarmen
 
Informatica Generalidades - Conceptos Básicos
Informatica Generalidades - Conceptos BásicosInformatica Generalidades - Conceptos Básicos
Informatica Generalidades - Conceptos BásicosCesarFernandez937857
 
Planificacion Anual 2do Grado Educacion Primaria 2024 Ccesa007.pdf
Planificacion Anual 2do Grado Educacion Primaria   2024   Ccesa007.pdfPlanificacion Anual 2do Grado Educacion Primaria   2024   Ccesa007.pdf
Planificacion Anual 2do Grado Educacion Primaria 2024 Ccesa007.pdfDemetrio Ccesa Rayme
 
RAIZ CUADRADA Y CUBICA PARA NIÑOS DE PRIMARIA
RAIZ CUADRADA Y CUBICA PARA NIÑOS DE PRIMARIARAIZ CUADRADA Y CUBICA PARA NIÑOS DE PRIMARIA
RAIZ CUADRADA Y CUBICA PARA NIÑOS DE PRIMARIACarlos Campaña Montenegro
 
DE LAS OLIMPIADAS GRIEGAS A LAS DEL MUNDO MODERNO.ppt
DE LAS OLIMPIADAS GRIEGAS A LAS DEL MUNDO MODERNO.pptDE LAS OLIMPIADAS GRIEGAS A LAS DEL MUNDO MODERNO.ppt
DE LAS OLIMPIADAS GRIEGAS A LAS DEL MUNDO MODERNO.pptELENA GALLARDO PAÚLS
 
Heinsohn Privacidad y Ciberseguridad para el sector educativo
Heinsohn Privacidad y Ciberseguridad para el sector educativoHeinsohn Privacidad y Ciberseguridad para el sector educativo
Heinsohn Privacidad y Ciberseguridad para el sector educativoFundación YOD YOD
 
Sesión de aprendizaje Planifica Textos argumentativo.docx
Sesión de aprendizaje Planifica Textos argumentativo.docxSesión de aprendizaje Planifica Textos argumentativo.docx
Sesión de aprendizaje Planifica Textos argumentativo.docxMaritzaRetamozoVera
 
ACUERDO MINISTERIAL 078-ORGANISMOS ESCOLARES..pptx
ACUERDO MINISTERIAL 078-ORGANISMOS ESCOLARES..pptxACUERDO MINISTERIAL 078-ORGANISMOS ESCOLARES..pptx
ACUERDO MINISTERIAL 078-ORGANISMOS ESCOLARES..pptxzulyvero07
 
CALENDARIZACION DE MAYO / RESPONSABILIDAD
CALENDARIZACION DE MAYO / RESPONSABILIDADCALENDARIZACION DE MAYO / RESPONSABILIDAD
CALENDARIZACION DE MAYO / RESPONSABILIDADauxsoporte
 
Neurociencias para Educadores NE24 Ccesa007.pdf
Neurociencias para Educadores  NE24  Ccesa007.pdfNeurociencias para Educadores  NE24  Ccesa007.pdf
Neurociencias para Educadores NE24 Ccesa007.pdfDemetrio Ccesa Rayme
 
30-de-abril-plebiscito-1902_240420_104511.pdf
30-de-abril-plebiscito-1902_240420_104511.pdf30-de-abril-plebiscito-1902_240420_104511.pdf
30-de-abril-plebiscito-1902_240420_104511.pdfgimenanahuel
 
Clasificaciones, modalidades y tendencias de investigación educativa.
Clasificaciones, modalidades y tendencias de investigación educativa.Clasificaciones, modalidades y tendencias de investigación educativa.
Clasificaciones, modalidades y tendencias de investigación educativa.José Luis Palma
 
el CTE 6 DOCENTES 2 2023-2024abcdefghijoklmnñopqrstuvwxyz
el CTE 6 DOCENTES 2 2023-2024abcdefghijoklmnñopqrstuvwxyzel CTE 6 DOCENTES 2 2023-2024abcdefghijoklmnñopqrstuvwxyz
el CTE 6 DOCENTES 2 2023-2024abcdefghijoklmnñopqrstuvwxyzprofefilete
 
la unidad de s sesion edussssssssssssssscacio fisca
la unidad de s sesion edussssssssssssssscacio fiscala unidad de s sesion edussssssssssssssscacio fisca
la unidad de s sesion edussssssssssssssscacio fiscaeliseo91
 
programa dia de las madres 10 de mayo para evento
programa dia de las madres 10 de mayo  para eventoprograma dia de las madres 10 de mayo  para evento
programa dia de las madres 10 de mayo para eventoDiegoMtsS
 
Planificacion Anual 4to Grado Educacion Primaria 2024 Ccesa007.pdf
Planificacion Anual 4to Grado Educacion Primaria   2024   Ccesa007.pdfPlanificacion Anual 4to Grado Educacion Primaria   2024   Ccesa007.pdf
Planificacion Anual 4to Grado Educacion Primaria 2024 Ccesa007.pdfDemetrio Ccesa Rayme
 
UNIDAD DPCC. 2DO. DE SECUNDARIA DEL 2024
UNIDAD DPCC. 2DO. DE  SECUNDARIA DEL 2024UNIDAD DPCC. 2DO. DE  SECUNDARIA DEL 2024
UNIDAD DPCC. 2DO. DE SECUNDARIA DEL 2024AndreRiva2
 
TECNOLOGÍA FARMACEUTICA OPERACIONES UNITARIAS.pptx
TECNOLOGÍA FARMACEUTICA OPERACIONES UNITARIAS.pptxTECNOLOGÍA FARMACEUTICA OPERACIONES UNITARIAS.pptx
TECNOLOGÍA FARMACEUTICA OPERACIONES UNITARIAS.pptxKarlaMassielMartinez
 

Último (20)

TIPOLOGÍA TEXTUAL- EXPOSICIÓN Y ARGUMENTACIÓN.pptx
TIPOLOGÍA TEXTUAL- EXPOSICIÓN Y ARGUMENTACIÓN.pptxTIPOLOGÍA TEXTUAL- EXPOSICIÓN Y ARGUMENTACIÓN.pptx
TIPOLOGÍA TEXTUAL- EXPOSICIÓN Y ARGUMENTACIÓN.pptx
 
Informatica Generalidades - Conceptos Básicos
Informatica Generalidades - Conceptos BásicosInformatica Generalidades - Conceptos Básicos
Informatica Generalidades - Conceptos Básicos
 
Planificacion Anual 2do Grado Educacion Primaria 2024 Ccesa007.pdf
Planificacion Anual 2do Grado Educacion Primaria   2024   Ccesa007.pdfPlanificacion Anual 2do Grado Educacion Primaria   2024   Ccesa007.pdf
Planificacion Anual 2do Grado Educacion Primaria 2024 Ccesa007.pdf
 
RAIZ CUADRADA Y CUBICA PARA NIÑOS DE PRIMARIA
RAIZ CUADRADA Y CUBICA PARA NIÑOS DE PRIMARIARAIZ CUADRADA Y CUBICA PARA NIÑOS DE PRIMARIA
RAIZ CUADRADA Y CUBICA PARA NIÑOS DE PRIMARIA
 
DE LAS OLIMPIADAS GRIEGAS A LAS DEL MUNDO MODERNO.ppt
DE LAS OLIMPIADAS GRIEGAS A LAS DEL MUNDO MODERNO.pptDE LAS OLIMPIADAS GRIEGAS A LAS DEL MUNDO MODERNO.ppt
DE LAS OLIMPIADAS GRIEGAS A LAS DEL MUNDO MODERNO.ppt
 
Repaso Pruebas CRECE PR 2024. Ciencia General
Repaso Pruebas CRECE PR 2024. Ciencia GeneralRepaso Pruebas CRECE PR 2024. Ciencia General
Repaso Pruebas CRECE PR 2024. Ciencia General
 
Heinsohn Privacidad y Ciberseguridad para el sector educativo
Heinsohn Privacidad y Ciberseguridad para el sector educativoHeinsohn Privacidad y Ciberseguridad para el sector educativo
Heinsohn Privacidad y Ciberseguridad para el sector educativo
 
Sesión de aprendizaje Planifica Textos argumentativo.docx
Sesión de aprendizaje Planifica Textos argumentativo.docxSesión de aprendizaje Planifica Textos argumentativo.docx
Sesión de aprendizaje Planifica Textos argumentativo.docx
 
ACUERDO MINISTERIAL 078-ORGANISMOS ESCOLARES..pptx
ACUERDO MINISTERIAL 078-ORGANISMOS ESCOLARES..pptxACUERDO MINISTERIAL 078-ORGANISMOS ESCOLARES..pptx
ACUERDO MINISTERIAL 078-ORGANISMOS ESCOLARES..pptx
 
CALENDARIZACION DE MAYO / RESPONSABILIDAD
CALENDARIZACION DE MAYO / RESPONSABILIDADCALENDARIZACION DE MAYO / RESPONSABILIDAD
CALENDARIZACION DE MAYO / RESPONSABILIDAD
 
Neurociencias para Educadores NE24 Ccesa007.pdf
Neurociencias para Educadores  NE24  Ccesa007.pdfNeurociencias para Educadores  NE24  Ccesa007.pdf
Neurociencias para Educadores NE24 Ccesa007.pdf
 
30-de-abril-plebiscito-1902_240420_104511.pdf
30-de-abril-plebiscito-1902_240420_104511.pdf30-de-abril-plebiscito-1902_240420_104511.pdf
30-de-abril-plebiscito-1902_240420_104511.pdf
 
Clasificaciones, modalidades y tendencias de investigación educativa.
Clasificaciones, modalidades y tendencias de investigación educativa.Clasificaciones, modalidades y tendencias de investigación educativa.
Clasificaciones, modalidades y tendencias de investigación educativa.
 
el CTE 6 DOCENTES 2 2023-2024abcdefghijoklmnñopqrstuvwxyz
el CTE 6 DOCENTES 2 2023-2024abcdefghijoklmnñopqrstuvwxyzel CTE 6 DOCENTES 2 2023-2024abcdefghijoklmnñopqrstuvwxyz
el CTE 6 DOCENTES 2 2023-2024abcdefghijoklmnñopqrstuvwxyz
 
la unidad de s sesion edussssssssssssssscacio fisca
la unidad de s sesion edussssssssssssssscacio fiscala unidad de s sesion edussssssssssssssscacio fisca
la unidad de s sesion edussssssssssssssscacio fisca
 
programa dia de las madres 10 de mayo para evento
programa dia de las madres 10 de mayo  para eventoprograma dia de las madres 10 de mayo  para evento
programa dia de las madres 10 de mayo para evento
 
Planificacion Anual 4to Grado Educacion Primaria 2024 Ccesa007.pdf
Planificacion Anual 4to Grado Educacion Primaria   2024   Ccesa007.pdfPlanificacion Anual 4to Grado Educacion Primaria   2024   Ccesa007.pdf
Planificacion Anual 4to Grado Educacion Primaria 2024 Ccesa007.pdf
 
UNIDAD DPCC. 2DO. DE SECUNDARIA DEL 2024
UNIDAD DPCC. 2DO. DE  SECUNDARIA DEL 2024UNIDAD DPCC. 2DO. DE  SECUNDARIA DEL 2024
UNIDAD DPCC. 2DO. DE SECUNDARIA DEL 2024
 
Presentacion Metodología de Enseñanza Multigrado
Presentacion Metodología de Enseñanza MultigradoPresentacion Metodología de Enseñanza Multigrado
Presentacion Metodología de Enseñanza Multigrado
 
TECNOLOGÍA FARMACEUTICA OPERACIONES UNITARIAS.pptx
TECNOLOGÍA FARMACEUTICA OPERACIONES UNITARIAS.pptxTECNOLOGÍA FARMACEUTICA OPERACIONES UNITARIAS.pptx
TECNOLOGÍA FARMACEUTICA OPERACIONES UNITARIAS.pptx
 

Logica Matemática

  • 1. Capítulo 1 Lógica matemática 1.1. Formas proposicionales La lógica matemática se ocupa del análisis de las proposiciones y demostraciones del razona- miento lógico, proporciona ideas claras y precisas sobre la naturaleza de la conclusión deductiva, desarrolla el pensamiento funcional y hace una contribución esencial al desarrollo del pensamiento científico y creador. Esto se manifiesta, por ejemplo, en la correcta comprensión de las estructuras lógicas y las tareas formales, en el reconocimiento de las semejanzas de los diferentes fenómenos lógicos, en la aplicación de las leyes y reglas lógicas y en la pretensión de claridad, sencillez y economía en la expresión lingüística. Una de las propiedades de la forma de expresión matemática, es la de representar los objetos, las imágenes mentales, los vínculos y las relaciones mediante símbolos (signos), y combinarlos entre sí. Definición 1.1 Constante Una constante es un signo que tiene una determinada significación fija. Es decir; una constante tiene, en todo el desarrollo de una investigación o en la solución de una tarea, siempre la misma significación. Definición 1.2 Variable Una variable es un signo que representa cualquier elemento de un dominio básico previamente establecido. Esto quiere decir que una variable se puede sustituir por el signo de cualquier elemento del dominio básico. Entonces se habla de la sustitución de la variable, o de la interpretación de la variable. Definición 1.3 Término Por término entendemos las constantes, las variables y sus combinaciones mediante los signos de operación y los signos técnicos. Los términos son, por tanto, las denominaciones de los objetos matemáticos o las combinacio- nes de signos donde se presentan variables, constantes y signos de operaciones, y que mediante la interpretación de las variables se omiten en las designaciones de los objetos matemáticos. El ob- jeto matemático, identificado como un término, y en cuya denominación se omite este calificativo 1
  • 2. CAPÍTULO 1. LÓGICA MATEMÁTICA 2 después de la interpretación de las variables, se conoce como valor del término. Las proposiciones son estructuras lingüísticas cuyo valor de verdad es, o verdadero o falso. La lógica clásica, a través de sus axiomas y principios, ha hecho algunas consideraciones sobre el con- tenido de verdad de una proposición. El principio de la bivalencia expresa: Toda proposición o es falsa o es verdadera. De este principio se pueden deducir dos teoremas. 1. El teorema de la tercera posibilidad excluida, expresa: Toda proposición es falsa o verdadera. 2. El teorema de la contradicción excluida, expresa: Ninguna proposición es falsa y verdadera al mismo tiempo. En las observaciones posteriores veremos que los dos teoremas, considerados en conjunto, ex- presan exactamente lo mismo que el principio de la bivalencia. Por consiguiente, se puede proceder a la inversa; es decir deducir el teorema de la bivalencia a partir del principio de la tercera posibi- lidad excluida y del principio de la contradicción excluida. A cada proposición se le hace corresponder un valor de verdad, o falso F o verdadero V. Es por esta razón que también se habla de una lógica bivalente. La asignación de los valores de verdad F o V de una proposición, no es tan sencillo de determinar. Aunque en el principio de la bivalencia se expresa claramente que una proposición es falsa o verdadera, no se puede decir inmediatamente si cada proposición es falsa o verdadera. En matemáticas existen actualmente muchas proposiciones que hasta el momento no han podido ser demostradas, concebida, la demostración, como una aseveración de la verdad, a continuación se dan dos ejemplos de este tipo de proposiciones. Ejemplo 1.1 La proposición: ¨Todo número par que sea mayor que 4, se puede representar como la suma de dos números primos, excepto el 2¨, existe desde el año 1742. Hasta el momento no se ha podido demostrar si es una proposición falsa o verdadera. (Suposición de Goldbach). Definición 1.4 Forma proposicional Una estructura lingüística que contiene por lo menos una variable libre, se convierte en una pro- posición, cuando se sustituyen todas las variables por símbolos, que denotan objetos del dominio básico, recibe el nombre de forma proposicional. Ejemplo 1.2 8 + x <12 con x ∈ N no representa evidentemente ninguna proposición. Esta sucesión de signos no es ni falsa ni verdadera. Mediante las sustituciones de la variable x podemos formar proposiciones falsas y verdaderas. Así, con las sustituciones 0, 1, 2, 3 obtenemos siempre proposiciones verdaderas, y cualquier otra sustitución dará lugar a proposiciones falsas. En es- te caso, encontramos una expresión lingüística especial que no es una proposición, pero que, sin embargo, se convierte en una proposición mediante la sustitución de la variable. A las expresiones matemáticas de este tipo se las denomina formas proposicionales. Las variables en tales expresiones se denominan variables libres. Las formas proposicionales surgen cuando entre los términos que contienen variables se coloca un determinado signo de relación. De forma análoga al convenio establecido para la notación de términos, denotamos una forma proposicional con P(x1, x2, ..., xn). Todos los elementos cuyos símbolos convierten una forma proposicional en una proposición, constituyen el conjunto solución de esta forma proposicional. El conjunto solución comprende solamente aquellos elementos del dominio básico cuyos símbolos convierten una forma proposicional en una proposición verdadera. Las formas proposicionales se pueden clasificar en la forma siguiente: aquellas formas proposicionales que mediante una sustitución por lo menos, se
  • 3. CAPÍTULO 1. LÓGICA MATEMÁTICA 3 pueden transformar en una proposición verdadera, se denominan interpretables. Todas las demás se denominan no interpretables. Entre las interpretables se destacan las formas proposicionales de validez general, que son aquellas que al hacer cualquier sustitución por los elementos del dominio básico se convierten en una proposición verdadera. Ejemplo 1.3 (x + y)2 = x2 + 2xy + y2 ∀ x, y ∈ R. En toda sustitución de x e y por elementos del dominio básico se obtiene una proposición verdadera. Este ejemplo es, por tanto, una forma proposicional de validez general en el conjunto de los números reales. El conjunto solución es el conjunto de todos los pares (x, y) donde x e y son elementos de un dominio básico; luego, en este caso coincide con el conjunto base de solución. Este ejemplo trata entonces de una identidad. Ejemplo 1.4 Sea (x + y)2 = x2 + y2 ∀ x, y ∈ R. (x + y)2 x2 + y2 Valor de verdad 0 0 V 9 9 V 36 36 V 9 5 F 361 193 F La presente tabla muestra que a partir de esta forma proposicional se pueden obtener propo- siciones falsas y verdaderas. El conjunto solución es, un subconjunto propio del conjunto base de solución. El conjunto solución consta, de los pares ordenados de elementos del dominio básico. Este ejemplo trata entonces de una neutralidad. Ejemplo 1.5 x2 − 5x + 10 = 0 x ∈ R. En el dominio básico no hay elementos que satisfagan esta forma proposicional, es decir, toda sustitución la convierte en una proposición falsa. Este ejemplo trata por consiguiente de una contradicción. 1.1.1. Operaciones entre proposiciones lógicas En esta sección trataremos exclusivamente las proposiciones y las formas proposicionales. Pri- meramente, introduciremos algunas combinaciones de proposiciones, mediante las cuales a su vez se obtienen otras proposiciones. Después obtendremos mediante definiciones las funciones proposicio- nales y más tarde las funciones veritativas. En todas las operaciones con proposiciones señalaremos el proceso de abstracción circunstancia - proposiciones - valores de verdad. Definición 1.5 Proposición Denominaremos proposición a una frase narrativa que puede calificarse como verdadera o falsa, pero no ambas al mismo tiempo. Los valores verdadero y falso mencionados en la definición se denominan valores de certeza o valores de verdad. Así cuando una proposición se considere verdadera o falsa diremos que dicha proposición tiene valor de certeza verdadero o falso. Ejemplo 1.6 Las siguientes frases son proposiciones: - La tierra es plana. - 547 es un número primo. - Los números irracionales son complejos. - Los números complejos son un subconjunto de los reales. - La Escuela Politécnica del Ejercito es un instituto de educación superior. - No es verdadero que 3 sea un entero par o 7 un primo.
  • 4. CAPÍTULO 1. LÓGICA MATEMÁTICA 4 - 2n = n2 para alguna n ∈ N. - 289301 + 1 es un número primo. - Si un árbol tiene n vértices, entonces tiene exactamente n - 1 aristas. - 2n + n es un número primo para una infinidad de n. - Todo entero par mayor que 4 es la suma de dos números primos. - Las matemáticas son divertidas. - Los árboles son más interesantes que las matrices. Ejemplo 1.7 Las siguientes frases no son proposiciones: - ¿Porqué es importante la lógica proposicional? - 323789 ext 205 - ¿Porqué es importante la inducción? - x - y = y - x. Es importante hacer notar que el valor de verdad de una proposición no es trabajo ni parte de la lógica aquí tratada, por tal razón dichos valores los supondremos ya asignados. Todas las proposiciones constituyen una clase que, a su vez, se descompone en dos subclases, en la clase de las proposiciones verdaderas V y en la clase de las proposiciones falsas F. La verdad o falsedad de las proposiciones no puede ser demostrada inmediatamente en todos los casos, pero, para toda proposición, independientemente de que aún no haya sido comprobada ni refutada, solo cabe una de las dos posibilidades, es verdadera V o es falsa F. El proceso de negación lo denominamos operación lógica de un lugar. Los enlaces de dos proposiciones, como resultado de los cuales se obtiene una proposición única se denominan operaciones lógicas de dos lugares. Definición 1.6 Función proposicional de n-lugares Cuando a cada n-uplo de proposiciones se le hace corresponder unívocamente una proposición, esta correspondencia se denomina función proposicional de n-lugares. Se entiende por n-uplo, un conjunto de n elementos dependientes del orden, en este caso pro- posiciones. De todas las funciones proposicionales, las llamadas funciones proposicionales clásicas tienen una gran importancia por las razones siguientes: a) porque las restantes funciones se pueden representar en términos de estas. b) porque en la lógica formal tradicional se han tratado especialmente las cinco siguientes fun- ciones: Nombre Argumento Funciones proposicionales Número de lugares Negación P No P Uno Conjunción P, Q P y Q Dos Disyunción P, Q P o Q Dos Implicación P, Q Si P, entonces Q Dos Equivalencia P, Q P exactamente cuando Q Dos En estas funciones proposicionales el valor de verdad de la proposición resultante depende so- lamente de los valores de verdad de los argumentos correspondientes, y no de su contenido, y se denominan funciones proposicionales extensionales. Además de las funciones proposicionales clási- cas existen otras funciones proposicionales que son extensionales. En el transcurso de las observaciones hemos hecho abstracción del contenido concreto de las proposiciones o de los enlaces de proposiciones y, alcanzado las etapas de abstracción de las fun- ciones proposicionales.
  • 5. CAPÍTULO 1. LÓGICA MATEMÁTICA 5 Continuamos el proceso de abstracción sobre la base de las afirmaciones ya hechas. Si también realizamos el paso de transición de las proposiciones a los valores de verdad, entonces obtenemos las funciones veritativas correspondientes a las funciones proposicionales. Definición 1.7 Función veritativa de n-lugares Cuando a cada n-uplo de valores de verdad se le hace corresponder unívocamente un valor de verdad, entonces esta correspondencia recibe el nombre de función veritativa de n lugares. Las funciones proposicionales y las funciones veritativas pertenecen a diferentes niveles de abs- tracción. Por este motivo es conveniente introducir otros símbolos para las funciones veritativas. Con la negación de una proposición queremos expresar la idea de que esto no se refiere a la cir- cunstancia que a ella corresponde. Cuando negamos una proposición P, entonces obtenemos otra proposición ¬P, es decir, la negación de P. A través de esta operación obtenemos una proposición cuyo valor de verdad es contrario al valor de verdad de P. Frecuentemente en el lenguaje común, una negación se expresa mediante prefijos que indican negación o mediante adverbios de negación. Por ejemplo, se dice en lugar de no regular, irregular, y en lugar de no un, simplemente ningún, etc. Al formular ciertas negaciones pueden surgir con mucha facilidad algunas confusiones, cuando simplemente se expresa la negación mediante antóni- mos o contrarios. Por ejemplo, negro y blanco, pequeño y grande, positivo y negativo, orden y caos son, en cierto sentido, contrarios que no pueden ser expresados a través de una negación. Aquí definiremos la negación como una función veritativa, aunque la denominación de negación se utiliza también para la función proposicional ¬P y para la operación negación. Definición 1.8 Negación Se denomina negación a la función veritativa de un lugar, cuyos valores se fijan de la manera siguiente: P ¬ P V F F V La negación corresponde a la función proposicional de un lugar ¬P. La afirmación ¬P es verda- dera cuando la proposición P es falsa, y ¬P es falsa cuando la proposición P es verdadera. Mediante la negación de una proposición P se obtiene una nueva proposición ¬P cuyo valor de verdad es opuesto al valor de verdad de P. Ejemplo 1.8 Sea P: ¨Los billetes de $ 5000 contienen la efigie de Rumiñahui¨. La negación de P es la proposición: ¬P: ¨Los billetes de $ 5000 no contienen la efigie de Rumiñahui¨. Dadas las proposiciones P, Q consideremos la construcción de proposiciones de la forma (P y Q). Ejemplo 1.9 En la búsqueda de un profesor la ESPE publica el siguiente aviso: ¨Se solicita profesor con especialidad en Pedagogía y Álgebra¨. Se presentan cuatro candidatos A, B, C y D con las características siguientes: A: Tiene ambas especialidades (V, V) B: Sólo tiene la especialidad de Pedagogía (V, F) C: Sólo tiene la especialidad de Álgebra (F, V)
  • 6. CAPÍTULO 1. LÓGICA MATEMÁTICA 6 D: Sólo tiene la especialidad de Química (F, F) Como podemos darnos cuenta, puesto que A cumplió con los requisitos, entonces A es el ganador del empleo. Definición 1.9 Conjunción Se denomina conjunción o producto lógico de las proposiciones P y Q, dadas en este orden, a la función veritativa de dos lugares que se obtiene enunciando Q luego de enunciar P, unidas ambas por la palabra ¨y¨, cuyos valores de verdad se fijan de la siguiente manera: P Q P ∧ Q V V V V F F F V F F F F El enlace de proposiciones P ∧ Q es verdadera cuando ambas proposiciones P, Q son verda- deras. Una conjunción cuyo valor de verdad es V, expresa que las circunstancias que se reflejan a través de las proposiciones parciales, existen en conjunto. Pero, cuando una conjunción tiene el valor de verdad F, esto significa que, por lo menos, una de sus proposiciones parciales no refleja co- rrectamente una circunstancia. En las explicaciones posteriores, consideraremos el enlace de varias proposiciones como una conjunción si aparece la expresión y/o sus sinónimos. Ejemplo 1.10 Sean las proposiciones: P: ¨2 es un divisor de 10¨ Q: ¨5 es un divisor de 10¨ La conjunción de P y Q es la siguiente proposición: P ∧ Q: ¨2 es un divisor de 10, pero también 5 es un divisor de 10¨ Por lo tanto la proposición P ∧ Q es verdadera. Estudiaremos ahora proposiciones de la forma (P o Q) y (o P o Q). Ejemplo 1.11 Consideremos ahora el siguiente aviso: ¨Se solicita profesor con especialidad de Pedagogía o Álgebra¨ Se presentan cuatro candidatos A, B, C y D con las características siguientes: A: Tiene ambas especialidades (V, V) B: Sólo tiene la especialidad de Pedagogía (V, F) C: Sólo tiene la especialidad de Algebra (F, V) D: Sólo tiene la especialidad de Química (F, F) En este caso sólo D no podrá ser seleccionado. El punto central de esta parte lo constituye el uso de la palabra ¨o¨, la cual puede ser utilizada en un sentido exclusivista ¨o ... o ...¨ o no exclusivista. Por este motivo procederemos en dos pasos intermedios. Definición 1.10 Disyunción Se denomina disyunción a la función veritativa bivalente cuyos valores se fijan de la manera si- guiente: P Q P ∨ Q V V V V F V F V V F F F
  • 7. CAPÍTULO 1. LÓGICA MATEMÁTICA 7 La disyunción corresponde a la función proposicional bivalente P o Q. Según la definición anterior P o Q es verdadera cuando, como mínimo, una de las proposiciones enlazadas es verdadera. En discusiones posteriores, el enlace de varias proposiciones con ¨o¨ recibe el nombre de disyunción. Entonces, un enlace de proposiciones de este tipo representa una proposición verdadera cuando todas las proposiciones enlazadas son verdaderas. Ejemplo 1.12 - 2 · 3 = 6 ó 3 + 2 = 5 - 75 % de 45 m es 135/4 m, ó 33,75 m. A causa de la extensionalidad de las funciones proposicionales las proposiciones enlazadas pue- den o no, tener relaciones de contenido entre sí. La abstracción hecha de las relaciones de contenido entre las proposiciones enlazadas es necesaria para poder fundamentar la relación lógica. Mediante la definición anterior se ha determinado el sentido de la palabra ¨o¨. Para nosotros son de gran interés aún los valores de verdad de las proposiciones parciales. En otros enlaces se procederá de forma similar. Definición 1.11 Alternativa Se denomina alternativa a la función veritativa de dos lugares cuyos valores se fijan de la manera siguiente: P Q P ∨ Q V V V V F V F V V F F F La alternativa corresponde a la función proposicional de dos lugares ¨o P o Q¨ es verdadero cuando una de las dos proposiciones es verdadera. Es falso cuando ambas proposiciones son verda- deras o falsas. La alternativa es igualmente extensional. En las explicaciones que demos posteriormente, un enlace de más de dos proposiciones con ¨o ... o¨ recibe el nombre de alternativa. En el uso diario del lenguaje se dice frecuentemente ¨o¨ en lugar de ¨o ... o¨, actuando esta palabra, en tales casos, de forma excluyente. Cuando en el lenguaje familiar corriente se habla de una disyunción, se hace referente a la alternativa que hemos definido. Estos hechos hay que tenerlos siempre en cuenta. Ejemplo 1.13 - La suma de los siete primeros números naturales es o par o impar. - 1969 es o un número primo o divisible por 9. Un verdadero enlace mediante la alternativa de ambas proposiciones refleja que de dos circuns- tancias posibles existe exactamente una. Para evitar las confusiones se debe utilizar, en tales casos, siempre ¨o ... o¨. ¨o¨ puede usarse en el lenguaje común pero con otro sentido, cuando se quiere decir que las dos circunstancias enlazadas entre sí no pueden existir en conjunto. Como máximo, esto puede referirse a una de ellas. El conocimiento de estas distintas interpretaciones de ¨o¨ en el lenguaje común es muy im- portante para la conclusión lógica y, además, nos motiva a velar por la exactitud de nuestras formulaciones. A continuación formularemos, mediante la disyunción, algunos teoremas importantes de la lógica de las proposiciones.
  • 8. CAPÍTULO 1. LÓGICA MATEMÁTICA 8 Teorema 1.1 Toda proposición es verdadera o falsa. Este teorema se denomina, teorema del tercero excluido. Esto podemos representarlo mediante la función proposicional ¨P o ¬P¨. Esta función proposicional es una identidad, porque para cualquier argumento P, siempre obtenemos una proposición verdadera. Teorema 1.2 Toda proposición es o verdadera o falsa. Este teorema se denomina, principio de la bivalencia. Este principio expresa que entre una proposición y su negación no hay una tercera posibilidad, y que una proposición no puede ser simultáneamente verdadera y falsa. Proposiciones como P ∧ Q y P ∨ Q que resultan de combinar otras proposiciones reciben el nombre de proposiciones compuestas. Es posible una proposición compuesta G ∼= G(P1, P2, ..., Pn) sea verdadera sin importar qué asignaciones de verdad se hayan hecho a las proposiciones P1, P2, ..., Pn. Ejemplo 1.14 En el Instituto de Ciencias Básicas existe el siguiente reglamento: ¨Para que un estudiante pueda tomar materias de avance de primer nivel, tiene que haber aprobado materias concatenadas de prepolitécnico¨. ¿En cuáles de los siguientes casos se viola el reglamento? A: Toma avances y aprobó materias concatenadas (V, V). B: Toma avances y no aprobó materias concatenadas (V, F). C: No toma avances pero aprobó materias concatenadas (F, V). D: Ni toma avances ni aprobó materias concatenadas (F, F). Un poco de reflexión nos conduce a aceptar que se viola el reglamento en el caso B. Definición 1.12 Implicación Se denomina implicación o condicional a la función veritativa de dos lugares cuyos valores de verdad se fijan de la manera siguiente: P Q P → Q V V V V F F F V V F F V La proposición P se denomina hipótesis o antecedente y la proposición Q, conclusión o conse- cuente. Considérese el problema de asignar un valor de verdad a la proposición implicación ¨si P, entonces Q¨. En efecto, si la hipótesis P es verdadera y la conclusión Q es también verdadera (esto es, la hipótesis y la conclusión son ambas verdaderas), entonces la proposición condicional ¨si P, entonces Q¨ debe ser verdadera. Por otra parte, si la hipótesis P es verdadera y la conclusión Q es falsa, entonces ¨si P, enton- ces Q¨ debe ser falsa. (No se debe deducir una conclusión falsa de una hipótesis verdadera). La definición normal dice que ¨si P, entonces Q¨ es verdadera en caso de que la hipótesis P sea falsa, sin considerar el valor de verdad de la conclusión Q. En las ulteriores explicaciones, al enlace de varias proposiciones con ¨si P, entonces Q¨ lo llamaremos implicación. En el lenguaje ordinario, la hipótesis y la conclusión en una proposición implicación están normalmente relacionadas, pero en lógica no se requiere que la hipótesis y la conclusión en una
  • 9. CAPÍTULO 1. LÓGICA MATEMÁTICA 9 proposición implicación se refieran al mismo tema. Tienen interés particular las proposiciones implicación verdaderas. Los teoremas de matemáti- cas con frecuencia se expresan como proposiciones implicación. Una demostración de un teorema de esta forma lo constituye la verificación de que la proposición implicación es verdadera. Sean P ∼= P(P1, P2, ..., Pn) y Q ∼= Q(P1, P2, ..., Pn) proposiciones compuestas y supóngase que P → Q es verdadera. Se sabe que si P es falsa, P → Q es verdadera, no importando si Q es verdadera o falsa. Por otra parte, si P es verdadera, Q también debe serlo, pues en caso contrario P → Q sería falsa. Ejemplo 1.15 Sean las siguientes proposiciones: P : Hoy es 30 de Febrero. Q : Entre 5 y 15 hay números primos. P → Q : Si hoy es 30 de febrero, entonces entre 5 y 15 hay números primos. Esta proposición compuesta es verdadera, ya que podemos decir que toda implicación, cuyo primer miembro sea falso tiene el valor de verdad V, sin tener en cuenta si el primero y el segundo miembros tienen relación de contenido o no. Ejemplo 1.16 La proposición compuesta ¨si entre 5 y 15 hay números primos, entonces entre 13 y 15 hay números primos¨, es falsa, ya que el primer miembro de esta implicación es verdadero y su segundo miembro es falso. Otra proposición compuesta de gran utilidad es P si y sólo si Q Este enunciado se interpreta como: (Si P, entonces Q) y (si Q, entonces P) Determínese el valor de verdad de la primera proposición. Supóngase que P y Q son ambas verdaderas. Entonces las dos proposiciones implicación de la segunda son verdaderas. Y como la conectiva ¨y¨ resulta verdadera para ambas verdaderas, se tiene que la segunda también lo es. Dado que la primera se interpreta como la segunda, se considera que la primera es verdadera cuando ambas P y Q lo son. Si P y Q son falsas, nuevamente las dos proposiciones implicación de la segunda son verdaderas. En consecuencia, la segunda es verdadera. Por lo tanto, si ambas proposiciones P y Q son falsas, se considera que la primera es verdadera. Si P es falsa y Q es verdadera, entonces la segunda proposición implicación en la segunda es falsa. Ahora bien, cuando en la conectiva ¨y¨ uno de los valores es falso, el resultado es falso. Por consiguiente, se considera que la primera es falsa si P es falsa y Q es verdadera. Esto motiva la siguiente definición. Definición 1.13 Equivalencia Se denomina implicación o condicional a la función veritativa de dos lugares cuyos valores de verdad se fijan de la manera siguiente: P Q P ↔ Q V V V V F F F V F F F V
  • 10. CAPÍTULO 1. LÓGICA MATEMÁTICA 10 Otra forma de enunciar ¨P si y sólo si Q¨ es ¨P es una condición necesaria y suficiente para Q¨. Así mismo, ¨P si y sólo si Q¨ en ocasiones se escribe ¨P ssi Q¨. La proposición compuesta (Si P, entonces Q) y (si Q, entonces P), también podemos expresarla en símbología lógica de la siguiente manera P ↔ Q ∼= (P → Q) ∧ (Q → P) Ejemplo 1.17 Sean P: El número 2013 es divisible por 3. Q: La suma de las cifras básicas de 2013 es divisible por 3. P ↔ Q: El número 2013 es divisible por 3 cuando la sumas de sus cifras básicas es divisible por 3. Esta proposición es verdadera, ya que ambos enlaces son verdaderos. Ejemplo 1.18 Si P → Q es una proposición implicación, entonces denominamos: Q → P recíproca de P → Q. ¬P → ¬Q inversa de P → Q. ¬Q → ¬P contrapositiva de P → Q. Ejemplo 1.19 Implicación: Si 2272 es divisible por 4, entonces 2272 es un número par. Recíproca: Si 2272 es un número par, entonces 2272 es divisible por 4. Contrapositiva: Si 2272 no es un número par, entonces 2272 no es divisible por 4. Inversa: Si 2272 no es divisible por 4, entonces 2272 no es un número par. Ejemplo 1.20 Implicación: Si un triángulo es equilátero, entonces es isósceles. Recíproca: Si un triángulo es isósceles, entonces es equilátero. Contrapositiva: Si un triángulo no es isósceles, entonces tampoco es equilátero. Inversa: Si un triángulo no es equilátero, entonces tampoco es isósceles. 1.1.2. Tarea 1. Suponga que x, y, z ∈ R. Represente en forma simbólica los enunciados dados tomando: P: x < y, Q: y < z, R: x < z a) (x ≥ y e y < z) o x ≥ z; b) No es cierto que (x < y e y < z); c) x < y o no es verdad que (y < z y x < z); d) (No es verdad que (x < y y (x < z o y < z))) o (x ≥ y y x < z). Resp: a) ; b) ; c) ; d) . 2. Sean P, Q, R las proposiciones: P: Está lloviendo, Q: El Sol está brillando, R: Hay nubes en el cielo. Traduzca la siguiente notación lógica, utilizando P, Q, R y conectivos lógicos. a) Está lloviendo y el Sol está brillando; b) Si está lloviendo, entonces hay nubes en el cielo; c) Si no está lloviendo, entonces el Sol no está brillando y hay nubes en el cielo; d) El Sol está brillando si y sólo si no está lloviendo; e) Si no hay nubes en el cielo, entonces el Sol está brillando. Resp: a) ; b) ; c) ; d) ; e) .
  • 11. CAPÍTULO 1. LÓGICA MATEMÁTICA 11 3. Sean P, Q, R como en el ejercicio anterior. Traduzca lo siguiente a oraciones en español: a) (P ∧ Q) → R; b) (P → R) → Q; c) ¬P ↔ (Q ∨ R); d) ¬(P ↔ (Q ∨ R)); e) ¬(P ∨ Q) ∧ R. Resp: a) ; b) ; c) ; d) ; e) . 4. Sean p : tengo un loro y q : tengo un gato, escribir en lenguaje corriente y luego simplificar ¬(¬p ∨ ¬(¬q)) ∧ ¬(¬p) Resp: p ∧ (¬q): tengo un loro y no tengo un gato. 5. A un blanco se han efectuado tres tiros. Sea Pi la proposición ¨el blanco ha sido batido por el i-ésimo tiro¨, i = 1, 2, 3. ¿Qué significan las siguientes proposiciones: a) P1 ∨ P2 ∨ P3; b) P1 ∧ P2 ∧ P3; c) (¬P1 ∨ ¬P2) ∧ P3? ¿Cuáles de estas tres proposiciones son verdaderas si P3 es verdadera y P1 y P2, falsas? Resp: a) ; b) ; c) . 6. ¿Cuáles de las siguientes expresiones son proposiciones? Proporcione los valores de verdad de las proposiciones: a) x2 = x para toda x ∈ R; b) x2 = x para alguna x ∈ R; c) x2 = x; d) x2 = x para exactamente una x ∈ R; e) xy = xz implica y = z; f) xy = xz implica y = z para toda x, y, z ∈ R. Resp: a) ; b) ; c) ; d) ; e) ; f) . 7. Considere la frase ambigua x2 = y2 implica x = y para todo x, y: a) Transforme esta frase en una proposición no ambigua cuyo valor de verdad sea verdadero; b) Transforme esta frase en una proposición no ambigua cuyo valor de verdad sea falso. Resp: a) ; b) . 8. Formule verbalmente las expresiones simbólicas contenidas en los siguientes literales, uti- lizando las proposiciones: P : Hoy es lunes, Q : Está lloviendo, R : Hace calor. a) ¬P ∧ (Q ∨ R); b) ¬(P ∨ Q) ∧ R; c) (P ∧ (Q ∨ R)) ∧ (R ∨ (Q ∨ P)); d) (P ∨ (¬P ∧ ¬(Q ∨ R))) ∧ (P ∨ ¬(R ∨ Q)); e) ¬P → (Q ∨ R); f) ¬(P ∨ Q) ↔ R; g) (P ∧ (Q ∨ R)) → (R ∨ (Q ∨ P)); h) (P ∨ (¬P ∧ ¬(Q ∨ R))) → (P ∨ ¬(R ∨ Q)). Resp: a) ; b) ; c) ; d) ; e) ; f) ; g) ; h) . 9. En los siguientes literales, represente cada proposición en la forma de una proposición condicional: a) Para todo número x ∈ R, |x| < 2 siempre que 0 < x < 2; b) Una condición suficiente para que una función f sea integrable es que f sea continua. Resp: a) ; b) . 10. Enuncie la recíproca, la inversa y la contrapositiva de cada uno de los literales del ejercicio anterior. Resp: . 11. Proporcione las recíprocas, las inversas y las contrapositivas de las siguientes proposiciones: a) Si soy listo entonces soy rico; b) Si x2 = x entonces x = 0 o x = 1; c) Si 2 + 2 = 4 entonces 2 + 4 = 8. Resp: a) ; b) ; c) .
  • 12. CAPÍTULO 1. LÓGICA MATEMÁTICA 12 12. a) Muestre que n = 3 es un contraejemplo de la afirmación n3 < 3n para toda n ∈ N; b) ¿Puede encontrar otros contraejemplos? Resp: a) ; b) . 13. a) Muestre que x = −1 es un contraejemplo de (x + 1)2 ≥ x2 para toda x ∈ R; b) Encuentre otro contraejemplo; c) ¿Puede servir de contraejemplo cualquier número no negativo? Explique su respuesta. Resp: a) ; b) ; c) . 14. Encuentre contraejemplos de las siguientes afirmaciones: a) 2n − 1 es primo para toda n ≥ 2; b) 2n + 3n es primo para toda n ∈ N; c) 2n + n es primo para todo entero impar positivo n. Resp: a) ; b) ; c) . 15. a) Proporcione un contraejemplo para: x > y implica x2 > y2 para toda x, y ∈ R. Su respuesta debe ser un par ordenado; b) ¿Cómo debe restringir x e y para que sea verdadera la proposición de la parte a)? Resp: a) ; b) . 16. Exprese en forma simbólica cada uno de los enunciados, suponiendo que x, y, z ∈ R y que P : x < y, Q : y < z, R : x < z : a) Si x < y, entonces y ≥ z; b) Si (x < y e y < z), entonces x < z; c) Si (x ≥ y e y < z), entonces x ≥ z; d) Si no es verdad que (x < z e y < z), entonces x ≥ z; e) x < y si y sólo si (y < z y x < z); f) Si es falso que (x < y y (ya sea x < y o y < z)), entonces (x ≥ y, entonces x < z). Resp: a) ; b) ; c) ; d) ; e) ; f) . 17. ¿Cuáles de las proposiciones P, Q, R deben ser verdaderas y cuáles falsas para que (¬(¬P ∨ P) ∧ Q) → R sea verdadera? Resp: . 18. Represente simbólicamente cada una de las proposiciones condicionales dadas a continua- ción. Escriba su recíproca, inversa y contrapositiva tanto con símbolos como con palabras. Determine también el valor de verdad para la proposición condicional, para su recíproca, inversa y para su contrapositiva: a) Si 4 < 6, entonces 9 > 12; b) Si 4 > 6, entonces 9 > 12; c) |1| < 3 si −3 < 1 < 3; d) |4| < 3 si −3 < 4 < 3. Resp: a) ; b) ; c) ; d) . 19. Proporcione la recíproca, inversa y contrapositiva de cada una de las siguientes proposi- ciones: a) Si x + y = 1 entonces x2 + y2 ≥ 1; b) Si 2 + 2 = 4 entonces 3 + 3 = 8. Resp: a) ; b) . 20. Considere la proposición: si x > 0 entonces x2 > 0 para x ∈ N: a) Proporcione la recíproca, inversa y contrapositiva de la proposición; b) ¿Cuál de las siguientes proposiciones es verdadera: la proposición original, la recíproca, la inversa o la contrapositiva? Resp: a) ; b) .
  • 13. CAPÍTULO 1. LÓGICA MATEMÁTICA 13 21. Determine los valores de verdad de las siguientes proposiciones compuestas: a) Si 2 + 2 = 4, entonces 2 + 4 = 8; b) Si 2 + 2 = 5, entonces 2 + 4 = 8; c) Si 2 + 2 = 4, entonces 2 + 4 = 6; d) Si 2 + 2 = 5, entonces 2 + 4 = 6; e) Si la tierra es plana, entonces Vicente Rocafuerte fue el primer presidente de Ecuador; f) Si la tierra es plana, entonces Sixto Durán-Ballen es presidente de Ecuador en el periódo 92 - 96; g) Si Sixto Durán-Ballen es presidente de Ecuador en el periódo 92 - 96, entonces la tierra es plana; h) Si Sixto Durán-Ballen es presidente de Ecuador en el periódo 92 - 96, entonces 2 + 2 = 4. Resp: a) ; b) ; c) ; d) ; e) ; f) ; g) ; h) . 22. Supóngase que sabemos que P → Q es falso. Proporcione los valores de verdad para: a) P ∧ Q; b) P ∨ Q; c) Q → P; d) P → Q; e) ¬P → ¬Q; f) ¬Q → ¬P; g) Q ∧ ¬P; h) P ∧ ¬Q; i) P ∨ Q; j) ¬(P ↔ Q). Resp: a) ; b) ; c) ; d) ; e) ; f) ; g) ; h) ; i) ; j) . 23. Un lógico le dijo a su hijo Si no terminas tu cena, te irás directo a dormir y no verás televisión. Terminó su cena y fue enviado directamente a la cama. Discútalo. Resp: . 24. A la pregunta de cuál de tres estudiantes estudiaba lógica fue obtenida una respuesta correcta: si la estudiaba el primero, también lo hacía el tercero, pero no era cierto que si la estudiaba el segundo lo hacía asímismo el tercero. ¿Quién estudiaba lógica? Resp: . 25. Luis, Carlos, Joe, Fred ocuparon en la olimpiada de matemáticas los cuatro primeros pues- tos. Cuando les preguntaron acerca de la distribución de los puestos, dieron las tres siguientes respuestas: a) Fred - primero, Carlos - segundo; b) Fred - segundo, Luis - tercero; c) Joe - segundo, Luis - cuarto. ¿Cómo se distribuyeron los puestos si en cada una de las respuestas sólo una de las afirma- ciones era verdadera? Resp: a) ; b) ; c) . 26. Determine cuál de cuatro estudiantes dio el examen si sabemos que: a) Si lo dio el primero, el segundo también; b) Si lo dio el segundo, el tercero también o bien el primero no lo dio; c) Si no lo dio el cuarto, lo dio el primero, pero el tercero no; d) Si el cuarto lo dio, el primero también. Resp: a) ; b) ; c) ; d) . 27. Para una expedición de ocho pretendientes A, B, C, D, E, F, G, H hay que elegir seis especialistas: biólogo, hidrólogo, sinóptico, radista, mecánico y médico. Las funciones del biólogo pueden ser realizadas por E y G, las del hidrólogo, B y F. Las del sinóptico, F y G, las del radista, C y D, las del mecánico, C y H, las del médico, A y D. Aunque algunos de los pretendientes tienen dos especialidades, en la expedición cada uno puede realizar sólo una función. ¿Quién y en calidad de qué ha de incluirse en la expedición si F no puede ir sin B, D sin H y sin C, C no puede ir simultáneamente con G, y A no puede ir junto con B? Resp: .
  • 14. CAPÍTULO 1. LÓGICA MATEMÁTICA 14 1.2. Construcción de tablas de verdad El enunciado G ∼= P → [(Q ∧ R) → Q] incluye tres proposiciones: P, Q y R, cada una puede ser verdadera o falsa de manera independiente. Existen en total 23 = 8 combinaciones posibles de los valores de verdad para P, Q y R y la tabla de verdad para G deberá dar el valor de verdad de G para cada uno de los casos. Definición 1.14 Combinaciones Si una proposición compuesta G consta de n enunciados, habrá 2n combinaciones de valores de verdad, es decir, n filas en la tabla de verdad de G. Una tabla que despliega todos los valores de verdad de una fórmula, para todas las posibles interpretaciones que pueda tener, se denomina tabla de verdad de la fórmula. Esta tabla puede construirse sistemáticamente de la siguiente manera: 1. Las primeras n columnas se encabezan con las variables proposicionales; y se construyen más columnas para las combinaciones parciales de enunciados y se culmina con el enunciado dado. 2. Bajo cada una de las primeras n columnas, se enlistan las 2n n-adas posibles de los valores de verdad de los componentes del enunciado G. Cada n-tupla se enlista en una fila separada. 3. Para cada fila se calculan sucesivamente los valores de verdad restantes. Ejemplo 1.21 Sea G ∼= (P → Q) → (¬P ∨ Q), construir la correspondiente tabla de verdad: P Q P → Q ¬ P ∨ Q G V V V V V V F F V F F V V V V F F V V F Ejemplo 1.22 Sea G ∼= [(P → Q) ∧ ¬Q] → ¬P), construir la correspondiente tabla de verdad: P Q P → Q (P → Q) ∧¬ Q G V V V F V V F F F V F V V V V F F F F V Ejemplo 1.23 Sea G ∼= [(P ∨ Q) ∧ ¬P] → Q), construir la correspondiente tabla de verdad: P Q P ∨ Q (P ∨ Q) ∧¬ P G V V V F V V F V F V F V V V V F F F F V Ejemplo 1.24 Sea G ∼= (P → Q) ↔ (¬Q → ¬P), construir la correspondiente tabla de verdad: P Q P → Q ¬ Q → ¬ P G V V V V V V F F F V F V V V V F F V V V
  • 15. CAPÍTULO 1. LÓGICA MATEMÁTICA 15 Ejemplo 1.25 Sea G ∼= (P ↔ Q) ↔ [(P → Q) ∧ (Q → P)], construir la correspondiente tabla de verdad: P Q P ↔ Q P → Q Q → P (P → Q) ∧ (Q → P) G V V V V V V V V F F F V F V F V F V F F V F F V V V V V 1.2.1. Operaciones con fórmulas lógicas y sus propiedades En el estudio de las funciones proposicionales hemos utilizado las variables P, Q, R, ... para designar las proposiciones. Estas variables podemos interpretarlas con elementos de un dominio básico, es decir, con proposiciones. Su dominio está formado solamente por dos elementos, los valores de verdad V y F. Las constantes en este caso las constituyen los conectores lógicos. Mediante el enlace lineal de las variables con valores de verdad P, Q, etc., y conectores, así como mediante la aplicación de los signos técnicos (paréntesis), podemos formar series de signos. Definición 1.15 Fórmula bien formada Una fórmula bien formada, se define dentro de la lógica proposicional en los siguientes términos recursivos: 1) Las variables P, Q, ... son fórmulas. 2) a) Si P es una fórmula, entonces ¬P también es una fórmula. b) Si P y Q son fórmulas entonces P ∨ Q, P ∧ Q, P → Q, P ↔ Q también son fórmulas. 3) Una serie de signos P, Q, ... es una fórmula solo cuando se trata de los casos 1 y 2. En la representación simbólica se interpretan los signos ¬, ∧, ∨, →, ↔, que reciben el nom- bre de conectores, como signos de funciones proposicionales y también como signos de funciones veritativas. A los literales tales como P, Q, R,... que son usados para denotar proposiciones se denominan fórmulas atómicas o átomos. No es difícil reconocer que expresiones como P →, P ∨ no son fórmulas. Cuando no exista confusión se suprimen los paréntesis asignando rangos decrecientes a los conectores proposicionales de la siguiente manera; ↔, →, ∧, ∨, ¬ de manera que al conector proposicional con mas alto rango se lo evalue al final. Ejemplo 1.26 1) P → Q ∧ R = P → (Q ∧ R); 2) P → Q ∧ ¬R ∨ S = P → Q ∧ (¬R ∨ S) = P → [Q ∧ (¬R ∨ S)]. Ahora vamos a establecer una relación entre los valores de verdad y las funciones veritativas por una parte y las expresiones, por otra. Las variables P, Q, ... las utilizamos ahora como variables del valor de verdad, y de igual forma los conectores proposicionales ¬, ∧, ∨, →, ↔ como signos de las funciones veritativas clásicas. Sobre la base de las afirmaciones hechas podemos indicar el correspondiente valor de verdad para cada interpretación de las variables P, Q, ... con los valores de verdad. En las expresiones complicadas de la lógica proposicional también es posible calcular de esta forma, en finitos pasos, los valores de verdad, al hacer las diferentes interpretaciones de las variables. Comparando las tablas de verdad podemos decidir si dos fórmulas G y H tienen la misma tabla de valores de verdad. Con esto también podemos mostrar si una fórmula formada a partir de G y H, es una identidad de la lógica proposicional. La igualdad de las tablas de valores de verdad y la identidad de la lógica proposicional, sin embargo, no son exactamente lo mismo. La igualdad
  • 16. CAPÍTULO 1. LÓGICA MATEMÁTICA 16 de la tabla de valores de verdad es una relación entre dos fórmulas; y la propiedad de ser una identidad es una peculiaridad de una fórmula. Cuando nos interesamos por la igualdad de la tabla de valores de verdad, entonces comparamos los valores de verdad de dos fórmulas en todas las sustituciones posibles. Cuando nos interesamos por la validez general de una fórmula, queremos establecer si esta determinada fórmula toma, en cada interpretación, el valor de verdad V. En este caso, se determina el valor de verdad de una nueva fórmula formada a partir de las fórmulas G y H en todas las sustituciones posibles. De las fórmulas con las mismas tablas de verdad, G y H, se pueden formar siempre identidades de la lógica proposicional, es decir, fórmulas de validez general. Teorema 1.3 Una fórmula doblemente negada tiene la misma tabla de valores de verdad que la correspondiente fórmula dada, es decir; ¬¬ P ∼= P es una identidad de la lógica proposicional. Demostración P ¬ P ¬¬ P V F V F V F Teorema 1.4 Para la conjunción, la disjunción y la equivalencia se cumplen la ley conmutativa y la ley asociativa con respecto a la igualdad de las tablas de valores de verdad. Para la implicación no se cumple ni la ley asociativa, ni la ley conmutativa. Demostración P Q P ∨ Q Q ∨ P P ∧ Q Q ∧ P P ↔ Q Q ↔ P V V V V V V V V V F V V F F F F F V V V F F F F F F F F F F V V Dado que G1 = (P ↔ Q) ↔ R y G2 = P ↔ (Q ↔ R), entonces P Q R (P ∨ Q) ∨ R P ∨ (Q ∨ R) (P ∧ Q) ∧ R P ∧ (Q ∧ R) G1 G2 V V V V V V V V V V V F V V F F F F V F V V V F F F F V F F V V F F V V F V V V V F F F F F V F V V F F V V F F V V V F F V V F F F F F F F F F En lógica las proposiciones idénticamente verdaderas o bien idénticamente falsas desempeñan importante papel. Las proposiciones idénticamente verdaderas son siempre verdaderas indepen- diente de si las proposiciones que las forman son verdaderas o falsas. Teorema 1.5 Para las proposiciones idénticamente verdaderas e idénticamente falsas, con todo P son ciertas las siguientes fórmulas: P ∨ ¬P ∼= V; P ∨ V ∼= V; P ∨ F ∼= P P ∧ ¬P ∼= F; P ∧ V ∼= P; P ∧ F ∼= F Demostración
  • 17. CAPÍTULO 1. LÓGICA MATEMÁTICA 17 P P ∨ ¬ Q P ∨ V P ∨ F P ∧ ¬ P P ∧ V P ∧ F V V V V F V F F V V F F F F Teorema 1.6 Las equivalencias siguientes P → Q ∼= ¬Q → ¬P; ¬P → Q ∼= ¬Q → P P → ¬Q ∼= Q → ¬P; ¬P → ¬Q ∼= Q → P son identidades de la lógica proposicional. Demostración P Q P → Q ¬ Q → ¬ P ¬ P → Q ¬ Q → P V V V V V V V F F F V V F V V V V V F F V V F F P → ¬ Q Q → ¬ P ¬ P → ¬ Q Q → P F F V V V V V V V V F F V V V V Teorema 1.7 Las equivalencias siguientes ¬(P ∨ Q) ∼= ¬P ∧ ¬Q; ¬(P ∧ Q) ∼= ¬P ∨ ¬Q (P ∨ Q) ∧ P ∼= P; (P ∧ Q) ∨ P ∼= P (P ∨ Q) ∧ Q ∼= Q; (P ∧ Q) ∨ Q ∼= Q P → Q ∼= ¬P ∨ Q; P ↔ Q ∼= (P → Q) ∧ (Q → P) son identidades de la lógica proposicional. Demostración P Q ¬(P ∨ Q) ¬ P ∧ ¬ Q ¬(P ∧ Q) ¬ P ∨¬ Q (P ∨ Q) ∧ P (P ∧ Q) ∨ P V V F F F F V V V F F F V V V V F V F F V V F F F F V V V V F F (P ∨ Q)∧ Q (P ∧ Q) ∨ Q P → Q ¬ P ∨ Q P ↔ Q (P → Q) ∧ (Q → P) V V V V V V F F F F F F V V V V F F F F V V V V Teorema 1.8 La conjunción es, con respecto a la disjunción en ambos lados, distributiva y viceversa, es decir, que las siguientes fórmulas son identidades de la lógica proposicional P ∧ (Q ∨ R) ∼= (P ∧ Q) ∨ (P ∧ R); (Q ∨ R) ∧ P ∼= (Q ∧ P) ∨ (R ∧ P) P ∨ (Q ∧ R) ∼= (P ∨ Q) ∧ (P ∨ R); (Q ∧ R) ∨ P ∼= (Q ∨ P) ∧ (R ∨ P) Demostración
  • 18. CAPÍTULO 1. LÓGICA MATEMÁTICA 18 P Q R P ∧ (Q ∨ R) (P ∧ Q) ∨ (P ∧ R) (Q ∨ R) ∧ P (Q ∧ P) ∨ (R ∧ P) V V V V V V V V V F V V V V V F V V V V V V F F F F F F F V V F F F F F V F F F F F F F V F F F F F F F F F F F P ∨ (Q ∧ R) (P ∨ Q) ∧ (P ∨ R) (Q ∧ R) ∨ P (Q ∨ P) ∧ (R ∨ P) V V V V V V V V V V V V V V V V V V V V F F F F F F F F F F F F Teorema 1.9 Conjuntamente con la distributividad se cumple que la implicación, con respecto a las demás funciones veritativas, es distributiva a la derecha, pero no distributiva a la izquierda, es decir, que las siguientes fórmulas son de validez general P → (Q ∧ R) ∼= (P → Q) ∧ (P → R); P → (Q ∨ R) ∼= (P → Q) ∨ (P → R) P → (Q → R) ∼= (P → Q) → (P → R); P → (Q ↔ R) ∼= (P → Q) ↔ (P → R) Demostración P Q R P → (Q ∧ R) (P → Q) ∧ (P → R) P → (Q ∨ R) (P → Q) ∨ (P → R) V V V V V V V V V F F F V V V F V F F V V V F F F F F F F V V V V V V F V F V V V V F F V V V V V F F F V V V V P → (Q → R) (P → Q) → (P → R) P → (Q ↔ R) (P → Q) ↔ (P → R) V V V V F F F F V V F F V V V V V V V V V V V V V V V V V V V V
  • 19. CAPÍTULO 1. LÓGICA MATEMÁTICA 19 Teorema 1.10 Si la conclusión, segundo miembro, de una implicación es igualmente una impli- cación, entonces las dos premisas (primeros miembros), se pueden unir formando una sola premisa P → (Q → R) ∼= (P ∧ Q) → R; (P ∧ Q) ↔ R ∼= (P → R) ∨ (Q → R) Demostración P Q R P → (Q → R) (P ∧ Q) → R (P ∧ Q) ↔ R (P → R) ∨ (Q → R) V V V V V V V V V F F F F F V F V V V V V V F F V V V V F V V V V V V F V F V V V V F F V V V V V F F F V V V V Ejemplo 1.27 Utilizando las leyes de la lógica proposicional, demostrar que: (P ∧ Q) ↔ (P ∨ Q) ∼= (P ∨ Q) → (P ∧ Q). Solución (P ∧ Q) ↔ (P ∨ Q) ∼= [(P ∧ Q) → (P ∨ Q)] ∧ [(P ∨ Q) → (P ∧ Q)] ∼= [¬(P ∧ Q) ∨ (P ∨ Q)] ∧ [(P ∨ Q)) → (P ∧ Q)] ∼= [(¬P ∨ ¬Q) ∨ (P ∨ Q)] ∧ [(P ∨ Q) → (P ∧ Q)] ∼= (¬P ∨ ¬Q ∨ P ∨ Q) ∧ [(P ∨ Q) → (P ∧ Q)] ∼= [(¬P ∨ P) ∨ (¬Q ∨ Q)] ∧ [(P ∨ Q) → (P ∧ Q)] ∼= (V ∨ V) ∧ [(P ∨ Q) → (P ∧ Q)] ∼= V ∧ [(P ∨ Q) → (P ∧ Q)] ∼= (P ∨ Q) → (P ∧ Q). Ejemplo 1.28 Utilizando las leyes de la lógica proposicional, demostrar que: [(P → Q) ∧ ¬P] → ¬Q ∼= Q → P. Solución [(P → Q) ∧ ¬P] → ¬Q ∼= ¬[(P → Q) ∧ ¬P] ∨ ¬Q ∼= ¬[(¬P ∨ Q) ∧ ¬P] ∨ ¬Q ∼= ¬(¬P) ∨ ¬Q ∼= P ∨ ¬Q ∼= Q → P. Ejemplo 1.29 Utilizando las leyes de la lógica proposicional, demostrar que: [(P → Q) ∧ (P → R)] → (Q → R) ∼= Q → (P ∨ R).
  • 20. CAPÍTULO 1. LÓGICA MATEMÁTICA 20 Solución [(P → Q) ∧ (P → R)] → (Q → R) ∼= ¬[(¬P ∨ Q) ∧ (¬P ∨ R)] ∨ (¬Q ∨ R) ∼= ¬(¬P ∨ Q) ∨ ¬(¬P ∨ R) ∨ (¬Q ∨ R) ∼= (P ∧ ¬Q) ∨ (P ∧ ¬R) ∨ ¬Q ∨ R ∼= ¬Q ∨ R ∨ (P ∧ ¬R) ∼= ¬Q ∨ [(R ∨ P) ∧ (R ∨ ¬R)] ∼= ¬Q ∨ [(R ∨ P) ∧ V] ∼= ¬Q ∨ (R ∨ P) ∼= Q → (P ∨ R). Ejemplo 1.30 Utilizando las leyes de la lógica proposicional, demostrar que: [(P → Q) → R] → [(Q → P) → R] ∼= (P ∧ ¬Q) → R. Solución [(P → Q) → R] → [(Q → P) → R] ∼= ¬[¬(¬P ∨ Q) ∨ R] ∨ [¬(¬Q ∨ P) ∨ R] ∼= ¬[(P ∧ ¬Q) ∨ R] ∨ [(Q ∧ ¬P) ∨ R] ∼= [¬(P ∧ ¬Q) ∧ ¬R] ∨ [(Q ∧ ¬P) ∨ R] ∼= [(¬P ∨ Q) ∧ ¬R] ∨ [(Q ∧ ¬P) ∨ R] ∼= [(¬P ∨ Q) ∧ (Q ∧ ¬P) ∨ R] ∧ [¬R ∨ (Q ∧ ¬P) ∨ R] ∼= [(¬P ∨ Q ∨ Q) ∧ (¬P ∨ Q ∨ ¬P)] ∨ R ∧ V ∼= [(¬P ∨ Q) ∧ (¬P ∨ Q)] ∨ R ∼= (¬P ∨ Q) ∨ R ∼= ¬(P ∧ ¬Q) ∨ R ∼= (P ∧ ¬Q) → R. Ejemplo 1.31 Utilizando las leyes de la lógica proposicional, demostrar que: [(P → Q) → P] → (P → Q) ∼= P → Q. Solución [(P → Q) → P] → (P → Q) ∼= ¬[¬(¬P ∨ Q) ∨ P] ∨ (¬P ∨ Q) ∼= ¬[(P ∧ ¬Q) ∨ P] ∨ (¬P ∨ Q) ∼= ¬P ∨ ¬P ∨ Q ∼= ¬P ∨ Q ∼= P → Q. 1.2.2. Tautologías y falacias Definición 1.16 Tautología Si una proposición compuesta es siempre verdadera bajo todas sus interpretaciones, independien- temente de los valores de verificación de sus componentes, decimos que la proposición compuesta es una tautología.
  • 21. CAPÍTULO 1. LÓGICA MATEMÁTICA 21 Es decir, a un enunciado que es verdadero para todos los valores posibles de sus variables proposicionales se le denomina tautología. Cuando se comprueba que una equivalencia es una tautología, significa que sus dos partes componentes son siempre o ambas verdaderas o ambas falsas, para cualesquier valores de las variables proposicionales. Por tanto los dos lados son sólo diferentes maneras de proponer el mismo enunciado y se dice que son logicamente equivalentes. Definición 1.17 Falacia Una fórmula G es una falacia, si ¬G es una tautología. Ejemplo 1.32 Utilizando una tabla de verdad, determinar si la fórmula G = (P → Q) → (¬P ∨ Q) es tautología. Solución P Q P → Q ¬ P ∨ Q (P → Q) → (¬ P ∨ Q) V V V V V V F F F V F V V V V F F V V V Por lo tanto G si es tautología. Ejemplo 1.33 Utilizando una tabla de verdad, determinar si la fórmula G = (Q → P) → (P → Q) es tautología. Solución P Q Q → P P → Q (Q → P) → (P → Q) V V V V V V F V F F F V F V V F F V V V Por lo tanto G no es tautología. Ejemplo 1.34 Utilizando una tabla de verdad, determinar si la fórmula G = (P → Q) ↔ (¬Q → ¬P) es tautología. Solución P Q P → Q ¬ Q → ¬ Q (P → Q) ↔ (¬ Q → ¬ P) V V V V V V F F F V F V V V V F F V V V Por lo tanto G si es tautología.
  • 22. CAPÍTULO 1. LÓGICA MATEMÁTICA 22 Ejemplo 1.35 Utilizando una tabla de verdad, determinar si la fórmula G = (P ↔ Q) ↔ [(P → Q) ∧ (Q → P)] es tautología. Solución P Q P ↔ Q (P → Q) ∧ (Q → P (P ↔ Q) ↔ [(P → Q) ∧ (Q → P)] V V V V V V F F F V F V F F V F F V V V Por lo tanto G si es tautología. Ejemplo 1.36 Utilizando una tabla de verdad, determinar si la fórmula G = [(P → Q) ∧ (Q → R)] → (P → R) es tautología. Solución P Q R (P → Q) ∧ (Q → P P → Q [(P → Q) ∧ (Q → R)] → (P → R) V V V V V V V V F F F V V F V F V V V F F V F V F V V V V V F V F F V V F F V V V V F F F V V V Por lo tanto G si es tautología. Ejemplo 1.37 Utilizando una tabla de verdad, determinar si la fórmula G = [P → (Q → R)] ↔ [(P → Q) → R] es tautología. Solución P Q R (P → (Q → R) (P → Q) → R [P → (Q → R)] ↔ [(P → Q) → R] V V V V V V V V F F F V V F V V V V V F F V V V F V V V V V F V F V F F F F V V V V F F F V F F Por lo tanto G no es tautología.
  • 23. CAPÍTULO 1. LÓGICA MATEMÁTICA 23 1.2.3. Tarea 1. Construya la tabla de verdad para cada una de las siguientes proposiciones: a) (P → Q) → [(P ∨ ¬ Q) → (P ∧ Q)]; b) [(P ∨ Q) ∧ R] → (P ∧ ¬Q); c) [(P ↔ Q) ∨ (P → R)] → (¬Q ∧ P)]; d) P ∨ P; e) (P ∨ Q) ∨ R; f) (P ∨ P) ∨ P; g) P ∨ Q; h) ¬(P ↔ Q). Resp: a) ; b) ; c) ; d) ; e) ; f) ; g) ; h) . 2. Utilizando las leyes de la lógica proposicional, demostrar que P ∨ Q ∼= (P ∨ Q) ∧ ¬(P ∧ Q) Resp: . 3. Utilizando las leyes de la lógica proposicional, demuestre o refute: a) P ∨ Q ∼= (P ∨ Q) ∧ ¬(P ∧ Q); b) P ∨ (Q → R) ∼= (P ∨ Q) → (P ∨ R); c) (P ∨ Q) ∨ R ∼= P ∨ (Q ∨ R). Resp: a) ; b) ; c) . 4. Simplifique las siguientes fórmulas y diga cuales son tautologías y cuales falacias: a) P ∨ (¬P ∧ ¬Q)] ∨ (P ∧ ¬Q); b) (P ∧ Q) ∨ [(R ∨ P) ∧ ¬Q]. Resp: a) ; b) . 5. Simplifique las siguientes fórmulas y diga cuales son tautologías y cuales falacias: a) (R ∧ Q) ∨ (P ∧ ¬Q ∧ R) ∨ (¬P ∧ ¬Q ∧ R); b) (P → Q) ∧ ¬(R → Q); c) (¬P ∨ Q) ∧ ¬Q ∧ [¬(R ∧ Q) → P]. Resp: a) ; b) ; c) . 6. Simplifique las siguientes fórmulas y diga cuales son tautologías y cuales falacias: a) (P ∧ Q) ∧ (R ∨ ¬S) ∧ (P → S); b) (¬P ∨ Q) ∧ (¬P → R) ∧ ¬R; c) (P ∧ Q) ∧ (P → R) ∧ (Q → S). Resp: a) ; b) ; c) . 7. Simplifique las siguientes fórmulas y diga cuales son tautologías y cuales falacias: a) (P ∨ Q) ∧ ¬Q ∧ (P → R); b) (P ∧ Q) ∧ (P → ¬R) ∧ (Q → ¬R); c) (P → ¬Q) ∧ Q ∧ (¬P → (R ∨ S)]. Resp: a) ; b) ; c) . 8. Simplifique las siguientes fórmulas y diga cuales son tautologías y cuales falacias: a) (P → S) ∧ (P ∧ Q) ∧ [(S ∧ R) → ¬T] ∧ (Q → R); b) ¬P ∧ (Q → P) ∧ [(¬Q ∨ R) → S]; c) (P ∧ ¬Q) ∧ (R → Q) ∧ (R ∨ S) ∧ [(S ∨ P) → T]. Resp: a) ; b) ; c) . 9. Simplifique las siguientes fórmulas y diga cuales son tautologías y cuales falacias: a) (P ↔ Q) ↔ [(P → Q) ∧ (Q → P)]; b) [(P → Q) ∧ (Q → R) ∧ P] → R; c) [P → (Q ∨ R)] ∧ (Q → ¬R) ∧ [(S → ¬R) ∧ P] → ¬S. Resp: a) ; b) ; c) . 1.3. Transformación de fórmulas La igualdad de los valores de verdad de dos proposiciones la hemos demostrado hasta ahora utilizando las tablas completas de valores de verdad. Con su ayuda pudimos decidir si una fórmula
  • 24. CAPÍTULO 1. LÓGICA MATEMÁTICA 24 dada es o no una identidad de la lógica proposicional. Por esta vía hemos conocido mumerosas fórmulas con las tablas de valores. Otras identidades, es decir; las leyes de la lógica proposicional, las obtenemos a partir de las fórmulas dadas y me- diante sustituciones o transformaciones en fórmulas equivalentes. En esta sección veremos cómo obtener equivalencias e implicaciones lógicas sin utilizar tablas de verdad. También explicaremos el significado de teorema y de demostración. Empezaremos con dos reglas útiles, que sin embargo deben manejarse con cuidado. Teorema 1.11 Si en una fórmula de validez general, es decir, en una identidad de la lógica proposicional, se sustituye una variable proposicional por una fórmula cualquiera en todos los lu- gares donde se presenta la fórmula correspondiente, entonces se obtiene nuevamente una fórmula de validez general. Teorema 1.12 Cuando en una fórmula G se sustituye una cierta subfórmula G1 por una fór- mula G2, que toma los mismos valores de verdad que G1, entonces la fórmula obtenida F tiene los mismos valores de verdad que la fórmula G. La fórmula G, una vez sustituida G1 debe sustituirse por G2 en todos los lugares donde esta se presenta. Ejemplo 1.38 Consideremos la proposición G ∼= [P ∧ (P → Q)] → Q que es una tautología. Si reemplazamos, cada vez que aparece P, por la proposición G1 ∼= Q → R obtenemos la tautología H ∼= [(Q → R) ∧ ((Q → R) → Q)] → Q. Si en cambio reemplazamos Q, cada vez que aparece, por G1, obtenemos la tautología H ∼= [P ∧ (P → (Q → R))] → (Q → R). Ejemplo 1.39 Consideremos la proposición G ∼= ¬[(P → Q) ∧ (P → R)] → [Q → (P → R)] que no es una tautología. Obtenemos proposiciones lógicamente equivalentes si reemplazamos P → Q por su equivalencia lógica ¬P ∨ Q o si reemplazamos una o las dos veces que aparece P → R por ¬P ∨ R. Podemos también reemplazar (P → Q) ∧ (P → R) por P → (Q ∧ R). De esta manera G es lógicamente equivalente a las siguientes proposiciones entre otras: ¬[(¬P ∨ Q) ∧ (P → R)] → [Q → (P → R)] ¬[(P → Q) ∧ (¬P ∨ R)] → [Q → (P → R)] ¬[(P → (Q ∧ R)] → [Q → (¬P ∨ R)]. Definición 1.18 Fórmula válida Una fórmula G es válida o constituye una tautología, si y sólo si es verdadera bajo todas las interpretaciones. En caso contrario la fórmula G es inválida.
  • 25. CAPÍTULO 1. LÓGICA MATEMÁTICA 25 Definición 1.19 Fórmula inconsistente Una fórmula G se denomina inconsistente o insatisfactible, si y sólo si es falsa bajo todas las interpretaciones. En caso contrario la fórmula G es consistente o satisfactible. De las definiciones anteriores, las observaciones siguientes son obvias: 1. Una fórmula es válida, si y sólo si su negación es inconsistente. 2. Una fórmula es inconsistente, si y sólo si su negación es válida. 3. Una fórmula es inválida, si y sólo si hay por lo menos una interpretación bajo la cual la fórmula es falsa. 4. Una fórmula es inconsistente, si y sólo si hay por lo menos una interpretación bajo la cual la fórmula es verdadera. 5. Si una fórmula es válida, entonces es consistente pero no viceversa. 6. Si una fórmula es inconsistente, entonces es inválida pero no viceversa. Ejemplo 1.40 Verificar la validez o inconsistencia de la fórmula: [(P → Q) ∧ (Q → R)] → (P → R) Solución P Q R (P → Q) ∧ (Q → R) P → R [(P → Q) ∧ (Q → R)] → (P → R) V V V V V V V V F F F V V F V F V V V F F V F V F V V V V V F V F F V V F F V V V V F F F V V V Por lo tanto G es una fórmula válida. Ejemplo 1.41 Verificar la validez o inconsistencia de la fórmula: [(P → (Q → R)] ↔ [(P → Q) → R] Solución P Q R (P → (Q → R) (P → Q) → R [(P → (Q → R)] ↔ [(P → Q) → R] V V V V V V V V F F F V V F V V V V V F F V V V F V V V V V F V F V F F F F V V V V F F F V F F Por lo tanto G no es una fórmula válida.
  • 26. CAPÍTULO 1. LÓGICA MATEMÁTICA 26 1.3.1. Formas normales En lógica matemática es muy importante el poder transformar fórmulas de una forma a otra, especialmente a las denominadas formas normales. Para lograr estas transformaciones de fórmulas, se utiliza el concepto de equivalencias de fórmulas. Definición 1.20 Fórmulas equivalentes Las fórmulas G y H son equivalentes si los valores de verdad de G y H son los mismos bajo todas las interpretaciones de estas fórmulas. Por supuesto que nuestro interés no se limita a estudiar una simple clasificación de los enun- ciados del lenguaje; pero tampoco intentamos internarnos en el fascinante mundo de la deducción lógica sin antes estar seguros de conocer y comprender algunos conceptos elementales. Las dos formas normales que nos interesa obtener y que son utilizadas en prueba mecánica de teoremas, son la forma normal conjuntiva y la forma normal disjuntiva. Definición 1.21 Forma normal conjuntiva Una fórmula G se dice que está en forma normal conjuntiva si y sólo si G tiene la forma G ∼= G1 ∧ G2 ∧ · · · ∧ Gn n ∈ N donde cada una de las fórmulas G1, G2, ..., Gn, se expresan como una conjunción de literales. Ejemplo 1.42 Expresar la fórmula G ∼= (Q → P) → (P → Q) en forma normal conjuntiva. Solución (Q → P) → (P → Q) ∼= ¬(Q → P) ∨ (P → Q) ∼= ¬(¬Q ∨ P) ∨ (P → Q) ∼= (Q ∧ ¬P) ∨ (P → Q) ∼= [Q ∨ (P → Q)] ∧ [¬P ∨ (P → Q)]. Ejemplo 1.43 Expresar la fórmula G ∼= (P ↔ Q) ↔ [(P → Q) ∧ (Q → P)] en forma normal conjuntiva. Solución (P ↔ Q) ↔ [(P → Q) ∧ (Q → P)] ∼= (P ↔ Q) ↔ (P ↔ Q) ∼= [(P ↔ Q) → (P ↔ Q)] ∧ [(P ↔ Q) → (P ↔ Q)]. Definición 1.22 Forma normal disjuntiva Una fórmula G se dice que está en forma normal disjuntiva si y sólo G si tiene la forma G ∼= G1 ∨ G2 ∨ · · · ∨ Gn n ∈ N donde cada una de las fórmulas G1, G2, ..., Gn, se expresan como una disjunción de literales.
  • 27. CAPÍTULO 1. LÓGICA MATEMÁTICA 27 Ejemplo 1.44 Expresar la fórmula G ∼= (Q → P) → (P → Q) en forma normal disjuntiva. Solución (Q → P) → (P → Q) ∼= ¬(Q → P) ∨ (P → Q) Ejemplo 1.45 Expresar la fórmula G ∼= (P ↔ Q) ↔ [(P → Q) ∧ (Q → P)] en forma normal conjuntiva. Solución (P ↔ Q) ↔ [(P → Q) ∧ (Q → P)] ∼= (P ↔ Q) ↔ (P ↔ Q) ∼= [(P ↔ Q) → (P ↔ Q)] ∧ [(P ↔ Q) → (P ↔ Q)] ∼= (P ↔ Q) → (P ↔ Q) ∼= ¬(P ↔ Q) ∨ (P ↔ Q) Un hecho que es muy importante anotar, es que cualquier fórmula de la lógica proposicional puede ser transformada a una de las formas normales, utilizando las leyes de la lógica proposicional. 1.3.2. Consecuencias lógicas Definición 1.23 Consecuencia lógica Dadas las fórmulas G1, G2, ..., Gn y una fórmula G, G se denomina consecuencia lógica de G1, G2, ..., Gn si y sólo si para cualquier interpretación en la cual G1 ∧ G2 ∧ · · · ∧ Gn es verdad, G también lo es G1, G2, ..., Gn se denominan axiomas de G. Teorema 1.13 Dadas las fórmulas G1, G2, ..., Gn y una fórmula G, G es una consecuencia lógica de G1, G2, ..., Gn si y sólo si la fórmula (G1 ∧ G2 ∧ · · · ∧ Gn) → G es válida. Demostración ⇒ Suponga que G es una consecuencia lógica de G1, G2, ..., Gn. Sea I una interpretación ar- bitraria. Si G1, G2, ..., Gn son verdaderos en I, entonces por definición de consecuencia lógica G es verdadero en I. Entonces (G1 ∧ G2 ∧ · · · ∧ Gn) → G es verdadero en I. Por otra parte, si G1, G2, ..., Gn son falsos en I, entonces (G1 ∧ G2 ∧ · · · ∧ Gn) → G es verdadero en I. Así, de- mostramos que (G1 ∧ G2 ∧ · · · ∧ Gn) → G es verdadero bajo cualquier interpretación. Esto es, (G1 ∧ G2 ∧ · · · ∧ Gn) → G es una fórmula válida. ⇐ Supongamos que (G1 ∧ G2 ∧ · · · ∧ Gn) → G es una fórmula válida. Para cualquier interpreta- ción I, si G1 ∧ G2 ∧ · · · ∧ Gn es verdadero en I, G debe ser verdadero en I. Por consiguiente G es una consecuencia lógica de G1, G2, ..., Gn. Ejemplo 1.46 Sean G1 P ∨ (¬Q → R) G2 ¬(P ∨ S) ∧ ¬R G Q Pruebe si G es consecuencia lógica de G1 y G2. Solución
  • 28. CAPÍTULO 1. LÓGICA MATEMÁTICA 28 Debemos probar que la fórmula {[P ∨ (¬Q → R)] ∧ [¬(P ∨ S) ∧ ¬R]} → Q, es verdadera o falsa, decir: {[P ∨ (¬Q → R)] ∧ [¬(P ∨ S) ∧ ¬R]} → Q ∼= ¬{[P ∨ (Q ∨ R)] ∧ [¬P ∨ ¬S ∧ ¬R]} ∨ Q ∼= ¬[(P ∨ Q ∨ R) ∧ (¬P ∨ ¬S ∧ ¬R)] ∨ Q ∼= (¬P ∧ ¬Q ∧ ¬R) ∨ (P ∨ S ∨ R) ∨ Q ∼= ¬(P ∨ Q ∨ R) ∨ (P ∨ Q ∨ R) ∨ S ∼= V. Lo cual indica que G es consecuencia lógica de G1 y G2. Ejemplo 1.47 Sean G1 P → Q) G2 Q → R G3 ¬R G ¬R Pruebe si G es consecuencia lógica de G1, G2 y G3. Solución Debemos probar que la fórmula [(P → Q) ∧ (Q → R) ∧ ¬R] → ¬R, es verdadera o falsa, decir: [(P → Q) ∧ (Q → R) ∧ ¬R] → ¬R ∼= ¬[(¬P ∨ Q) ∧ (¬Q ∨ R) ∧ ¬R] ∨ ¬R ∼= ¬(¬P ∨ Q) ∧ ¬(¬Q ∨ R) ∨ (R ∨ ¬R) ∼= V. Lo cual indica que G es consecuencia lógica de G1, G2 y G3. Teorema 1.14 Dadas las fórmulas G1, G2, ..., Gn y una fórmula G, G es una consecuencia lógica de G1, G2, ..., Gn si y sólo si la fórmula G1 ∧ G2 ∧ · · · ∧ Gn ∧ G es inconsistente. Demostración Por el teorema anterior, G es una consecuencia lógica de G1, G2, ..., Gn si y sólo si la fórmula (G1 ∧ G2 ∧ · · · ∧ Gn) → G es válida. Así, G es una consecuencia lógica de G1, G2, ..., Gn si y sólo si la negación de (G1 ∧ G2 ∧ · · · ∧ Gn) → G es inconsistente ¬[(G1 ∧ G2 ∧ · · · ∧ Gn) → G] ∼= ¬[¬(G1 ∧ G2 ∧ · · · ∧ Gn) ∨ G] ∼= ¬¬(G1 ∧ G2 ∧ · · · ∧ Gn) ∧ ¬G ∼= (G1 ∧ G2 ∧ · · · ∧ Gn) ∧ ¬G ∼= G1 ∧ G2 ∧ · · · ∧ Gn ∧ ¬G Por lo tanto, concluimos que el teorema es verdadero. Ejemplo 1.48 Sean G1 P ∨ (¬Q → R) G2 ¬(P ∨ S) ∧ ¬R G Q Pruebe si G es consecuencia lógica de G1 y G2. Solución
  • 29. CAPÍTULO 1. LÓGICA MATEMÁTICA 29 Debemos probar que la fórmula {[P ∨ (¬Q → R)] ∧ [¬(P ∨ S) ∧ ¬R]} ∧ ¬Q, es verdadera o falsa, decir: {[P ∨ (¬Q → R)] ∧ [¬(P ∨ S) ∧ ¬R]} ∧ ¬Q ∼= [(P ∨ Q ∨ R) ∧ (¬P ∧ ¬S ∧ ¬R)] ∧ ¬Q ∼= (P ∨ Q ∨ R) ∧ (¬P ∧ ¬Q ∧ ¬R) ∧ ¬S ∼= (P ∨ Q ∨ R) ∧ ¬(P ∨ Q ∨ R) ∧ ¬S ∼= F. Lo cual indica que G es consecuencia lógica de G1 y G2. Ejemplo 1.49 Sean G1 P → Q) G2 Q → R G2 ¬R G ¬R Pruebe si G es consecuencia lógica de G1, G2 y G3. Solución Debemos probar que la fórmula [P → Q) ∧ (Q → R) ∧ ¬R] ∧ R, es verdadera o falsa, decir: [P → Q) ∧ (Q → R) ∧ ¬R] ∧ R ∼= (P → Q) ∧ (Q → R) ∧ (¬R ∧ R) ∼= F. Lo cual indica que G es consecuencia lógica de G1, G2 y G3. 1.3.3. Tarea 1. Determine la validez o inconsistencia, luego transforme a una de sus formas normales las siguientes fórmulas:: a) [P ∨ (¬P ∧ ¬Q)] ∨ (P ∧ ¬Q); b) (P ∧ Q) ∨ [(R ∨ P) ∧ ¬Q]; c) (R ∧ Q) ∨ (P ∧ ¬Q ∧ R) ∨ (¬P ∧ ¬Q ∧ R); d) (¬P ∨ Q) ∧ ¬Q ∧ [¬(R ∧ Q) → P]. Resp: a) ; b) ; c) ; d) . 2. Determine la validez o inconsistencia, luego transforme a una de sus formas normales las siguientes fórmulas: a) (P ∧ Q) ∧ (R ∨ ¬S) ∧ (P → S); b) (¬P ∨ Q) ∧ (¬P → R) ∧ ¬R; c) (P ∧ Q) ∧ (P → R) ∧ (Q → S); d) (P ∨ Q) ∧ ¬Q ∧ (P → R); e) (P ∧ Q) ∧ (P → ¬R) ∧ (Q → ¬R); f) [(P → Q) ∧ (Q → R) ∧ P] → R. Resp: a) ; b) ; c) ; d) ; e) ; f) . 3. Determine la validez o inconsistencia, luego transforme a una de sus formas normales las siguientes fórmulas: a) (P → ¬Q) ∧ Q ∧ [¬P → (R ∨ S)]; b) (P → S) ∧ (P ∧ Q) ∧ [(S ∧ R) → ¬T] ∧ (Q → R); c) ¬P ∧ (Q → P) ∧ [(¬Q ∨ R) → S];
  • 30. CAPÍTULO 1. LÓGICA MATEMÁTICA 30 d) (P ∧ ¬Q) ∧ (R → Q) ∧ (R ∨ S) ∧ [(S ∨ P) → T]; e) (P ↔ Q) ↔ [(P → Q) ∧ (Q → P)]; f) [P → (Q ∨ R)] ∧ (Q → ¬R) ∧ [(S → ¬R) ∧ P] → ¬S. Resp: a) ; b) ; c) ; d) ; e) ; f) . 4. Decir cual de los siguientes enunciados son consecuencia lógica: a) G1 (P ∨ Q) → R) G2 S ∧ T G3 U → ¬L G4 P ∨ U G5 S → L G R b) G1 (P → Q) → R G2 ¬R ∨ S G3 ¬(P ∧ ¬Q) G4 (S ∨ T) → U G U c) G1 ¬(P ∨ Q) → (R ∨ S) G2 ¬(P ∨ Q) G3 ¬(R ∨ S) ∨ (T ∧ U) G T ∧ U Resp: a) ; b) ; c) . 5. Los alumnos son estudiosos o los estudiosos reprueban. Si los estudiosos reprueban, enton- ces los inteligentes son felices o los alumnos no son estudiosos. Los alumnos son estudiosos y los inteligentes no son felices. No es verdad que los inteligentes son felices. Los estudiantes no reprueban? Resp: . 6. Juego fútbol o estudio. Si paso el examen no estudio. Sucede que no voy a jugar fútbol. En consecuencia no pasé el examen. Resp: . 7. La lógica es fácil. Si el álgebra es hermosa, entonces la Lógica no es fácil o la Matemática es la reina de las ciencias. El Algebra es hermosa. En consecuencia, la Matemática es la reina de las ciencias. Resp: . 8. Ayer no fue miércoles o mañana no es martes. Hoy es jueves y ayer fue miércoles. Hoy es lunes si y sólo si mañana es martes. En consecuencia, hoy es lunes. Resp: .
  • 31. CAPÍTULO 1. LÓGICA MATEMÁTICA 31 9. Luis hará un viaje a Europa si logra terminar su carrera. Luis termina su carrera, y si hace un viaje a Europa, entonces no asiste a nuestra reunión anual. En consecuencia, Luis no asistirá a nuestra reunión anual. Resp: . 10. Si faltan ejercicios o encuentro premisas, entonces acabo la tarea. Si el libro está claro y no me falta creatividad, entonces encuentro premisas. No acabo la tarea. En consecuencia me falta creatividad o el libro no está claro. Resp: . 11. Si ganamos el campeonato, recibimos el premio. Si jugamos y ganamos el campeonato, recibiremos el premio. Jugaremos y ganaremos el campeonato. En consecuencia, recibiremos el premio. Resp: . 12. Repruebo el examen o sigo mis estudios. Si repruebo el examen, perderé la beca y me iré de la ciudad. No perderé la beca o no me iré de la ciudad. Luego, seguire estudiando. Resp: . 13. Los aviones son veloces o las diligencias respetan los semáforos. Si los hombres vuelan y las bicicletas no contaminan, entonces no es verdad que las diligencias respetan los semáforos. Los hombres vuelas y las bicicletas no contaminan. En conclusión, los aviones son veloces. Resp: . 1.4. Expresiones de la lógica de predicados El cálculo proposicional es una teoría de la lógica, completa y autónoma, pero totalmente inade- cuada para la mayor parte de las matemáticas. El problema reside en que el cálculo proposicional no permite el uso de un número infinito de proposiciones. Además, la notación es difícil para ma- nejar un gran número finito de proposiciones. Por ejemplo, con frecuencia encontramos una sucesión infinita de proposiciones P(x) con índi- ces en N. La afirmación informal ¨P(x) es verdadera para toda x¨ significa ¨P(0) es verdadera, P(1) es verdadera, P(2) es verdadera, etc.¨ El único simbolismo que podríamos utilizar, según el cálculo proposicional sería P(0)∧P(1)∧P(2)∧..., pero no es aceptable en el cálculo proposicional. En forma similar, la afirmación informal ¨P(x) es verdadera para alguna x¨ correspondería al inaceptado P(0)∨P(1)∨P(2)∨.... Para darle la vuelta a este problema, necesitamos dos símbolos nuevos: uno que signifique ¨para todo¨ y otro que signifique ¨para algún¨. Entonces necesitamos saber las reglas para utilizar los nuevos símbolos y combinarlos con los viejos. Este sistema de símbolos y reglas se llama cálculo de predicados. Los nuevos símbolos que introduciremos se llaman cuantificadores. Supongamos que {P(x)/x ∈ U} es una familia con índices en un conjunto U que puede se infinito; el conjunto U se llama el dominio de individuos o universo de individuos. Mediante la introducción de ¨existe ...¨ es confirmada la existencia de por lo menos un elemento del conjunto base que satisface la forma proposicional dada. Esta proposición es una proposición existencial. Proposiciones con la formulación una parte, casi todo, la mayoría, algunos, etc., son
  • 32. CAPÍTULO 1. LÓGICA MATEMÁTICA 32 también proposiciones existenciales. Cuando hablamos de proposiciones existenciales, nos referi- mos también a proposiciones particulares, ya que estas no se refieren a todos los elementos del conjunto que nos interesa, sino solo a una parte. En este caso denominamos a la cuantificación, particularidad. De forma análoga, se denomina a las proposiciones en que aparece la formulación ¨para todos¨, proposiciones universales o generales, ya que estas se refieren a todos los elementos del conjunto de variables. Tal cuantificación se denomina también generalización. La cuantificación particularidad y generalización son operaciones de la lógica de predicados. Partiendo de las formas proposicionales relacionadas previamente con los operadores, tales como ¨existe ...¨, ¨para todo ...¨, ¨no existe ningún ...¨, hemos obtenido proposiciones falsas o verdade- ras. Para estos operadores denominados también cuantificadores, se han introducido en la lógica matemática signos especiales. El cuantificador existencial (particularizador) ¨existe (por lo menos) un ...¨ es simbolizado con ?∃. Si el símbolo ∃? se encuentra ante una forma proposicional P(x), esto quiere decir que existe por lo menos un elemento del conjunto fundamental que posee la propiedad reflejada en la forma proposicional P(x). Utilizamos las escrituras ¨∃ x P(x)¨. La tachadura vertical o la relación que se establece entre el símbolo ¨∃¨ y el símbolo ¨ ∃¨, debe expresar que no existe ningún elemento del conjunto fundamental que posea la propiedad indicada en la forma proposicional P(x). El cuantificador universal (operador universal, generalizador) ¨para todo ...¨ se representa con el símbolo ¨∀¨. Si el símbolo ¨∀¨ se encuentra ante una forma proposicional P(x), esto quiere decir que la propiedad reflejada en la forma proposicional P(x) es aplicable para cada elemento del dominio de individuos. El cuantificador universal forma pareja con una variable, ∀ x, significa, ¨para todo x ...¨. La tachadura vertical o la relación que se establece entre el cuantificador universal y el símbolo ¨ ∀¨ debe expresar que la propiedad reflejada en P(x) no es aplicable para todos los elementos del dominio de individuos. La lógica de predicados o lógica de primer grado, nos enseña que para la cuantificación sólo son admisibles las variables de individuos. Las variables de individuos cuantificados dejan de ser variables libres para convertirse en variables ligadas. Para crear expresiones de la lógica de predi- cados utilizamos además de los símbolos para las variables de individuos, constantes de individuos, variables predicativas, cuantificadores y los conectores de la lógica proposicional. En la lógica proposicional comprobamos el valor de verdad de una expresión mediante la sus- titución de las variables de dicha expresión por sus valores de verdad, teniendo en cuenta las disposiciones correspondientes. El valor de verdad de una expresión de la lógica de predicados depende no solo del cuantificador sino también de las variables de individuos y del conjunto de in- dividuos tomado como base, así como de la sustitución o interpretación de las variables predicativas. A la proposición compuesta ∀ x P(x) se le asignan valores de verdad de la manera siguiente: ¨∀ x P(x) es verdadero si P(x) es verdadero para toda x en U; en cualquier otro caso ∀ x P(x) es falsa¨
  • 33. CAPÍTULO 1. LÓGICA MATEMÁTICA 33 La proposición compuesta ∃ x P(x) tiene los siguientes valores de verdad: ¨∃ x P(x) es verdadero si P(x) es verdadera para al menos una x en U; ∃ x P(x) es falso si P(x) es falsa para toda x en U¨ Analicemos la proposición ∀ x P(x) de manera más detallada. La expresión P(x) se llama predicado. Para formar una oración hay que tener un sujeto. Por ejemplo, el predicado ¨... es mas poblada que Quito¨ se transforma en la oración ¨Guayaquil es mas poblada que Quito¨ al dar co- mo sujeto Guayaquil. Si llamamos P al predicado la oración podría escribirse como P(Guayaquil). Cada sujeto da una oración. En nuestra lógica simbólica dar un predicado es establecer una función que produce una proposi- ción siempre que le demos un elemento del dominio de individuos, esto es, una función proposicional - valuada con dominio de individuos U. Seguimos nuestra práctica usual y denotamos tal función por P(x). La variable x en la expresión P(x) se llama variable libre del predicado. En tanto x varía en U los valores de verdad de P(x) pueden variar. En contraste, la proposición ∀ x P(x) tiene un significado fijo y un valor de verdad que no varía con x. La variable x en ∀ x P(x) se llama variable acotada; está acotada por el cuantificador ∀. Como ∀ x P(x) tiene un significado fijo y un valor de verdad sería inútil y poco natural cuantificarla de nuevo. Esto es, sería vano intro- ducir ∀ x[∀ x P(x)] y ∃ x[∃ x P(x)] ya que sus valores de verdad son los mismos que los de ∀ x P(x). Podemos también considerar predicados que son funciones de más de una variable, posiblemente de más de un dominio de individuos, y en tales casos el uso de varios cuantificadores resulta natural. Ejemplo 1.50 Con estos ejemplos en mente vamos a dar una descripción más detallada y formal. Sean U1, U2, ..., Un conjuntos no vacíos. Un predicado de n argumentos sobre U1 x U2 x ... x Un es una función P(x1, x2, ..., xn) con dominio de individuos U1 x U2 x ... x Un y los valores de la función son proposiciones. Las variables x1, x2, ..., xn para P(x1, x2, ..., xn) son todas variables libres para el predicado y cada xj varía en su correspondiente dominio de individuos Uj. El término ¨libre¨ es la abreviación de ¨libre para sustitución¨, queriendo decir que la variable xj está disponible en caso de que queramos sustituir un valor particular de Uj cada vez que aparezca xj. Si sustituimos xj por un valor, digamos que por ejemplo sustituimos x1 por a, en P(x1, x2, ..., xn) obtenemos el predicado P(a, x2, ..., xn) que es libre en las restantes n − 1 variables x2, ..., xn pero ya no lo es en x1. Al aplicar un cuantificador ∀xj o ∃xj a un predicado P(x1, x2, ..., xn) obtenemos un predicado ∀xj P(x1, x2, ..., xn) o ∃xj P(x1, x2, ..., xn) cuyos valores dependen únicamente de las restantes n − 1 variables. Decimos que el cuantificador liga la variable xj, haciendo que xj sea una variable acotada para el predicado. Al aplicar n cuantificadores, uno para cada variable, obtenemos que todas las variables estén acotadas y obtenemos una proposición cuyo valor de verdad puede determinarse aplicando las reglas para ∀x y ∃x, para los dominios de individuos U1, U2 , ..., Un. Ejemplo 1.51 Anteriormente notamos que un predicado de n argumentos se transforma en un predicado de (n − 1) argumentos cuando se liga una de las variables con un cuantificador. Su valor de verdad depende de los valores de verdad de las restantes (n − 1) variables libres y en particular no depende de qué nombre elijamos para llamar la variable acotada. De esta manera si P(x) es predicado de un argumento con dominio de individuos U, entonces ∀x P(x), ∀y P(y) y ∀z P(z) tienen todas el mismo valor de verdad, es decir P(n), es verdadero para toda n en U y falso en cualquier otro caso. De manera semejante, si Q(x, y) es un predicado de dos argumentos con dominio de individuos U y V , entonces ∃y Q(x, y), ∃t Q(x, t) y ∃s Q(x, s) describen todas el mismo predicado de un argumento, a saber, el predicado que es verdadero para una x dada en U si y sólo si Q(x, V ) es verdadero para alguna V en V que es el dominio de la segunda variable. Por otro lado, el predicado ∃x Q(x, x), no es el mismo que los tres últimos. La diferencia consiste en
  • 34. CAPÍTULO 1. LÓGICA MATEMÁTICA 34 que el cuantificador en este caso liga las dos variables libres. Otra práctica común es dar una descripción del dominio de individuos justo después de la variable cuantificada. Por ejemplo, en lugar de ¨sea R el dominio de individuos ... ∀x P(x)¨ podríamos escribi ∀x ∈ R P(x). De manera similar, ∃x ∈ R ∀n ∈ P(xn > x) se lee como ¨hay un número real x tal que para toda n en P, xn > x¨ o como ¨hay un número real x tal que xn > x para toda n en P¨. 1.4.1. Leyes de la lógica de predicados Las ideas de demostración y de teorema que se discutió para el cálculo proposicional, pueden extenderse al ámbito del cálculo de predicados. No es sorprendente que con más expresiones po- sibles tengamos también mayores complicaciones. Una relación moderadamente completa de este tema puede formar una parte sustancial de otro libro. En esta sección nos limitaremos a discutir algunas de las más básicas y útiles conexiones entre los cuantificadores y los operadores lógicos. En el capítulo anterior utilizamos la expresión proposición compuesta de manera informal para describir proposiciones construidas a partir de proposiciones más simples. Las leyes de la lógica de predicados que no se pueden obtener por medio de la sustitución de las leyes de la lógica proposicional, son por ejemplo: 1. ∀x P(x) → P(a) ∀x P(x) → P(a) prueba que, si cada individuo de un conjunto posee una determinada propiedad P, entonces existe también un individuo determinado a que posee esta propiedad. 2. P(a) → ∃x P(x) P(a) → ∃x P(x) prueba que, si un individuo determinado de un conjunto de individuos posee una determinada propiedad P, existe entonces, por lo menos un individuo a con esta propiedad. Toda expresión de la lógica proposicional con validez general puede convertirse en una expresión de la lógica de predicados con validez general, pero el recíproco es falso. Podríamos intentar obtener, por medio de la ssustitución de una expresión de la lógica pro- posicional satisfactible sin validez general, una expresión de la lógica de predicados igualmente satisfactible, pero sin validez general. Pongamos por ejemplo en la neutralidad de la lógica propo- sicional P ∧ Q para la variable proposicional P ∼= ∀x[P(x) ∨ ¬P(x)] y para Q ∼= ∃x[P(x) ∧ ¬P(x)] de esta forma obtenemos la expresión ∀x[P(x) ∨ ¬P(x)] ∧ Q ∼= ∃x[P(x) ∧ ¬P(x)]. Esta expresión es una contradicción. Por el contrario resulta que: ¨Toda expresión de la lógica proposicional, no ejecutable, satisfac- tible, es también una expresión de la lógica de predicados, no ejecutable, satisfactible¨. Algunas equivalencias de la lógica de predicados, que expresan la relación que se establece entre los cuantificadores ∀ y ∃ reciben especial atención. Una equivalencia de la lógica de predicados tiene
  • 35. CAPÍTULO 1. LÓGICA MATEMÁTICA 35 tanta validez general como una equivalencia de la lógica proposicional, si coinciden en cada caso los valores de verdad de ambos términos en iguales sustituciones de sus variables. Se obtiene una proposición verdadera en cada sustitución de las variables del dominio, a partir de un conjunto no vacío dado, y en cada sustitución de las variables del predicado P. Esta expre- sión es una forma, en la lógica de predicados del conocido teorema del tercer excluido de la lógica proposicional. Las identidades de la lógica de predicados (leyes) se pueden obtener de las identidades lógicas proposicionales si las variables son sustituidas por formas proposicionales de la lógica de predicados en las expresiones de la lógica proposicional correspondiente. En muchos casos nos encontramos que estas expresiones tienen que ver con formas proposicio- nales, que se han obtenido mediante la combinación de dos o más proposiciones como dos formas proposicionales. La traducción de expresiones de la lógica de predicados en el lenguaje común es generalmente más fácil que la traducción en dirección contraria. Sobre todo existen dificultades cuando se presentan, por ejemplo, dos o más operadores. Teorema 1.15 Las siguientes equivalencias son válidas: ∀x ∀y P(x, y) ∼= ∀y ∀x P(x, y) y ∃x ∃y P(x, y) ∼= ∃y ∃x P(x, y) Demostración Para demostrar que ∃x ∃y P(x, y) ∼= ∃y ∃x P(x, y) es una tautología, debemos revisar que esta proposición es verdadera para todos los dominios del discurso posibles. Por la definición de ↔, necesitamos revisar solamente que ∃y ∃x P(x, y) es verdadera para un dominio dado si y sólo si ∃x ∃y P(x, y) es verdadera para ese dominio. Supongamos que ∃x ∃y P(x, y) tiene valor verdadero. Entonces ∃y P(x0, y) es verdadera para alguna x0 en el universo, por lo tanto P(x0, y0) es verdadera para alguna y0 en el dominio. De ahí que ∃x P(x, y0) es verdadera y por lo tanto ∃y ∃x P(x, y) es verdadera. La implicación en la otra dirección es similar. Más aún, las dos proposiciones ∃x ∃y P(x, y) y ∃y ∃x P(x, y) son lógicamente equivalentes a la proposición ∃(x, y) P(x, y) donde (x, y) varía sobre D1 x D2, con D1 y D2 los dominios del discurso de las variables x e y respectivamente. Teorema 1.16 Es válida la siguiente identidad: ∃x ∃y P(x, y) ∼= ∀y ∃x P(x, y) Demostración Para poder demostrar este teorema, asumimos que si la parte izquierda de esta proposición es verdadero entonces existe x0 en el dominio de discurso tal que ∀y P(x0, y) es verdadero y así P(x0, y) es verdadero para toda y. Por lo tanto, para cada y, ∃x P(x, y) es verdadero; de hecho la misma x0 sirve para cada y. Como ∃x P(x, y) es verdadero para toda y, el lado derecho de la proposición tiene valor de verdad verdadero. De esta manera la proposición es una tautología. Por otra parte el recíproco de esta proposición, es decir ∀y ∃x P(x, y) ∼= ∃x ∀y P(x, y) no es en general verdadero. Para enfatizar la diferencia, supongamos que x e y varían sobre un dominio D de tres elementos, digamos D = {a, b, c}. El predicado de 2 argumentos P(x, y) tiene nueve posibles valores; P(a, a); P(a, b); P(a, c); P(b, a); P(b, b); P(b, c); P(c, a); P(c, b); P(c, c).
  • 36. CAPÍTULO 1. LÓGICA MATEMÁTICA 36 Entonces ∃x ∀y P(x, y) es verdadero si ∀y P(x0, y) es verdadero para alguna x0. Como x0 tiene que ser igual a a, b o c vemos que ∃x ∀y P(x, y) es verdadero si y sólo si todas las proposiciones de una de las filas dadas arriba son verdaderas. En contraste, ∀y ∃x P(x, y) sería verdadera siempre que al menos una proposición de cada columna sea verdadera. Por ejemplo si consideramos un predicado P(x, y) con valores de verdad P(a, a) P(a, b) P(a, c) P(b, a) P(b, b) P(b, c) P(c, a) P(c, b) P(c, c) V F F F F V F V V entonces ∀y ∃x P(x, y) será verdadera en tanto que ∃x ∀y P(x, y) será falsa. Para esta elección de predicado P(x, y), ∃x P(x, y) es verdadera para toda y pero la x adecuada depende de la y, ninguna x única sirve para toda y. Teorema 1.17 Las identidades siguientes son válidas: ¬∀x P(x) ∼= ∃x [¬P(x)]; ¬∃x P(x) ∼= ∀x [¬P(x)]; ∀x P(x) ∼= ¬∃x [¬P(x)]; ∃x P(x) ∼= ¬∀x [¬P(x)]. Ejemplo 1.52 Las leyes de DeMorgan pueden utilizarse repetidamente para negar cualquier proposición cuantificada ¬∃w ∀x ∃y ∃z P(w, x, y, z) es sucesivamente lógica equivalente a ∀w[¬∀x ∃y ∃z P(w, x, y, z)]; ∀w ∃x[¬∃y ∃z P(w, x, y, z)]; ∀w ∃x ∀y[¬∃z P(w, x, y, z)]; ∀w ∃x ∀y ∀z[¬P(w, x, y, z)]; Esto ilustra la regla general: La negación de un predicado cuantificado es lógicamente equiva- lente a la proposición que se obtiene al sustituir cada ∀ por ∃ y cada ∃ por ∀ y reemplazando el mismo predicado por su negación. Ejemplo 1.53 La negación de ∀x ∀y ∃z (x < z < y) es ∃x ∃y ∀z [¬(x < z < y)]. Aplicando las leyes de DeMorgan vemos que la negación es lógicamente equivalente a ∃x ∃y ∀z [(z ≤ x) ∨ (z ∧ y)] Ejemplo 1.54 La negación de ∀x ∀y (x < y → x2 < y2 ) es ∃x ∃y [¬(x < z → x2 < y2 )]. Aplicando las leyes de DeMorgan vemos que la negación es lógicamente equivalente a ∃x ∃y [(x < y) ∧ (x2 ≥ y2 )]
  • 37. CAPÍTULO 1. LÓGICA MATEMÁTICA 37 1.4.2. Interpretación de fórmulas en la lógica de predicados En la lógica proposicional una interpretación es una asignación de valores de verdad a átomos. En la lógica de predicados, puesto que hay variables involucradas, hay que hacer más que eso. Para definir una interpretación para una fórmula en la lógica de predicados, tenemos que especificar dos cosas, el dominio y una asignación a constantes, símbolos de función y símbolos de predicado que ocurren en la fórmula. A continuación se da la definición formal de interpretación de una fórmula en la lógica de predicados. Definición 1.24 Interpretación de una fórmula Una interpretación de una fórmula G en la lógica de predicados, consicte de un dominio D no vacío, y una asignación de valores a cada constante, símbolos de función, y símbolos de predicado que ocurre en G de la siguiente manera: 1. A cada constante asignamos un elemento en D; 2. A cada símbolo de función asignamos una aplicación de Dn a D, Dn = {x1, x2, ..., xn ∈ D} 3. A cada símbolo de predicado asignamos una aplicación de Dn a {V, F}. Algunas veces para enfatizar el dominio D, hablaremos de una interpretación de la fórmula sobre D. Cuando evaluamos el valor de verdad de una fórmula en una interpretación sobre el dominio D, ∀x será interpretada como ¨para todos los elementos x en D¨, y ∃x como ¨hay un elemento en D¨. Para cada interpretación de una fórmula sobre un dominio de individuos D, la fórmula puede ser evaluada a V o F de acuerdo a las siguientes reglas: 1. Si los valores de verdad de las fórmulas H y G son evaluadas, entonces los valores de verdad de las fórmulas ¬H, H ∨ G, H ∧ G, H → G, H ↔ G son evaluadas de la siguiente manera: H G ¬ H H ∨ G H ∧ G H → G H ↔ G V V F V V V V V F F V F F F F V V V F V F F F V F F V V 2. ∀x H es evaluada a V si el valor verdadero de H es valuado a V para cada d ∈ D, de otra manera es evaluado a F. 3. ∃x H es evaluado a V si el valor de verdad de H es V para por lo menos un d ∈ D, de otra manera es evaluada a F. Se puede notar fácilmente que cualquier fórmula conteniendo variables libres no puede ser evaluada. En adelante asumiremos, ya sea que las fórmulas no contienen variables libres o que las variables son tratadas como constantes. Ejemplo 1.55 Considere la fórmula ∀x ∃y P(x, y), D = {1, 2} P(1, 1) ∼= V ; P(1, 2) ∼= F; P(2, 1) ∼= F; P(2, 2) ∼= V. Si x = 1, podemos ver que hay un y tal que P(1, y) es verdadero. Si x = 2 hay también un y denominado 2 tal que P(2, y) es verdadero, por consiguiente en las interpretaciones de arriba, para cada x en D hay un y tal que P(x, y) es verdadero, esto es ∀x ∃y P(x, y) es verdadero en esta interpretación.
  • 38. CAPÍTULO 1. LÓGICA MATEMÁTICA 38 Ejemplo 1.56 Considere la fórmula ∀x [P(x) → Q(f(x), k)]. Hay una constante k, un símbolo de función f de un lugar, un símbolo de predicado P de un lugar, y un símbolo de predicado Q de dos lugares. La siguiente es una interpretación I. Dominio D = {1, 2}. Asignación para k: a = 1. Asignación para f: f(1) = 2; f(2) = 1. Asignaciones para P y Q: P(1) = F; P(2) = V ; Q(1, 1) = V ; Q(1, 2) = V ; Q(2, 1) = F; Q(2, 2) = V. Si x = 1, entonces P(x) → Q(f(x), k) = P(1) → Q(f(1), k) = P(1) → Q(2, 1) = F → F = V. Si x = 2, entonces P(x) → Q(f(x), k) = P(2) → Q(f(2), k) = P(2) → Q(1, 1) = V → V = V. Puesto que P(x) → Q(f(x), k) es verdadero para todos los valores de x en D, la fórmula ∀x [P(x) → Q(f(x), k)] es verdadera bajo las interpretaciones I. Ejemplo 1.57 Evaluar los valores de verdad de las siguientes fórmulas bajo las interpretacio- nes dadas en el ejemplo anterior. 1. ∃x [P(f(x)) ∧ Q(x, f(k))]; 2. ∃x [P(x) ∧ Q(x, k)]; 3. ∀x ∃y [P(x) ∧ Q(x, y)]. Para 1): Si x = 1, entonces P(f(x)) ∧ Q(x, f(k)) = P(f(1)) ∧ Q(1, f(1)) = P(2) ∧ Q(1, f(1)) = P(2) ∧ Q(1, 2) = V ∧ V = V. Si x = 2, entonces P(f(x)) ∧ Q(x, f(k)) = P(f(2)) ∧ Q(2, f(1)) = P(1) ∧ Q(2, 1) = F ∧ F = F. Puesto que hay un elemento en el dominio D, esto es x = 1 tal que P(f(x))∧Q(x, f(k)) es verdadero, el valor de verdad de la fórmula ∃x [P(f(x)) ∧ Q(x, f(k))] es verdadera bajo la interpretación I. Para b): Si x = 1, entonces P(x) ∧ Q(x, k) = P(1) ∧ Q(1, 1) = F ∧ V = F. Si x = 2, entonces P(x) ∧ Q(x, k) = P(2) ∧ Q(2, 1) = V ∧ F = F. Puesto que no hay elemento en el dominio D tal que P(x) ∧ Q(x, k) sea verdadero, la fórmula ∃x [P(x) ∧ Q(x, k)] es evaluada a falsa bajo la interpretación I. Para c): Si x = 1, entonces P(x) = P(1) = F. Por consiguiente P(x) ∧ Q(x, y) = F para y = 1 e y = 2. Puesto que existe un x, que es x = 1, la fórmula ∃y [P(x)∧Q(x, y)] es falsa, la fórmula ∀x ∃y [P(x)∧Q(x, y)] es falsa bajo la interpretación I, esto es, la fórmula es falsificada por I.
  • 39. CAPÍTULO 1. LÓGICA MATEMÁTICA 39 Definición 1.25 Fórmula consistente Una fórmula G es consistente (satisfactible) si y sólo si existe una interpretación I tal que G es evaluada verdadero en I. Si una fórmula G es verdadera en una interpretación I, decimos que I es un modelo de G e I satisface a G. Definición 1.26 Fórmula válida Una fórmula G es válida si, y sólo si cada interpretación de G satisface a G. Definición 1.27 Fórmula inconsistente Una fórmula G es inconsistente (insatisfactible) si y sólo si, no existe una interpretación que satisface a G. Las relaciones entre validez (inconsistencia) y consecuencias lógicas, como se indica en la lógica proposicional, son también verdaderas para la lógica de predicados. En efecto, la lógica de predica- dos puede ser considerada como una extensión de la lógica proposicional. Cuando una fórmula en la lógica de predicados no contiene variables y cuantificadores, puede ser tratada justo como una fórmula en la lógica proposicional. Ejemplo 1.58 1. ∀x P(x) ∧ ∃y ¬P(y) es inconsistente; 2. ∀x P(x) → ∃y P(y) es válido; 3. P(k) → ¬∃x P(x) es consistente; 4. ∀x P(x) ∨ ∃y ¬P(y) es válido. En la lógica de predicados, puesto que hay un número infinito de elementos en el dominio D, en general, hay un número infinito de interpretaciones de una fórmula. Por consiguiente al contrario de la lógica proposicional, no es posible verificar la validez e inconsistencia de una fórmula, evaluando la fórmula bajo todas las posibles interpretaciones. 1.4.3. Forma normal prenexa En la lógica proposicional hemos introducido dos formas normales, la forma normal conjuntiva y la forma normal disjuntiva. En la lógica de predicados hay una forma normal llamada forma normal Prenexa. La razón para considerar una forma normal Prenexa de una fórmula es simplificar procedimientos de prueba. Definición 1.28 Forma normal prenexa Una fórmula G en la lógica de predicados se dice que es una forma normal Prenexa si y sólo si, la fórmula G está en la forma (Q1x1)(Q2x2)...(Qnxn)(M) donde cada (Qixi), i = 1, 2, ..., n ya sea ∀xi o ∃xi, y M es una fórmula que no contiene cuan- tificadores, (Q1x1)(Q2x2)...(Qnxn) es llamada el prefijo y M es llamada la matriz de la fórmula G. Dada una fórmula G, consideraremos un método de transformarla en una forma normal Prene- xa. Esto se logra primero considerando algunos pasos básicos de fórmulas equivalentes en la lógica de predicados. Recordemos que dos fórmulas G y H son equivalentes si, y sólo si los valores de verdad de G y H son los mismos bajo cada interpretación.
  • 40. CAPÍTULO 1. LÓGICA MATEMÁTICA 40 Los pares básicos de fórmulas equivalentes dadas en la lógica proposicional son todavía verdad para la lógica de predicados, adicionalmente hay otros pares de fórmulas equivalentes conteniendo cuantificadores, y que se estudiaron en secciones anteriores. Consideraremos estos pares adicionales de fórmulas equivalentes. Sea G una fórmula que contiene una variable libre x, para enfatizar que la variable libre está en G, representamos G por G[x]. Sea H una fórmula que no contiene variable x, tenemos los siguientes pares de fórmulas equivalentes, donde Q es ya sea ∀ o ∃: 1. (Qx)G[x] ∨ H ∼= (Qx)(G[x] ∨ H); 2. (Qx)G[x] ∧ H ∼= (Qx)(G[x] ∧ H); 3. ¬(∀xG[x]) ∼= ∃x(¬G[x]); 4. ¬(∃xG[x]) ∼= ∀x(¬G[x]). Las leyes 1 y 2 son obviamente verdaderas puesto que H no contiene x, por consiguiente puede ser introducida en el alcance del cuantificador Q. Las leyes 3 y 4 no son difíciles de probar. Sea I cualquier interpretación arbitraria sobre el dominio D. Si ¬(∀x G[x]) es verdadera en I, entonces ∀x G[x] es falsa en I. Esto significa que hay un elemento d en D tal que G[d] es falso. Esto es ¬G[d] es verdadero en I. Por consiguiente, ∃x (¬G[x]) es verdadera en I. Por otra parte si ¬(∀x G[x]) es falsa en I, entonces ∀x G[x] es verdadera en I. Esto significa que G[x] es verdadera para cada elemento x en D, esto es ¬G[x] es falso para cada elemento x en D, por consiguiente, ∀x (¬G[x]) es falsa en I. Puesto que ¬(∀x G[x]) y ∀x (¬G[x]) siempre asume el mismo valor de verdad para cada interpretación arbitraria, por definición, ¬(∀x G[x]) ∼= ∃x (¬G[x]). Así la ley 3 es probada e igualmente podemos probar la ley 4. Supongamos que F[x] y G[x] son dos fórmulas que contienen x, 5. ∀x F[x] ∧ ∀x G[x] ∼= ∀x (F[x] ∧ G[x]) 6. ∃x F[x] ∨ ∃x G[x] ∼= ∃x (F[x] ∨ G[x]) Esto es, el cuantificador universal ∀ y el existencial ∃, pueden distribuirse sobre ∧ y ∨, respectivamente. Sin embargo el cuantificador universal y existencial no pueden distribuirse sobre ∨ y ∧ respectivamente. Esto es ∀x F[x] ∨ ∀x G[x] = ∀x (F[x] ∨ G[x]) ∃x F[x] ∧ ∃x G[x] = ∃x (F[x] ∧ G[x]) Para casos como estos tenemos que hacer algo especial. Puesto que cada variable ligada en una fórmula puede ser considerada como una variable renombrable, cada variable x puede ser renombrada z, y la fórmula ∀x G[x] se transforma en ∀z G[z]. Supongamos que escogemos la variable z que no aparece en F[x]. Entonces ∀x F[x] ∨ ∀x G[x] ∼= ∀x F[x] ∨ ∀z G[z] ∼= ∀x∀z (F[x] ∨ G[z]) Similarmente, renombrando todas las x que ocurren en ∃x G[x] como z, podemos tener ∃x F[x] ∧ ∃x G[x] ∼= ∃x F[x] ∧ ∃z G[z] ∼= ∃x∃z (F[x] ∧ G[z])
  • 41. CAPÍTULO 1. LÓGICA MATEMÁTICA 41 Por consiguiente, para estos dos casos podemos todavía pasar todos los cuantificadores a la izquierda de la fórmula. En general, tenemos 7. (Q1x)F[x] ∨ (Q2x)G[x] ∼= (Q1x)(Q2x)(F[x] ∨ G[z]) 8. (Q3x)F[x] ∧ (Q4x)G[x] ∼= (Q3x)(Q4x)(F[x] ∧ G[z]) donde Q1, Q2, Q3 y Q4 son ya sea ∀ o ∃, y z no aparece en F[x]. Naturalmente si Q1 = Q2 = ∃ y Q3 = Q4 = ∀, entonces no tenemos que renombrar las x en (Q2x)G[x] o (Q4x)G[x]. Podemos usar las leyes 5 y 6 directamente. Usando las leyes de la lógica proposicional y las leyes 1 - 8, podemos siempre transformar una fórmula dada en forma normal Prenexa. La siguiente es una guía del procedimiento de transformación: PASO 1: Use las leyes 1. F ↔ G ∼= (F → G) ∧ (G → F); 2. F → G ∼= ¬F ∨ G; Para eliminar las conectividades lógicas ↔ y →. PASO 2: Repetidamente use las leyes 3. ¬(¬F) ∼= F; 4. ¬(F ∨ G) ∼= ¬F ∧ ¬G; 5. ¬(G ∧ G) ∼= ¬F ∨ ¬G; 6. ¬(∀x F[x]) ∼= ∃x (¬F[x]); 7. ¬(∃x F[x]) ∼= ∀x (¬F[x]); para traer los signos de negación inmediatamente antes de los átomos. PASO 3: Renombrar las variables ligadas si es necesario. PASO 4: Use las leyes 8. (Qx)F[x] ∨ G ∼= (Qx)(F[x] ∨ G); 9. (Qx)F[x] ∧ G ∼= (Qx)(F[x] ∧ G); 10. ∀x F[x] ∧ x G[x] ∼= ∀x (F[x] ∧ G[x]); 11. ∃x F[x] ∨ ∃x G[x] ∼= ∃x (F[x] ∨ G[x]); 12. (Q1x)F[x] ∨ (Q2x)G[x] ∼= (Q1x)(Q2x)(F[x] ∨ G[z]); 13. (Q3x)F[x] ∧ (Q4x)G[x] ∼= (Q3x)(Q4x)(F[x] ∧ G[z]). para mover los cuantificadores a la izquierda de la fórmula y obtener una forma normal Prenexa.
  • 42. CAPÍTULO 1. LÓGICA MATEMÁTICA 42 Ejemplo 1.59 Transformar la fórmula ∀x P(x) → ∃x Q(x) en forma normal prenexa. Solución ∀x P(x) → ∃x Q(x) ∼= ¬∀x P(x) ∨ ∃x Q(x) ∼= ∃x¬P(x) ∨ ∃x Q(x) ∼= ∃x [¬P(x) ∨ Q(x)]. Ejemplo 1.60 Transformar la fórmula ∀x ∀y {∃z [P(x, z) ∧ P(y, z)] → ∃u Q(x, y, u)} en forma normal Prenexa. Solución ∀x ∀y{∃z [P(x, z) ∧ P(y, z)] → ∃u Q(x, y, u)} ∼= ∀x ∀y{¬∃z [P(x, z) ∧ P(y, z)] ∨ ∃u Q(x, y, u)} ∼= ∀x ∀y{∀z ¬[P(x, z) ∧ P(y, z)] ∨ ∃u Q(x, y, u)} ∼= ∀x ∀y ∀z ∃u {¬P(x, z) ∨ ¬P(y, z) ∨ ∃u Q(x, y, u)}. 1.4.4. Tarea 1. Sea A = {1, 2, 3, 4} el conjunto universal. Determine el valor de verdad de cada enunciado: a) ∀x : x + 3 < 6; b) ∀x : x2 − 10 ≤ 8; c) ∃x : x2 > 1 → x + 2 = 0; d) ∃x : 2x2 + x = 15. Resp: a) Falso; b) Verdadero; c) Verdadero; d) Falso. 2. Determine el valor de verdad de las siguientes proposiciones, siendo N el universo: a) ∀x ∃y (2y = x); b) ∃y ∀x (2x = y); c) ∀x ∃y (2x = y); d) ∃y ∀x (2y = x); e) ∀x ∀y [¬(2y = x)]. Resp: a) ; b) ; c) ; d) ; e) . 3. Determine el valor de verdad de las siguientes proposiciones, siendo R el universo: a) ∀x ∃y (xy = 1); b) ∀x ∀y [(x + y)2 = x2 + y2 ]; c) ∃x ∃y (x2 + y2 + 1 = 2xy); d) ∃x ∃y [(x + 2y = 4) ∧ (2x − y = 2)]. Resp: a) ; b) ; c) ; d) . 4. Determine el valor de verdad de las siguientes proposiciones, siendo R el universo: a) ∀x ∈ R x2 ≥ x; b) ∃x ∈ R 2x = x; c) ∀x ∈ R 2x−1 4x−2 = 1 2 ; d) ∃x ∈ R x2 + 2x + 1 ≤ 0. Resp: a) ; b) ; c) ; d) . 5. Negar los siguientes enunciados: a) ∃y p(y) → ∀x(¬q(x)); b) ∃x(¬p(x)) ∨ ∀x q(x); c) ∃x ∃y (p(x, y) → q(x, y)). Resp: a) ∃y p(y) ∧ ∃x q(x); b) ∀x p(x) ∧ ∃x(¬q(x)); c) ∀ ∃y(p(x, y) ∧ ¬q(x, y)). 6. Negar las siguientes afirmaciones: a) ∀x ∀y [(x + y es impar) → (x es impar ∨ y es impar)]; b) ∀x ∃y (x + y = 5 → y = −x); c) ∃x ∀y (x < y ∧ x2 ≥ y); d) ∀x ∀y ∃z (x < y → x + z = y). Resp: a) ; b) ; c) ; d) . 7. Averiguar el valor de verdad siendo U = R: a) ∀x ∈ R (x < 0 → x < 3); b) ∃x ∈ R (x2 ≥ 0 → x4 = x3 ); c) ∀x ∈ R, ∃y ∈ R (x2 + y2 = 1); d) ∀x ∈ R, ∀y ∈ R (y < x → 2y < 10). Resp: a) Verdadero; b) Verdadero; c) Falso; d) Falso.